SlideShare a Scribd company logo
1 of 156
MedicalResearch.com
Exclusive Interviews with Medical Research and
Health Care Researchers from Major and Specialty Medical
Research Journals and Meetings
Editor: Marie Benz, MD
info@medicalresearch.com
August 24 2015
For Informational Purposes Only: Not for Specific Medical Advice.
Medical Disclaimer | Terms and Conditions
• The contents of the MedicalResearch.com Site, such as text, graphics, images, and
other material contained on the MedicalResearch.com Site ("Content") are for
informational purposes only. The Content is not intended to be a substitute for
professional medical advice, diagnosis, or treatment. Always seek the advice of
your physician or other qualified health provider with any questions you may have
regarding a medical condition. Never disregard professional medical advice or
delay in seeking it because of something you have read on the Hemodialysis.com
Site!
• If you think you may have a medical emergency, call your doctor or 911
immediately. MedicalResearch.com does not recommend or endorse any specific
tests, physicians, products, procedures, opinions, or other information that may be
mentioned on the Site. Reliance on any information provided by
MedicalResearch.com or other Eminent Domains Inc (EDI) websites, EDI
employees, others appearing on the Site at the invitation of MedicalResearch.com
or EDI, or other visitors to the Site is solely at your own risk.
• The Site may contain health- or medical-related materials that are sexually explicit.
If you find these materials offensive, you may not want to use our Site. The Site
and the Content are provided on an "as is" basis.
Read more interviews on
MedicalResearch.com
Hospital Improvements Have Led To Marked Decreased In Inpatient Mortality Over Ten Years
Posted on August 16, 2015
MedicalResearch.com Interview with:
Kevin Heslin, Ph.D., Staff Service Fellow, Center for Delivery, Organization and Markets, Agency for Healthcare Research and
Quality
Medical Research: What is the background for this study?
Dr. Heslin: Previous trends in inpatient mortality suggest that rates have been decreasing for
acute myocardial infarction (AMI), congestive heart failure (CHF), stroke, and pneumonia (Hines
et al., 2010; Stepanova et al., 2013). Continued study of these trends can help researchers and
policymakers assess the impact of health care quality efforts. Further, examining trends across
patient and hospital subgroups may inform strategies for addressing disparities in health care
quality by identifying groups that are leading and lagging in improvement.
Read the rest of the interviews on MedicalResearch.com
Content NOT an endorsement of efficacy and NOT intended as specific medical advice.
Hospital Improvements Have Led To Marked Decreased In Inpatient Mortality Over Ten Years
Posted on August 16, 2015
MedicalResearch.com Interview with:
Kevin Heslin, Ph.D., Staff Service Fellow, Center for Delivery, Organization and Markets, Agency for Healthcare Research and
Quality
• Medical Research: What is the background for this study
• Dr. Heslin: From 2002 to 2012, inpatient mortality decreased among patients admitted to
U.S. hospitals for pneumonia (45 percent decrease, from 65.0 to 35.8 deaths per 1,000
admissions), AMI (41 percent decrease, from 94.0 to 55.9 deaths per 1,000 admissions), CHF
(29 percent decrease, from 44.4 to 31.4 deaths per 1,000 admissions), and stroke (27 percent
decrease, from 112.6 to 82.6 deaths per 1,000 admissions). The inpatient mortality rate for
all four conditions decreased among both younger and older patients, and among men and
women.
Read the rest of the interviews on MedicalResearch.com
Content NOT an endorsement of efficacy and NOT intended as specific medical advice.
Hospital Improvements Have Led To Marked Decreased In Inpatient Mortality Over Ten Years
Posted on August 16, 2015
MedicalResearch.com Interview with:
Kevin Heslin, Ph.D., Staff Service Fellow, Center for Delivery, Organization and Markets, Agency for Healthcare Research and
Quality
• Medical Research: What should clinicians and patients take away from your report?
• Dr. Heslin: The reductions in inpatient mortality rates reported by this brief suggest that
hospitals are improving care for these conditions. This explanation is supported by other
work on the quality of inpatient care for AMI, CHF, and pneumonia. Specifically, Nuti and
colleagues (2015) analyzed data on more than 4,000 hospitals over the 2006-2011 period and
ranked hospitals by their performance on evidence-based process of care measures, with a
quality score of 100 being equal to perfect performance. Over this period, hospital
performance on all measures improved. There was a decreasing difference between the
quality scores of hospitals ranked at the 90th percentile and those of hospitals ranked 10th
percentile. The variation between hospitals at these percentiles decreased from a 9% to 2%
absolute difference for AMI, a 25% to 8% difference for congestive heart failure, and a 20% to
7% difference for pneumonia in 2006 and 2011, respectively.
Read the rest of the interviews on MedicalResearch.com
Content NOT an endorsement of efficacy and NOT intended as specific medical advice.
Hospital Improvements Have Led To Marked Decreased In Inpatient Mortality Over Ten Years
Posted on August 16, 2015
MedicalResearch.com Interview with:
Kevin Heslin, Ph.D., Staff Service Fellow, Center for Delivery, Organization and Markets, Agency for Healthcare Research and
Quality
• Medical Research: What recommendations do you have for future research as a result of
this study?
• Dr. Heslin: Future research to better ascertain the reasons for these reductions in inpatient
mortality could inform not only improvements in the clinical management of these
conditions, but also the development and modification of relevant health policies. Given the
dramatic decline in the number of Americans without health insurance since 2013 (Cohen
and Martinez, 2015), one of the most important areas for future research is to assess the
impact of the insurance coverage expansions made possible by the Affordable Care Act on
hospitalization rates and outcomes for these four conditions.
• References
• Cohen RA, Martinez ME. Health Insurance Coverage: Early Release of Estimates from the
National Health Interview Survey, January–March 2015. Centers for Disease Control and
Prevention, National Center for Health Statistics, National Health Interview Survey, 2010–
2015.
Read the rest of the interviews on MedicalResearch.com
Content NOT an endorsement of efficacy and NOT intended as specific medical advice.
MMR Vaccination May Not Be Reliable in HIV+ Children
MedicalResearch.com Interview with: George K Siberry, MD, MPH, Medical Officer
Maternal and Pediatric Infectious Disease (MPID) Branch
Eunice Kennedy Shriver National Institute of Child Health and Human Development National Institutes of Health
Bethesda, MD
• Medical Research: What is the background for this study?
• Dr. Siberry: Vaccines may not work as reliably in children with HIV infection, especially when
their HIV is not under effective treatment. Today, most children in the United States who
were born with HIV infection are receiving effective HIV treatment and have reached school
age or even young adulthood. However, many received their childhood vaccines before they
got started on their HIV treatment (because modern HIV treatments weren’t available when
they were very young or their HIV infection was diagnosed late). So we wanted to see if these
older children still had immunity from the vaccines they received when they were much
younger.
Read the rest of the interviews on MedicalResearch.com
Content NOT an endorsement of efficacy and NOT intended as specific medical advice.
MMR Vaccination May Not Be Reliable in HIV+ Children
MedicalResearch.com Interview with: George K Siberry, MD, MPH, Medical Officer
Maternal and Pediatric Infectious Disease (MPID) Branch
Eunice Kennedy Shriver National Institute of Child Health and Human Development National Institutes of Health
Bethesda, MD
• Medical Research: What are the main findings?
• Dr. Siberry: We looked specifically at whether older children with HIV since birth were
protected against measles, mumps, and rubella, the three viral infections covered by the
measles-mumps-rubella (or MMR) vaccine. We found that 1/3 up to almost 1/2 of these
children were not protected against these viruses, even though nearly all of the children had
received at least 2 MMR doses, as recommended. And even if their HIV was currently under
excellent control. When we analyzed factors that were linked to being protected, we found
that one of the most important factors was whether you got your MMR vaccine
doses after you got on good treatment for your HIV infection. For instance, over 85% of
children who had gotten at least 2 MMR vaccine doses after being on effective HIV treatment
were protected against measles compared to less than half of those who didn’t get both of
their MMR vaccine doses while on effective HIV treatment.
Read the rest of the interviews on MedicalResearch.com
Content NOT an endorsement of efficacy and NOT intended as specific medical advice.
MMR Vaccination May Not Be Reliable in HIV+ Children
MedicalResearch.com Interview with: George K Siberry, MD, MPH, Medical Officer
Maternal and Pediatric Infectious Disease (MPID) Branch
Eunice Kennedy Shriver National Institute of Child Health and Human Development National Institutes of Health
Bethesda, MD
• Medical Research: What should clinicians and patients take away from your report?
• Dr. Siberry: For clinicians caring for people (children and young adults) who were born with
HIV infection, it is important to review each patient’s medical record to determine the dates
the MMR vaccines were given and the date when effective HIV treatment was started. If 2
MMR vaccines were given at least 3 months after effective HIV treatment was started, then
that patient should be protected against measles, mumps and rubella. But MMR vaccine
doses given prior to at least 3 months of effective HIV treatment should be repeated now
(while the patient is on effective HIV treatment) to make sure the patient is protected against
these viral infections. This practice is consistent with the current recommendations from the
CDC for measles immunization in people born with HIV infection.
Read the rest of the interviews on MedicalResearch.com
Content NOT an endorsement of efficacy and NOT intended as specific medical advice.
MMR Vaccination May Not Be Reliable in HIV+ Children
MedicalResearch.com Interview with: George K Siberry, MD, MPH, Medical Officer
Maternal and Pediatric Infectious Disease (MPID) Branch
Eunice Kennedy Shriver National Institute of Child Health and Human Development National Institutes of Health
Bethesda, MD
• Medical Research: What recommendations do you have for future research as a result of
this study?
• Dr. Siberry: We noted that the levels of protection were much higher (e.g., 85% for measles)
in HIV-infected children who got their MMR doses while on effective HIV treatment but still
not as high as the >95% of healthy children protected after 2 doses of MMR. We would like to
find strategies that further improve MMR vaccine response in HIV-infected children.
• We are also interested in determining if this approach for determining which HIV-infected
children need additional MMR vaccine doses may also apply to other childhood vaccines.
• Citation:
• George K. Siberry, Kunjal Patel, William Bellini, Brad Karalius, Murli Purswani, Sandra K.
Burchett, William A. Meyer III, Sun Bae Sowers, Angela Ellis,and Russell B. Van Dyke For the
Pediatric HIV AIDS Cohort Study (PHACS)
• Immunity to Measles, Mumps and Rubella in US Children with Perinatal HIV Infection or
Perinatal HIV Exposure without Infection
• Clin Infect Dis. 2015 Jun 9. pii: civ440. [Epub ahead of print]
Read the rest of the interviews on MedicalResearch.com
Content NOT an endorsement of efficacy and NOT intended as specific medical advice.
Biopolymers from Algae May Provide Broad Spectrum Photoprotection
Susana C. M. Fernandes, PhD Researcher (Individual Marie Curie Fellowship – IEF) and
Professor Vincent Bulone
• MedicalResearch: What is the background for this study? What are the main findings?
• Response: We have exploited unique properties of natural compounds to develop novel
materials that are capable of absorbing both UV-A and UV-B radiations. The active UV-
absorbing molecules are known as mycosporines and mycosporines like-amino acids and they
occur in different organisms such as algae, photosynthetic bacteria (cyanobacteria) and some
fish species that thrive in, e.g., the tropical waters of the Great Barrier Reef. These
compounds were combined with a carbohydrate polymer found in the shells of crustaceans,
the exoskeleton of insects and the cell walls of fungi. Chitosan provided a matrix on which
mycosporines were attached using a simple chemical method already used for other
purposes in, e.g., the pharmaceutical industry. Chitosan can typically be extracted from food
waste such as the shells of shrimps. The immobilization of mycosporines on chitosan allowed
the development of unique materials that have many potential applications relevant to a
wide range of sectors, including cosmetics, sunscreen creams, wound dressings, plasticizers
in paints and varnishes, coatings of outdoor furniture and other materials such as fabrics for
shades, textiles, car dashboards, etc. In addition to being highly efficient for protection
against UV-A and UV-B, the materials were shown to be photostable, thermoresistant and
biocompatible. Compared to existing sunblock formulations, the materials have no
detrimental effects on health and the environment. They are also fully recyclable.
Read the rest of the interviews on MedicalResearch.com
Content NOT an endorsement of efficacy and NOT intended as specific medical advice.
Biopolymers from Algae May Provide Broad Spectrum Photoprotection
Susana C. M. Fernandes, PhD Researcher (Individual Marie Curie Fellowship – IEF) and
Professor Vincent Bulone
• MedicalResearch: What should clinicians and patients take away from your report?
• Response: The materials we have generated can be applicable to different biomedical areas.
They are compatible with cell proliferation and adhesion with no cytotoxic or allergenic
effect. They are therefore ideal for human use and can be exploited for tissue regeneration in
humans. Biomedical applications comprise the engineering of artificial skin, contact lenses,
artificial cornea, and so on. They can be further modified by other bioactive compounds to
generate multifunctional materials in different forms, that is, films, sprays, solutions,
hydrogels and 3D scaffolds. The materials we have generated are highly relevant to patients
with photopathologies (e.g., albinos) in which the lack of skin pigmentation makes the
organism more susceptible to sunburn and skin cancers.
Read the rest of the interviews on MedicalResearch.com
Content NOT an endorsement of efficacy and NOT intended as specific medical advice.
Biopolymers from Algae May Provide Broad Spectrum Photoprotection
Susana C. M. Fernandes, PhD Researcher (Individual Marie Curie Fellowship – IEF) and
Professor Vincent Bulone
• MedicalResearch: What recommendations do you have for future research as a result of
this study?
• Response: Development toward commercialization requires the actual assessment of more
developed materials for their specific targeted applications. For instance, in the next phase,
we will prepare physical creams, hydrogels and other forms of the materials, and evaluate
the potential for commercialization by further assessing their advantages over existing
materials through experiments specifically relevant to the desired applications.
• The concept will be extended to the modification of other biopolymers and to the generation
of multifunctional materials, thereby increasing the range of products and potential areas of
application.
• Citation:
• Exploiting Mycosporines as Natural Molecular Sunscreens for the Fabrication of UV-
Absorbing Green Materials ACS Appl. Mater. Interfaces, 2015, 7 (30), pp 16558–16564
• DOI: 10.1021/acsami.5b04064
• Susana C. M. Fernandes, Ana Alonso-Varona , Teodoro Palomares , Verónica Zubillaga , Jalel
Labidi ‡, and Vincent Bulone
Read the rest of the interviews on MedicalResearch.com
Content NOT an endorsement of efficacy and NOT intended as specific medical advice.
Hospital Mortality Outcomes Differ After Lung Cancer Surgery Complications
MedicalResearch.com Interview with: Tyler Grenda, MD House Officer VI Section of General Surgery Department of Surgery
University of Michigan
• Medical Research: What is the background for this study? What are the main findings?
• Dr. Grenda: The main purpose for this study was to better understand the factors underlying
differences in mortality rates for hospitals performing lung cancer resection. The
methodology we used included only the highest and lowest mortality hospitals (Commission
on Cancer accredited cancer programs) so the sampling frame was specific. There are wide
variations in mortality rates across hospitals performing lung cancer resection (overall
unadjusted mortality rates were 10.8% vs. 1.6%, respectively.
Read the rest of the interviews on MedicalResearch.com
Content NOT an endorsement of efficacy and NOT intended as specific medical advice.
Hospital Mortality Outcomes Differ After Lung Cancer Surgery Complications
MedicalResearch.com Interview with: Tyler Grenda, MD House Officer VI Section of General Surgery Department of Surgery
University of Michigan
• Medical Research: What should clinicians and patients take away from your report?
• Dr. Grenda: To better understand the factors that may underlie differences in mortality rates,
we examined adherence to perioperative processes of care (e.g. adherence to venous
thromboembolism prophylaxis) and outcomes following surgery between high mortality
hospitals (HMHs) and low mortality hospitals (LMHs). While there were variations in
adherence to some evidence-based perioperative processes of care, those variations did not
translate to differences in directly related complications. In fact, there was no significant
difference in complication rates between HMH and LMHs. However, we found that failure to
rescue rates (or case fatality rates—deaths that occur after complications) explain much of
the differences in mortality rates between HMHs and LMHs.
Read the rest of the interviews on MedicalResearch.com
Content NOT an endorsement of efficacy and NOT intended as specific medical advice.
Hospital Mortality Outcomes Differ After Lung Cancer Surgery Complications
MedicalResearch.com Interview with: Tyler Grenda, MD House Officer VI Section of General Surgery Department of Surgery
University of Michigan
• Medical Research: What recommendations do you have for future research as a result of
this study?
• Dr. Grenda: Further investigation is needed to better understand the main factors related to
the development of complications with a focus on their subsequent management. We need
to improve our understanding of why there are differences in rates of failure to rescue.
• Citation:
• Grenda TR, Revels SL, Yin H, Birkmeyer JD, Wong SL. Lung Cancer Resection at Hospitals With
High vs Low Mortality Rates JAMA Surg. Published online August 12, 2015.
doi:10.1001/jamasurg.2015.2199.
Read the rest of the interviews on MedicalResearch.com
Content NOT an endorsement of efficacy and NOT intended as specific medical advice.
Increased Indoor Tanning Among Gay/Bisexual Men Raises Skin Cancer Risk
MedicalResearch.com Interview with: Howa Yeung, MD PGY3, Emory Dermatology
Emory University
• Medical Research: What is the background for this study? What are the main findings?
• Dr. Yeung: Indoor tanning is a well-established and preventable cause for melanomas and
non-melanoma skin cancers. Public health efforts in curbing indoor tanning have focused on
known high-risk populations, such as young, college-aged, White women. However, other
demographic risk factors for indoor tanning remain unknown.
• As our nation increasingly focuses on addressing and improving the health of lesbian, gay,
bisexual, and transgender (LGBT) individuals, more and more evidence demonstrates that
various LGBT subpopulations face higher rates of cancer-related behavior risk factors, such as
smoking, alcohol use, obesity, etc. We wanted to find out whether risk factors for skin cancer,
such as indoor tanning, disproportionately affected LGBT populations.
• Our study showed higher rates of indoor tanning among gay and bisexual men, with 1.8-fold
and 3.6-fold higher odds of tanning bed use within the past year, compared to straight men,
after adjusting for sociodemographic factors. Disparities in frequent tanning, defined as using
tanning bed 10 or more times within the past year, are even more prominent among gay and
bisexual men. In contrast, no significant sexual orientation disparities were noted among
women after adjusting for sociodemographic factors.
Read the rest of the interviews on MedicalResearch.com
Content NOT an endorsement of efficacy and NOT intended as specific medical advice.
Increased Indoor Tanning Among Gay/Bisexual Men Raises Skin Cancer Risk
MedicalResearch.com Interview with: Howa Yeung, MD PGY3, Emory Dermatology
Emory University
• Medical Research: What should clinicians and patients take away from your report?
• Dr. Yeung: Indoor tanning among gay and bisexual men is significant and underappreciated
problem, with rates approaching those among other known high-risk demographic groups.
These results call for critically needed clinical a
Read the rest of the interviews on MedicalResearch.com
Content NOT an endorsement of efficacy and NOT intended as specific medical advice.
Increased Indoor Tanning Among Gay/Bisexual Men Raises Skin Cancer Risk
MedicalResearch.com Interview with: Howa Yeung, MD PGY3, Emory Dermatology
Emory University
• Medical Research: What recommendations do you have for future research as a result of
this study?
• Dr. Yeung: We hope our study will encourage further research on skin health among LGBT
individuals. Future studies should delineate the burden of skin cancer and its modifiable risk
factors among LGBT individuals. In addition, specific mediators for indoor tanning among gay
and bisexual men should be examined, in order to develop targeted interventions that
address the root causes of disparities.
• Citation:
• Yeung H, Chen SC. Sexual Orientation and Indoor Tanning Device Use: A Population-Based
Study. JAMA Dermatol. Published online August 05, 2015.
doi:10.1001/jamadermatol.2015.2038.
Read the rest of the interviews on MedicalResearch.com
Content NOT an endorsement of efficacy and NOT intended as specific medical advice.
Several Modifiable Risk Factors for Alzheimer’s Disease Identified
MedicalResearch.com Interview with: Jin-Tai Yu MD, PhD
Memory and Aging Center, Department of Neurology University of California San Francisco
San Francisco, CA 94158
• Medical Research: What is the background for this study? What are the main findings?
• Response: The number of dementia cases in the whole world was estimated to be 35.6
million in 2010 and this number was expected to almost double every 20 years, to 65.7
million in 2030 and 115.4 million in 2050. The global prevalence of dementia was 5-7% and
Alzheimer’s disease accounts for roughly 60%. This data means that we are facing an
increasing number of global populations of this specific neurodegenerative disease and also
the heavy burden brought by it.
• Data from the website of global clinical trials (http://clinicalTrials.gov) showed that a total of
1,732 clinical trials for Alzheimer’s disease were under way. However, the previous results are
not so optimistic, possibly due to the complex etiological mechanisms. In one word, we had
currently no effective drugs for this disease. Figuring out how to effectively prevent its
occurrence is increasingly attracting people’s attentions.Therefore, we have done the most
extensive and comprehensive systematic review and meta-analysis to date, which employs a
full-scale search of observational studies to calculate effect sizes and grade the evidence
strength of various modifiable risk factors for this disease. We hope these results will be
informative and instructive.
Read the rest of the interviews on MedicalResearch.com
Content NOT an endorsement of efficacy and NOT intended as specific medical advice.
Several Modifiable Risk Factors for Alzheimer’s Disease Identified
MedicalResearch.com Interview with: Jin-Tai Yu MD, PhD
Memory and Aging Center, Department of Neurology University of California San Francisco
San Francisco, CA 94158
We systematically searched PubMed and the Cochrane Database of Systematic Reviews from
inception to July 2014, and the references of retrieved relevant articles. We included prospective
cohort studies and retrospective case–control studies. 16,.906 articles were identified of which
323 with 93 factors met the inclusion criteria for meta-analysis. In addition, we assigned three
grades of evidence in support of the conclusion according to two elements including the pooled
sample size and heterogeneity: ‘grade I evidence’ was defined as both pooled population >5000
and lower heterogeneity (I2<50%); ‘grade II-A evidence’ was defined as pooled population >5000
but with higher heterogeneity (I2≥50%); ‘grade II-B evidence’ was defined as lower heterogeneity
(I2≥50%) but with pooled population <5000; ‘grade III evidence’ was defined as both pooled
population <5000 and higher heterogeneity.
Read the rest of the interviews on MedicalResearch.com
Content NOT an endorsement of efficacy and NOT intended as specific medical advice.
Several Modifiable Risk Factors for Alzheimer’s Disease Identified
MedicalResearch.com Interview with: Jin-Tai Yu MD, PhD
Memory and Aging Center, Department of Neurology University of California San Francisco
San Francisco, CA 94158
Eleven risk factors with grade I evidence included heavy smoking, low DBP, high BMI in midlife,
carotid atherosclerosis, DM-2 in Asian population, low BMI, low educational attainment, high
tHcy level, depression, SBP > 160mmHg and frailty; and 2 risk factors with grade II-A evidence
included current smoking in Asian population and neuroticism. Our study also found that 23
protective factors for Alzheimer’s Disease (Grade I evidence: coffee/caffeine drinking, high folate
intake, cognitive activity, high vitamin E intake, high vitamin C intake, current statin use, arthritis,
light-to-moderate drinking, ever use of estrogens, anti-hypertensive medications, NASIDs use,
high BMI in late-life, et al.; Grad II-A evidence: healthy dietary pattern, fish consumption, high
education and physical activity). Given that most of the studies included in the meta-analysis are
case-control studies, what is seen here is an association rather than a direct cause and effect
relationship between any one factor and Alzheimer’s risk.
Read the rest of the interviews on MedicalResearch.com
Content NOT an endorsement of efficacy and NOT intended as specific medical advice.
Several Modifiable Risk Factors for Alzheimer’s Disease Identified
MedicalResearch.com Interview with: Jin-Tai Yu MD, PhD
Memory and Aging Center, Department of Neurology University of California San Francisco
San Francisco, CA 94158
• Medical Research: What should clinicians and patients take away from your report?
• Response: Our study found that individuals would benefit from addressing the related
potentially modifiable factors. Effective interventions in diet, medications, biochemical
exposures, psychological condition, preexisting disease and lifestyle might decrease new
incidence of Alzheimer’s Disease.
Read the rest of the interviews on MedicalResearch.com
Content NOT an endorsement of efficacy and NOT intended as specific medical advice.
Several Modifiable Risk Factors for Alzheimer’s Disease Identified
MedicalResearch.com Interview with: Jin-Tai Yu MD, PhD
Memory and Aging Center, Department of Neurology University of California San Francisco
San Francisco, CA 94158
• Medical Research: What recommendations do you have for future research as a result of
this study?
• Response: Given that, as yet, there is no cure forAlzheimer’s Disease. Our study indicates that
Alzheimer’s Disease might be preventable by effectively addressing the identified modifiable
factors. More prospective cohort studies are necessary for clearing the exact direct
relationship between the related modifiable risk factors and Alzheimer’s risk and exploring
the possibility of preventing Alzheimer’s Disease by effectively addressing the identified
modifiable factors.
• Citation:
• J Neurol Neurosurg Psychiatry. 2015 Aug 20. pii: jnnp-2015-310548. doi: 10.1136/jnnp-2015-
310548. [Epub ahead of print]
• Meta-analysis of modifiable risk factors for Alzheimer’s disease.
• Xu W1, Tan L2, Wang HF3, Jiang T3, Tan MS1, Tan L4, Zhao QF1, Li JQ1, Wang J1, Yu JT5.
Read the rest of the interviews on MedicalResearch.com
Content NOT an endorsement of efficacy and NOT intended as specific medical advice.
Research Into Many Skin Diseases Receives Too Little NIH Funding
MedicalResearch.com Interview with: Erika Hagstrom, M.D., M.A. Preliminary Internal Medicine PGY-1
Loyola University Medical Center
• Medical Research: What is the background for this study? What are the main findings?
• Dr. Hagstrom: Allocation of funding dollars to research is a critical and daunting task. While
many factors may impact research-funding decisions, establishing a transparent priority-
setting exercise is paramount. This is particularly important for the National Institutes of
Health, which invests over $30 billion for medical research each year. Diseases that have the
greatest impact on our population warrant increased research dollars to reduce disease
burden. The Global Burden of Disease Study (GBD) is an epidemiological effort to quantify
the global burden of disease in a universal metric called disability-adjusted life years (DALYs).
Focusing on our particular interest of dermatology, we investigated the 2012-2013 NIH
funding for 15 skin diseases and matched this to the corresponding DALY metrics.
Read the rest of the interviews on MedicalResearch.com
Content NOT an endorsement of efficacy and NOT intended as specific medical advice.
Research Into Many Skin Diseases Receives Too Little NIH Funding
MedicalResearch.com Interview with: Erika Hagstrom, M.D., M.A. Preliminary Internal Medicine PGY-1
Loyola University Medical Center
• Medical Research: What is the background for this study? What are the main findings?
• Dr. Hagstrom: Allocation of funding dollars to research is a critical and daunting task. While
many factors may impact research-funding decisions, establishing a transparent priority-
setting exercise is paramount. This is particularly important for the National Institutes of
Health, which invests over $30 billion for medical research each year. Diseases that have the
greatest impact on our population warrant increased research dollars to reduce disease
burden. The Global Burden of Disease Study (GBD) is an epidemiological effort to quantify
the global burden of disease in a universal metric called disability-adjusted life years (DALYs).
Focusing on our particular interest of dermatology, we investigated the 2012-2013 NIH
funding for 15 skin diseases and matched this to the corresponding DALY metrics.
Read the rest of the interviews on MedicalResearch.com
Content NOT an endorsement of efficacy and NOT intended as specific medical advice.
Research Into Many Skin Diseases Receives Too Little NIH Funding
MedicalResearch.com Interview with: Erika Hagstrom, M.D., M.A. Preliminary Internal Medicine PGY-1
Loyola University Medical Center
• Medical Research: What recommendations do you have for future research as a result of
this study?
• Dr. Hagstrom: We hope that our investigation demonstrates the usefulness of high-quality
disease burden metrics to inform priority-setting decisions for any funding center.
• Citation:
• J Am Acad Dermatol. 2015 Sep;73(3):383-391.e1. doi: 10.1016/j.jaad.2015.04.039. Epub
2015 Jun 4.
• Comparing cutaneous research funded by the US National Institutes of Health (NIH) with the
US skin disease burden.
• Hagstrom EL1, Patel S2, Karimkhani C3, Boyers LN4, Williams HC5, Hay RJ6, Weinstock
MA7, Armstrong AW8, Dunnick CA9, Margolis DJ10, Dellavalle RP11.
Read the rest of the interviews on MedicalResearch.com
Content NOT an endorsement of efficacy and NOT intended as specific medical advice.
Spinach Bleach Contours May Prevent Adequate Decontamination
MedicalResearch.com Interview with: Nichola Kinsinger, Ph.D Postdoctoral Researcher, Chemical & Environmental Engineering
University of California, Riverside
USDA National Institute for Food and Agricultural Postdoctoral Fellow
DoD Office of Naval
• Medical Research: What is the background for this study? What are the main findings?
• Dr. Kinsinger: Outbreaks observed in produce are becoming increasing common possibly due
to contaminated irrigation waters or contaminated waters used during processing. In 2006
California had spinach-borne E. coli outbreak that impacted 26 states and 200 confirmed
sickened. Leafy greens account for 20% of the outbreaks alone and are of increased concern
since they are frequently consumed raw. These outbreaks drew our attention over the past
few years and we started applying methods originally developed for studying bacterial
adhesion on engineered surfaces to the issue of food safety. Although food safety is a new
area of study for our lab, the project is based upon the concept of pathogen adhesion
transport which has been the focus of my advisor’s lab for many years previously. Rather
than the previous scenario looking at pathogen interaction with engineered or mineral
surfaces, we are looking at a spinach leaf instead.
Read the rest of the interviews on MedicalResearch.com
Content NOT an endorsement of efficacy and NOT intended as specific medical advice.
Spinach Bleach Contours May Prevent Adequate Decontamination
MedicalResearch.com Interview with: Nichola Kinsinger, Ph.D Postdoctoral Researcher, Chemical & Environmental Engineering
University of California, Riverside
USDA National Institute for Food and Agricultural Postdoctoral Fellow
DoD Office of Naval
We are using a parallel plate flow chamber system developed by Professor Sharon Walker to
evaluate the real time attachment and detachment of the pathogens to the spinach epicuticle
layer (surface layer of the leaf) in realistic water chemistries and flow conditions. Subsequently
we can evaluate the efficacy of the rinsing process to kill the bacteria that may remain on the
leaf. Initially we were finding that at low concentrations of bleach, bacteria will detach from the
leaf surface allowing for potential cross-contamination later in the process. However above
500ppb we observed 100% of the attached bacterial cells are killed.
Read the rest of the interviews on MedicalResearch.com
Content NOT an endorsement of efficacy and NOT intended as specific medical advice.
Spinach Bleach Contours May Prevent Adequate Decontamination
MedicalResearch.com Interview with: Nichola Kinsinger, Ph.D Postdoctoral Researcher, Chemical & Environmental Engineering
University of California, Riverside
USDA National Institute for Food and Agricultural Postdoctoral Fellow
DoD Office of Naval
So how in commercial rinsing operations that use bleach concentration ranging from 50-200ppm
result in outbreaks? Through this study we analyzed the topography of the leaf and modeled the
concentration gradient across the surface of a leaf based on commercial rinsing practices. We
found that even at high bleach concentrations within rinse water result in low bleach
concentrations at the leaf surface on the order of 5-1000 ppb bleach, which in our study has
shown to that the bacteria can survive when attached to the leaf and/or detach causing concern
for cross-contamination. In this case the very disinfection processes intended to clean, remove,
and prevent contamination was found to be a potential pathway to amplifying foodborne
outbreaks.
Read the rest of the interviews on MedicalResearch.com
Content NOT an endorsement of efficacy and NOT intended as specific medical advice.
Spinach Bleach Contours May Prevent Adequate Decontamination
MedicalResearch.com Interview with: Nichola Kinsinger, Ph.D Postdoctoral Researcher, Chemical & Environmental Engineering
University of California, Riverside
USDA National Institute for Food and Agricultural Postdoctoral Fellow
DoD Office of Naval
• Medical Research: What recommendations do you have for future research as a result of
this study?
• Dr. Kinsinger: The work presented at the conference is part of an ongoing effort in my
advisor’s lab. The work will continue to look at a broader range of foods, engineered
surfaces, and pathogen types.
• Citation:
• 250th American Chemical Society National Meeting & Exposition discussing:
• “Is our salad safe? Efficacy of disinfection techniques to decontaminate spinach leaves and
reduce cross-contamination”
Read the rest of the interviews on MedicalResearch.com
Content NOT an endorsement of efficacy and NOT intended as specific medical advice.
Early Life Intelligence Linked To Better Physical Fitness in Middle Age
MedicalResearch.com Interview with: Rikke Hodal Meincke PhD student
Center for Healthy Aging and the Department of Public Health
University of Copenhagen
• Medical Research: What is the background for this study?
• Response: A sufficient level of physical capability is a precondition for maintaining
independence and quality of life. Physical capability can be assessed objectively by tests of
physical performance, for instance handgrip strength, chair-rising and postural balance.
Physical performance is associated with mortality and disability in late life, so gaining insights
into the variance in physical performance is important to promote sustained physical
capability and prevent disability. Research has previously found physical activity, health status
and socioeconomic position to be associated with physical performance. In addition, early life
factors, such as childhood SEP, have been found to be associated with measures of physical
performance later in life. The objective of our study was to examine the association between
intelligence in early adulthood and midlife physical performance in Danish men.
Read the rest of the interviews on MedicalResearch.com
Content NOT an endorsement of efficacy and NOT intended as specific medical advice.
Early Life Intelligence Linked To Better Physical Fitness in Middle Age
MedicalResearch.com Interview with: Rikke Hodal Meincke PhD student
Center for Healthy Aging and the Department of Public Health
University of Copenhagen
If an association between intelligence in early life and midlife physical performance exists it may
indicate that cognitive abilities and physical performance share some of the same
neurodevelopmental processes, but may also indicate that intelligence has an independent effect
on later physical performance through various pathways.
Read the rest of the interviews on MedicalResearch.com
Content NOT an endorsement of efficacy and NOT intended as specific medical advice.
Early Life Intelligence Linked To Better Physical Fitness in Middle Age
MedicalResearch.com Interview with: Rikke Hodal Meincke PhD student
Center for Healthy Aging and the Department of Public Health
University of Copenhagen
• Medical Research: What are the main findings?
• Response: In our study of more than 2800 Danish men, we found positive associations
between intelligence in early adulthood and five objective measures of physical performance
in midlife independent of other early life factors. A one standard deviation increase in
intelligence score resulted in 1.1 more chair-rises in 30 seconds, a 1 cm higher jump, a 3.7%
smaller balance area, a 0.7 kg increase in handgrip-strength, and a 0.5 kg increase in lower
back force.
Read the rest of the interviews on MedicalResearch.com
Content NOT an endorsement of efficacy and NOT intended as specific medical advice.
Early Life Intelligence Linked To Better Physical Fitness in Middle Age
MedicalResearch.com Interview with: Rikke Hodal Meincke PhD student
Center for Healthy Aging and the Department of Public Health
University of Copenhagen
• Medical Research: What should clinicians and patients take away from your report?
• Response: This knowledge can be used in the development of interventions which should be
designed to address people with different cognitive abilities. Furthermore, policy-makers and
scientists should bear in mind that the prevention of mobility limitations in midlife might
need to start early in life.
Read the rest of the interviews on MedicalResearch.com
Content NOT an endorsement of efficacy and NOT intended as specific medical advice.
Early Life Intelligence Linked To Better Physical Fitness in Middle Age
MedicalResearch.com Interview with: Rikke Hodal Meincke PhD student
Center for Healthy Aging and the Department of Public Health
University of Copenhagen
• Medical Research: What recommendations do you have for future research as a result of
this study?
• Response: Future research should look into the mechanisms behind the association between
intelligence and physical performance, for instance if physical activity or other lifestyle factors
mediate the relationship. When the mechanisms are better understood, interventions aiming
to prevent mobility limitations should incorporate this knowledge.
• Citation:
• H. Meincke, M. Osler, E. L. Mortensen, A. M. Hansen. Is Intelligence in Early Adulthood
Associated With Midlife Physical Performance Among Danish Males?Journal of Aging and
Health, 2015; DOI:10.1177/0898264315594139
Read the rest of the interviews on MedicalResearch.com
Content NOT an endorsement of efficacy and NOT intended as specific medical advice.
Extent of Liver Damage May Be Underestimated In Hepatitis C Patients
MedicalResearch.com Interview with: Stuart Gordon, M.D.
Director of Hepatology at Henry Ford Hospital
Detroit, Michigan
• Medical Research: What is the background for this study? What are the main findings?
• Dr. Gordon: The U.S. Centers for Disease Control and Prevention’s Division of Viral Hepatitis
estimates 2.7 to 3.9 million people in the United States currently suffer from chronic hepatitis
C. But, unfortunately, many of these patients may be unaware of the severity of their liver
damage. We looked at evidence of cirrhosis among hepatitis C patients by examining four
different parameters: ICD9 codes; liver biopsy reports; evidence of liver failure; and the FIB-4
test, an easily calculated biomarker. By using all four indicators of cirrhosis, we found a far
higher prevalence of cirrhosis than would be indicated by any one method.
Read the rest of the interviews on MedicalResearch.com
Content NOT an endorsement of efficacy and NOT intended as specific medical advice.
Extent of Liver Damage May Be Underestimated In Hepatitis C Patients
MedicalResearch.com Interview with: Stuart Gordon, M.D.
Director of Hepatology at Henry Ford Hospital
Detroit, Michigan
• Medical Research: What should clinicians and patients take away from your report?
• Dr. Gordon: Presence of cirrhosis in hepatitis C may be hidden or not easily recognizable; is
often asymptomatic in its early form; and may require a high degree of suspicion in order to
diagnose.
Read the rest of the interviews on MedicalResearch.com
Content NOT an endorsement of efficacy and NOT intended as specific medical advice.
Extent of Liver Damage May Be Underestimated In Hepatitis C Patients
MedicalResearch.com Interview with: Stuart Gordon, M.D.
Director of Hepatology at Henry Ford Hospital
Detroit, Michigan
• Medical Research: What recommendations do you have for future research as a result of
this study?
• Dr. Gordon: Studies that rely on diagnostic codes or biopsies performed in routine care for
evidence of cirrhosis may be underestimating the extent of cirrhosis, which is a leading cause
of death in the United States. Future research should investigate the value of indirect markers
of cirrhosis. This could result in earlier diagnosis and prevention of liver disease
complications.
• Citation:
• Stuart C Gordon, Lois E Lamerato, Loralee B Rupp, Scott D Holmberg, Anne C Moorman,
Philip R Spradling, Eyasu Teshale, Fujie Xu, Joseph A Boscarino, Vinutha Vijayadeva, Mark A
Schmidt, Nancy Oja-Tebbe, Mei Lu. Prevalence of Cirrhosis in Hepatitis C Patients in the
Chronic Hepatitis Cohort Study (CHeCS): A Retrospective and Prospective Observational
Study. The American Journal of Gastroenterology, 2015; 110 (8): 1169
DOI: 10.1038/ajg.2015.203
Read the rest of the interviews on MedicalResearch.com
Content NOT an endorsement of efficacy and NOT intended as specific medical advice.
Protein Irisin Linked to Muscle Wasting and Atherosclerosis in Dialysis Patients
MedicalResearch.com Interview with: Tae-Hyun Yoo MD PhD
Department of Internal Medicine, College of Medicine
Severance Biomedical Science Institute, Brain Korea 21 PLUS
Yonsei University, Seoul, Korea
• Medical Research: What is the background for this study? What are the main findings?
• Dr. Tae-Hyun Yoo: Sarcopenia, reduction in muscle mass, is frequently observed in PEW and
is prevalent in chronic kidney disease (CKD) patients. In ESRD patients, sarcopenia is
significantly associated with greater mortality. Skeletal muscles produce and release
myokines, which suggested to mediate their protective effects. Irisin, a novel myokine, has
been introduced to drive brown-fat-like conversion of white adipose tissue and has beneficial
effects of skeletal muscle on energy homeostasis and glucose metabolisms. Therefore, we
hypothesized that irisin had significant association with sarcopenia and cardiovascular
disease in dialysis patients. In peritoneal dialysis patients, serum irisin was positively
correlated with mid-arm muscle circumference and thigh circumference. In addition, serum
irisin was a significant independent predictor for carotid atherosclerosis even after
adjustment for high-sensitivity C-reactive protein in these patients. This study demonstrated
that serum irisin was significantly associated with sarcopenia and carotid atherosclerosis in
peritoneal dialysis patients.
Read the rest of the interviews on MedicalResearch.com
Content NOT an endorsement of efficacy and NOT intended as specific medical advice.
Protein Irisin Linked to Muscle Wasting and Atherosclerosis in Dialysis Patients
MedicalResearch.com Interview with: Tae-Hyun Yoo MD PhD
Department of Internal Medicine, College of Medicine
Severance Biomedical Science Institute, Brain Korea 21 PLUS
Yonsei University, Seoul, Korea
• Medical Research: What should clinicians and patients take away from your report?
• Dr. Tae-Hyun Yoo: Malnutrition and chronic inflammation have been proposed to explain the
significant association between sarcopenia and cardiovascular disease in (end stage renal
disease) ESRD population. Lower muscle mass might reflect poor nutritional status and
higher degrees of unopposed inflammation. In addition to these factors, a myokine could be
a contributor. In this study, irisin as a myokine could be helpful for stratifying risk of
sarcopenia and carotid atherosclerosis in peritoneal dialysis patients. In addition, present
study suggested muscle is an another important organ for protection and modulating
cardiovascular diseases in dialysis population.
Read the rest of the interviews on MedicalResearch.com
Content NOT an endorsement of efficacy and NOT intended as specific medical advice.
Protein Irisin Linked to Muscle Wasting and Atherosclerosis in Dialysis Patients
MedicalResearch.com Interview with: Tae-Hyun Yoo MD PhD
Department of Internal Medicine, College of Medicine
Severance Biomedical Science Institute, Brain Korea 21 PLUS
Yonsei University, Seoul, Korea
• Medical Research: What recommendations do you have for future research as a result of
this study?
• Dr. Tae-Hyun Yoo: A study with follow-up carotid ultrasound and measurement of
anthropometric indices and serum irisin is necessary to clarify the association of irisin,
sarcopenia, and cardiovascular disease. Therefore, the impact of irisin on future
cardiovascular outcomes should be clarified in the future study. The effect of the intervention
using exercise training on irisin, sarcopenia, and cardiovascular disease might be also worth
investigating in dialysis patients.
• Citation:
• Irisin, a novel myokine is an independent predictor for sarcopenia and carotid atherosclerosis
in dialysis patients
• Lee, Mi Jung et al. Atherosclerosis
• Received: February 20, 2015; Received in revised form: July 7, 2015; Accepted: August 6,
2015; Published Online: August 13, 2015
• Publication stage: In Press Accepted Manuscript
• DOI: http://dx.doi.org/10.1016/j.atherosclerosis.2015.08.002
Read the rest of the interviews on MedicalResearch.com
Content NOT an endorsement of efficacy and NOT intended as specific medical advice.
Factors Identified For Lasting Total Hip Replacement
MedicalResearch.com Interview with: Sandrine Colas, MSc, MPH Department of Epidemiology of Health Products
French National Agency for Medicines and Health Products Safety (ANSM)
Saint-Denis, France
• Medical Research: What is the background for this study? What are the main findings?
• Response: Total hip replacement (THR) is to replace a damaged coxofemoral joint with a
prosthetic implant. Primary or secondary degenerative osteoarthritis of the hip joint is the
main indication for THR (other indications are essentially trauma, which is more common in
the elderly and mostly affects women over 80 years of age). The number of THR has
increased in all industrialized countries, particularly on account of the ageng population.Total
hip replacement is one of the most common and successful surgical procedures in modern
practice. Although results are generally good, revision (consisting in changing one or all
components of the implant) is sometimes necessary (about 1% per year). Prosthetic revision
is a longer and more complex operation than primary implantation and it has a higher
incidence of post-surgical complications.
Read the rest of the interviews on MedicalResearch.com
Content NOT an endorsement of efficacy and NOT intended as specific medical advice.
Factors Identified For Lasting Total Hip Replacement
MedicalResearch.com Interview with: Sandrine Colas, MSc, MPH Department of Epidemiology of Health Products
French National Agency for Medicines and Health Products Safety (ANSM)
Saint-Denis, France
• Several prosthetic revision risk factors have been highlighted recently in published studies,
but results relating to prosthetic and/or patient characteristics and total hip replacement
survivorship tend to vary. The existence of an association between the fixation technique
and/or bearing surface and prosthetic survivorship has yet to be established.
• The main aim of our work was therefore to compare total hip replacement short-term
survivorship according to cement type and bearing surface, in a large population of subjects
who have undergone total hip replacement for reasons other than trauma (25%) and bone
tumor (<0.1%), taking prosthetic revision risk factors (age, gender, comorbidities,
concomitant medication, implanting center, etc.) into account.
• Total hip replacement characteristics are related to early implant survivorship. After 33
months of follow-up, antibiotic-impregnated cemented THRs have a better prognosis. MoM
total hip replacemenst have a slightly worse prognosis.
Read the rest of the interviews on MedicalResearch.com
Content NOT an endorsement of efficacy and NOT intended as specific medical advice.
Factors Identified For Lasting Total Hip Replacement
MedicalResearch.com Interview with: Sandrine Colas, MSc, MPH Department of Epidemiology of Health Products
French National Agency for Medicines and Health Products Safety (ANSM)
Saint-Denis, France
• Medical Research: What should clinicians and patients take away from your report?
• Response: These findings are useful in helping surgeons select a total hip replacement
fixation technique and bearing surface, and helpful for both patient and surgeon in the
decision-making process.
• Citation:
• Colas S, Collin C, Piriou P, Zureik M. Association Between Total Hip Replacement
Characteristics and 3-Year Prosthetic Survivorship : A Population-Based Study.JAMA Surg.
Published online August 19, 2015. doi:10.1001/jamasurg.2015.1325.
Read the rest of the interviews on MedicalResearch.com
Content NOT an endorsement of efficacy and NOT intended as specific medical advice.
Noninvasive FFR Study Superior to CT Angiography In Patients With Highly Calcified Coronary Disease
MedicalResearch.com Interview with: Bjarne Linde Norgaard, MD PhD Department of Cardiology, Aarhus University Hospital
Skejby
Aarhus, Denmark
• Medical Research: What is the background for this study? What are the main findings?
• Dr. Norgaard: Noninvasive fractional flow reserve derived from standard acquired coronary
CT angiography (CTA) (FFRct) in patients with suspected coronary artery disease exhibits high
and superior diagnostic performance when compared to coronary CTA alone in identifying
lesion-specific ischemia (which is the established metric for decision-making on coronary
revascularization). As the presence of coronary calcification may compromise the
diagnostic accuracy and specificity of coronary CTA, this study was performed in order to
investigate the influence of calcification on the diagnostic performance of FFRct in patients
(214) and vessels (333) with suspected coronary artery disease. The main finding in this study
was that FFRct provides high and superior diagnostic performance and discrimination of
ischemia compared with coronary CTA interpretation alone in patients and vessels with high
levels of calcification.
Read the rest of the interviews on MedicalResearch.com
Content NOT an endorsement of efficacy and NOT intended as specific medical advice.
Noninvasive FFR Study Superior to CT Angiography In Patients With Highly Calcified Coronary Disease
MedicalResearch.com Interview with: Bjarne Linde Norgaard, MD PhD Department of Cardiology, Aarhus University Hospital
Skejby
Aarhus, Denmark
• Medical Research: What should clinicians and patients take away from your report?
• Dr. Norgaard: In the event of good CT image quality, which can be established in most
patients by careful attention to pre-acquisition heart-rate control and use of nitroglycerin,
noninvasive FFRct provides high diagnostic performance in identifying lesion-specific
ischemia in patients and vessels over a wide range of coronary calcification.
Read the rest of the interviews on MedicalResearch.com
Content NOT an endorsement of efficacy and NOT intended as specific medical advice.
Novel DNA Vaccine Provides Protection Against MERS Virus
MedicalResearch.com Interview with: David B. Weiner, Ph.D.
Professor, Department of Pathology and Laboratory Medicine
Chair, Gene Therapy and Vaccine Program, CAMB
Co-Leader Tumor Virology Program, Abramson Cancer Program
University of Pennsylvania, Perelman School of Medicine
• Medical Research: What is the background for this study? What are the main findings?
• Dr. Weiner: MERS, like the Severe Acute Respiratory Syndrome (SARS), is characterized by
high fever and severe cough from pneumonia. MERS is caused by an emerging human
coronavirus, which is distinct from the SARS coronavirus. Since its identification in 2012,
MERS has been linked to over 1,300 infections and close to 400 deaths. It has occurred in the
Arabian Peninsula, Europe, and in the US and in Asia. It can be spread in a hospital setting.
• Scientists now report that a novel synthetic DNA vaccine can, for the first time, induce
protective immunity against the Middle EastRespiratory Syndrome (MERS) coronavirus in
animal species. Researchers from the Perelman School of Medicine at the University of
Pennsylvania. The NIH, the Public Health agency of Canada, and from a leading company in
the development of synthetic DNA vaccine technology, Inovio described the results in a
paper published their work in Science Translational Medicine (STM) this
week. The experimental, preventive vaccine, given six weeks before exposure to the MERS
virus, fully protects rhesus macaques from disease. The vaccine also generated potentially
protective antibodies in blood drawn from camels, the purported source of MERS
transmission in the Middle East.
Read the rest of the interviews on MedicalResearch.com
Content NOT an endorsement of efficacy and NOT intended as specific medical advice.
Novel DNA Vaccine Provides Protection Against MERS Virus
MedicalResearch.com Interview with: David B. Weiner, Ph.D.
Professor, Department of Pathology and Laboratory Medicine
Chair, Gene Therapy and Vaccine Program, CAMB
Co-Leader Tumor Virology Program, Abramson Cancer Program
University of Pennsylvania, Perelman School of Medicine
• Medical Research: What should clinicians and patients take away from your report?
• Response: Dr David Weiner, of the Perelman School of Medicine, senior author of the study
described “The significant recent increase in MERS cases, coupled with the lack of effective
antiviral therapies or vaccines to treat or prevent this infection, have raised significant
concern,” Weiner said. “Accordingly the development of a vaccine for MERS remains a high
priority. and the results reported here in this regard are encouraging.”
• The STM study describes a vaccine to prevent disease and control virus replication in an
important primate model of MERS pathogenesis, which was developed by the National
Institutes of Health (NIH). The vaccine was able to prevent MERS disease and offered benefit
to 100 percent of the animals in this study. In addition, the vaccine induced antibodies that
are linked with protection in camels, a species that is thought to be a major source of
transmission to humans from the Middle East, showing that this vaccine could be deployed to
break this this link in the MERS transmission cycle. In the field, say the researchers, this
vaccine could decrease person-to-person spread of infection in the event of an outbreak and
help to protect health care workers or exposed individuals.
• Dr. Muthumani first author of the study described “This simple synthetic vaccine has the
potential to overcome important production and deployment limitations, and what’s more,
the vaccine is non-live, so does not pose a risk of spreading to unintended individuals.”
Read the rest of the interviews on MedicalResearch.com
Content NOT an endorsement of efficacy and NOT intended as specific medical advice.
Novel DNA Vaccine Provides Protection Against MERS Virus
MedicalResearch.com Interview with: David B. Weiner, Ph.D.
Professor, Department of Pathology and Laboratory Medicine
Chair, Gene Therapy and Vaccine Program, CAMB
Co-Leader Tumor Virology Program, Abramson Cancer Program
University of Pennsylvania, Perelman School of Medicine
• Medical Research: What recommendations do you have for future research as a result of
this study?
• Dr. Weiner: Most importantly the corportate collaborator for this study, Inovio, will move this
program into human testing. Under a previously announced collaboration agreement with
GeneOne Life Science Inc. (KSE: 011000), Inovio’s MERS vaccine product will advance into a
clinical trial in healthy volunteers before the year end. In addition studies of the use of such a
vaccine in a formulation specific for Camels, the animal species associated with transmission
to humans, is continuing.
• Citation:
• Muthumani, D. Falzarano, E. L. Reuschel, C. Tingey, S. Flingai, D. O. Villarreal, M. Wise, A.
Patel, A. Izmirly, A. Aljuaid, A. M. Seliga, G. Soule, M. Morrow, K. A. Kraynyak, A. S. Khan, D. P.
Scott, F. Feldmann, R. LaCasse, K. Meade-White, A. Okumura, K. E. Ugen, N. Y. Sardesai, J. J.
Kim, G. Kobinger, H. Feldmann, D. B. Weiner. A synthetic consensus anti-spike protein DNA
vaccine induces protective immunity against Middle East respiratory syndrome coronavirus in
nonhuman primates. Science Translational Medicine, 2015; 7 (301): 301ra132
DOI:10.1126/scitranslmed.aac7462
Read the rest of the interviews on MedicalResearch.com
Content NOT an endorsement of efficacy and NOT intended as specific medical advice.
Single Vaccine Can Protect Against Six Common Childhood Diseases
MedicalResearch.com Interview with: Gary S. Marshall, M.D. Professor of Pediatrics
Chief, Division of Pediatric Infectious Diseases
Director, Pediatric Clinical Trials Unit
University of Louisville School of Medicine
• Medical Research: What is the background for this study? What are the main findings?
• Dr. Marshall: The infant immunization schedule has become crowded. That’s great news, in a
sense, because it means that our children have become better protected against more
diseases. At the same time, this has led to well child visits during which many shots are
recommended, and some parents want to limit the number of injections their children
receive at one time. This leads to deferrals, poor timeliness and decreased coverage rates, all
of which could impair protection. This study shows that a hexavalent vaccine—one that
combines diphtheria, tetanus, pertussis, polio, Haemophilus influenzae type b, and hepatitis
B vaccines in one syringe—is safe and just as immunogenic as the currently used component
vaccines.
Read the rest of the interviews on MedicalResearch.com
Content NOT an endorsement of efficacy and NOT intended as specific medical advice.
Single Vaccine Can Protect Against Six Common Childhood Diseases
MedicalResearch.com Interview with: Gary S. Marshall, M.D. Professor of Pediatrics
Chief, Division of Pediatric Infectious Diseases
Director, Pediatric Clinical Trials Unit
University of Louisville School of Medicine
• Medical Research: What should clinicians and patients take away from your report?
• Dr. Marshall: Pending FDA approval, pediatric practices should have a new option for infant
immunization, one that will allow for fewer shots without compromising protection.
• Medical Research: What recommendations do you have for future research as a result of
this study?
• Dr. Marshall: The more vaccine antigens that can be combined into single injections the
better, as long as safety and protection are preserved. The performance of higher-valency
combination vaccines cannot be predicted a priori, so the formulations have to be prepared
and the clinical studies have to be done to assess reactogenicity, safety and immunogenicity.
• Citation:
• Immunogenicity, Safety, and Tolerability of a Hexavalent Vaccine in Infants
• Gary S. Marshall, Gregory L. Adams, Michael L. Leonardi, Maria Petrecz,Sheryl A.
Flores, Angela L. Ngai, Jin Xu, Guanghan Liu, Jon E. Stek, Ginamarie Foglia, and Andrew W. Lee
• Pediatrics peds.2014-4102; published ahead of print July 27, 2015,doi:10.1542/peds.2014-
4102
Read the rest of the interviews on MedicalResearch.com
Content NOT an endorsement of efficacy and NOT intended as specific medical advice.
How Do Female Cells Inactivate One X Chromosome?
MedicalResearch.com Interview with: Hendrik Marks Ph.D Group leader Epigenetics of Stem Cells
Radboud University, Department of Molecular Biology, RIMLS
Nijmegen, The Netherlands
• Medical Research: What is the background for this study? What are the main findings?
• Dr. Marks: In mammals, sex is determined by two so-called “sex” chromosome: males have a
single X chromosome as well as a Y chromosome, whereas females have two copies of the X
chromosome. However, if both X chromosomes were to be active in female cells, these cells
would have a double dosis of X-chromosomal gene products as compared to male cells. As
this is lethal for almost all cells, female cells shut off one X chromosome in every cell in a
process called X inactivation. This process occurs during early embryonic development.
Read the rest of the interviews on MedicalResearch.com
Content NOT an endorsement of efficacy and NOT intended as specific medical advice.
How Do Female Cells Inactivate One X Chromosome?
MedicalResearch.com Interview with: Hendrik Marks Ph.D Group leader Epigenetics of Stem Cells
Radboud University, Department of Molecular Biology, RIMLS
Nijmegen, The Netherlands
A lot is known about how this process is turned on, but is was unclear how such a silencing
process spreads along a full chromosome. In order to further study this, we used female mouse
embryonic stem cells (mESCs) as a model system and initiated X inactivation by means of
differentiation. With the latest technologies, we were able to keep the two X chromosomes apart
and measure one of them – with its 166 million base pairs (Mbs) – in detail. Every day we
checked which parts of the chromosome had been switched off. The whole process took about
eight days, and the inactivation spreads out from the centre of the X chromosome towards the
ends. That doesn’t happen gradually but moves jumpwise from domain to domain. Domains are
long pieces of DNA (of around 1Mb) that cluster together in knots. As it seems that X inactivation
jumps from domain to domain, we now know that these domains are co-regulated. Also, we
collected strong evidence that the same process is occurring in human.
Read the rest of the interviews on MedicalResearch.com
Content NOT an endorsement of efficacy and NOT intended as specific medical advice.
How Do Female Cells Inactivate One X Chromosome?
MedicalResearch.com Interview with: Hendrik Marks Ph.D Group leader Epigenetics of Stem Cells
Radboud University, Department of Molecular Biology, RIMLS
Nijmegen, The Netherlands
• Medical Research: What should clinicians and patients take away from your report?
• Dr. Marks: Our study is very fundamental in nature. It shows how a full chromosome gets
silenced, and it also points towards a very important role of the recently-discovered domains
it this process. However, It is not unlikely that some diseases that are linked to the X
chromosome in female, the so-called X linked diseases, are due to improper spreading across
domains. Thus, in some of these diseases it might turn out very helpful investigating the
domain structure of the X chromosomes, and/or SNPs that present at the borders of the
domains.
Read the rest of the interviews on MedicalResearch.com
Content NOT an endorsement of efficacy and NOT intended as specific medical advice.
How Do Female Cells Inactivate One X Chromosome?
MedicalResearch.com Interview with: Hendrik Marks Ph.D Group leader Epigenetics of Stem Cells
Radboud University, Department of Molecular Biology, RIMLS
Nijmegen, The Netherlands
• Medical Research: What recommendations do you have for future research as a result of
this study?
• Dr. Marks: After one of the X chromosomes has been inactivated, it will stay inactive forever.
It would be very interesting to discover why sometimes the one while in other cases the
other X chromosome is inactivated during development. That could help in treating X-linked
diseases – like Rett syndrome and fragile X syndrome in females. Reactivating (part of) the
‘right’ X chromosome could be a potential treatment for these diseases. So one of the next
steps that the field is aiming at is to figure out how to do that, and our current studies
provide new clues for potential strategies.
• Citation:
• ‘Dynamics of gene silencing during X inactivation using allele-specific RNA-seq’
• Hendrik Marks, Hindrik H. D. Kerstens, Tahsin Stefan Barakat, Erik Splinter, René A. M. Dirks,
Guido van Mierlo, Onkar Joshi, Shuang-Yin Wang, Tomas Babak, Cornelis A. Albers, Tüzer
Kalkan, Austin Smith, Alice Jouneau, Wouter de Laat, Joost Gribnau and Hendrik G.
Stunnenberg
• Genome Biology; doi:10.1186/s13059-015-0698-x
Read the rest of the interviews on MedicalResearch.com
Content NOT an endorsement of efficacy and NOT intended as specific medical advice.
Florida’s Drug and Pill Mills Law Reduced Opioids Prescriptions
MedicalResearch.com Interview with: Lainie Rutkow, JD, PhD, MPH Associate Professor
Department of Health Policy and Management
Johns Hopkins Bloomberg School of Public Health
• Medical Research: What is the background for this study? What are the main findings?
• Dr. Rutkow: Rates of prescription drug diversion and misuse, as well as overdose deaths,
have increased throughout the United States. CDC estimates that each day, 44 people die
from a prescription drug overdose. In the mid-2000s, Florida was viewed as the epicenter of
this epidemic, with prescription drug overdose deaths increasing more than 80% from 2003
to 2009. In response, Florida enacted several laws to mitigate prescription drug abuse and
diversion. Its pill mill law required pain management clinics to register with the state and
prohibited physician dispensing of certain drugs. Florida’s Prescription Drug Monitoring
Program (PDMP) collects data about dispensing of prescription drugs and can be accessed by
physicians and pharmacists. Little is known about how these laws have affected prescribing
of opioids.
Read the rest of the interviews on MedicalResearch.com
Content NOT an endorsement of efficacy and NOT intended as specific medical advice.
Florida’s Drug and Pill Mills Law Reduced Opioids Prescriptions
MedicalResearch.com Interview with: Lainie Rutkow, JD, PhD, MPH Associate Professor
Department of Health Policy and Management
Johns Hopkins Bloomberg School of Public Health
We applied comparative interrupted time series analyses to pharmacy claims data to examine
four outcomes related to opioid prescribing in Florida, with Georgia as a comparison state. We
found that in the first year of implementation, Florida’s Prescription Drug Monitoring
Program and pill mill law were associated with modest reductions in prescription opioid volume,
prescriptions written, and the dose per prescription. These declines were statistically significant
among the highest volume prescribers and patients at baseline.
Read the rest of the interviews on MedicalResearch.com
Content NOT an endorsement of efficacy and NOT intended as specific medical advice.
Florida’s Drug and Pill Mills Law Reduced Opioids Prescriptions
MedicalResearch.com Interview with: Lainie Rutkow, JD, PhD, MPH Associate Professor
Department of Health Policy and Management
Johns Hopkins Bloomberg School of Public Health
• Medical Research: What should clinicians and patients take away from your report?
• Dr. Rutkow: Our findings suggest that policy responses to the prescription drug epidemic –
such as Prescription Drug Monitoring Program and pill mill laws – are viable options. Most
prescribers are supportive of policies that limit access to opioids. Given this support, along
with our findings, other states may want to consider implementing both a PDMP and a pill
mill law.
Read the rest of the interviews on MedicalResearch.com
Content NOT an endorsement of efficacy and NOT intended as specific medical advice.
Florida’s Drug and Pill Mills Law Reduced Opioids Prescriptions
MedicalResearch.com Interview with: Lainie Rutkow, JD, PhD, MPH Associate Professor
Department of Health Policy and Management
Johns Hopkins Bloomberg School of Public Health
• Medical Research: What recommendations do you have for future research as a result of
this study?
• Dr. Rutkow: While our study looked at the first year of implementation for these laws, future
research should consider their longer-term effects.
• Citation:
• Rutkow L, Chang H, Daubresse M, Webster DW, Stuart EA, Alexander G. Effect of Florida’s
Prescription Drug Monitoring Program and Pill Mill Laws on Opioid Prescribing and Use. JAMA
Intern Med.Published online August 17, 2015. doi:10.1001/jamainternmed.2015.3931.
Read the rest of the interviews on MedicalResearch.com
Content NOT an endorsement of efficacy and NOT intended as specific medical advice.
Polygenetic Score Predicts Educational Attainment
MedicalResearch.com Interview with:
Ben Domingue
Assistant Professor (starting 9/2015)
Stanford Graduate School of Education
• Medical Research: What is the background for this study? What are the main findings?
• Response: Earlier research has started to illuminate which genetic variants are associated
with educational attainment. Subsequent work has taken these variants, combined them into
a “polygenic score”, and studied how that polygenic score predicts educational attainment.
Our research continues this line of inquiry by examining the predictive performance of that
polygenic score in a representative sample of US adults who are now in their 30s. A few
notable findings include that:
• (A) the polygenic score predicts educational attainment in the African Americans in our
sample and
• (B) that the polygenic score is associated with neighborhood characteristics. As with earlier
research, we are able to show that the higher score sibling from within a family will complete
more years of schooling (on average) than their lower score co-sib.
Read the rest of the interviews on MedicalResearch.com
Content NOT an endorsement of efficacy and NOT intended as specific medical advice.
Polygenetic Score Predicts Educational Attainment
MedicalResearch.com Interview with:
Ben Domingue
Assistant Professor (starting 9/2015)
Stanford Graduate School of Education
• Medical Research: What should clinicians and patients take away from your report?
• Response: Our study adds to the growing body of knowledge about how genetics predict
outcomes that we care about. Although it is unlikely that this score would be used as a basis
for intervention, our results may serve as a useful basis for comparison for those polygenic
scores (e.g., for cardiovascular disease) which may be of clinical utility.
Read the rest of the interviews on MedicalResearch.com
Content NOT an endorsement of efficacy and NOT intended as specific medical advice.
Polygenetic Score Predicts Educational Attainment
MedicalResearch.com Interview with:
Ben Domingue
Assistant Professor (starting 9/2015)
Stanford Graduate School of Education
• Medical Research: What recommendations do you have for future research as a result of
this study?
• Response: As a social scientist, my attention will now be shifting towards research focusing
on how the predictive performance of this score may function as a product of the
environment in which a focal individual resides.
• Citation:
• W. Domingue, D. W. Belsky, D. Conley, K. M. Harris, J. D. Boardman.Polygenic Influence on
Educational Attainment: New Evidence From the National Longitudinal Study of Adolescent
to Adult Health. AERA Open, 2015; 1 (3) DOI: 10.1177/2332858415599972
Read the rest of the interviews on MedicalResearch.com
Content NOT an endorsement of efficacy and NOT intended as specific medical advice.
Extreme Obesity Can Cause Kidney Disease Especially In Elderly
MedicalResearch.com Interview with: Dr. Csaba P. Kovesdy, MD
Professor of Medicine University of Tennessee Health Science Center Chief of Nephrology
Memphis Veterans Affairs Medical Center
• Medical Research: What is the background for this study? What are the main findings?
• Dr. Kovesdy: Obesity has reached epidemic proportions in modern societies, and has been
linked to adverse outcomes such as diabetes mellitus, hypertension, cardiovascular disease
and mortality. In addition, obesity is also associated with chronic kidney disease through a
variety of mechanisms. Our population is ageing, and previous studies have suggested that
the effect of obesity on certain outcomes like mortality may be different in older vs. younger
individuals, but this has not been previously examined for chronic kidney disease. We have
this examined the association of granular BMI categories with progressive loss of kidney
function in a very large cohort of patients with normal estimated GFR in patients of different
ages. We found that the association of a BMI of >30 kg/m2 with progressive loss of kidney
function was not present in younger individuals (< 40 years of age), and increased as people
aged, with >80 years old displaying the strongest associations between obesity and loss of
kidney function. In addition to this we also examined the association of BMI with mortality in
different age groups, and found uniform U-shaped associations that did not vary by age.
Read the rest of the interviews on MedicalResearch.com
Content NOT an endorsement of efficacy and NOT intended as specific medical advice.
Extreme Obesity Can Cause Kidney Disease Especially In Elderly
MedicalResearch.com Interview with: Dr. Csaba P. Kovesdy, MD
Professor of Medicine University of Tennessee Health Science Center
Chief of Nephrology
Memphis Veterans Affairs Medical Center
• Medical Research: What should clinicians and patients take away from your report?
• Dr. Kovesdy: These results reinforce the widely held notion that extreme obesity can cause
kidney damage. The novelty of these data is that it seems that oldest patients are most prone
to such damage. This may be because ageing kidneys are more vulnerable to any kind of
insult, or because older individuals have been exposed to the ill-effects of obesity for a much
longer time, experiencing cumulative damage.
Read the rest of the interviews on MedicalResearch.com
Content NOT an endorsement of efficacy and NOT intended as specific medical advice.
Extreme Obesity Can Cause Kidney Disease Especially In Elderly
MedicalResearch.com Interview with: Dr. Csaba P. Kovesdy, MD
Professor of Medicine University of Tennessee Health Science Center
Chief of Nephrology
Memphis Veterans Affairs Medical Center
• Medical Research: What recommendations do you have for future research as a result of this
study?
• Dr. Kovesdy: The observational nature of our data does not allow us to make concrete practice
recommendations; that would require clinical trials of weight loss. Our data raises interesting
questions about when weight loss interventions should commence: if the kidney damage in older
individuals is due to the combined effects of age-related changes and obesity-related
pathophysiology, then interventions could be effective at any age, and especially when started in
the oldest individuals. If, on the other hand, kidney damage is more of a function of cumulative
damage experienced over many decades, then interventions started at earlier ages, and maintained
for a long time would be most effective. This dilemma will have to be answered by future
interventional studies.
• Another important practical point is the ideal BMI which is associated with the best clinical
outcomes. In our population this was a BMI of 25-30 kg/m2. This range has been supported by
several other studies in the recent past, and suggests that current guidelines about what an ideal
BMI should be may need to be re-evaluated in certain groups of patients.
• Citation:
• Association of age and BMI with kidney function and mortality: a cohort study
• Jun Ling Lu, MD, Miklos Z Molnar, MD, Adnan Naseer, MD,Margit K Mikkelsen, Prof Kamyar
Kalantar-Zadeh, MD,ProfCsaba P Kovesdy, MD
• The Lancet Diabetes & Endocrinology Published Online: 30 July 2015
• DOI: http://dx.doi.org/10.1016/S2213-8587(15)00128-X
Read the rest of the interviews on MedicalResearch.com
Content NOT an endorsement of efficacy and NOT intended as specific medical advice.
Mental and Physical Health Issues Common In Caregivers of Stroke Survivors
MedicalResearch.com Interview with: Josefine Persson Doctoral student
Institute of Neuroscience and Physiology Sahlgrenska Academy
University of Gothenburg
• Medical Research: What is the background for this study?
• Response: Stroke is a major global disease that requires extensive care and support from the
society and the family. We know from previous research that a stroke often has a wide-
spread impact on the daily life of the family. To provide support to a partner is often
perceived as natural and important, but can be demanding and have an impact on the
spouses own health. The situation for spouses as caregivers is well studied during the first
two years after the stroke, while the long-term effects are less well known. By this, we
studied the physical and mental health of 248 spouses of stroke survivors, below age 70 at
stroke onset, seven years after the stroke event and compared our result with 245 spouses of
non-stroke, age- and sex-matched controls.
Read the rest of the interviews on MedicalResearch.com
Content NOT an endorsement of efficacy and NOT intended as specific medical advice.
Mental and Physical Health Issues Common In Caregivers of Stroke Survivors
MedicalResearch.com Interview with: Josefine Persson Doctoral student
Institute of Neuroscience and Physiology Sahlgrenska Academy
University of Gothenburg
• Medical Research: What are the main findings?
• Response: The main finding of our study is that caregiver spouses of stroke survivors are at
an increased risk of mental and physical health issues even seven years after stroke onset.
This is the first study with this long period of follow up and the results show that the
restriction on the spouses own activity and social relationships studied in shorter follow up is
also obvious for a large proportion of the spouses in a very long perspective. Spouses’ quality
of life was most adversely affected by their partners’ level of disability, cognitive difficulties
and depressive symptoms.
Read the rest of the interviews on MedicalResearch.com
Content NOT an endorsement of efficacy and NOT intended as specific medical advice.
Mental and Physical Health Issues Common In Caregivers of Stroke Survivors
MedicalResearch.com Interview with: Josefine Persson Doctoral student
Institute of Neuroscience and Physiology Sahlgrenska Academy
University of Gothenburg
• Medical Research: What should clinicians and patients take away from your report?
• Response: The findings of this study suggest the importance for long-lasting, target societal
support to the spouses to prevent or reduce the burden, perceived stress and strain which
might contribute to health benefits, strength to fulfil goals in work and education reduced
risk of disease. In addition, our study indicates that the level of depression and level of
disability of the stroke survivor is associated with the spouses’ physical and mental health,
which strengthens the need for adequate workarounds for rehabilitation and follow-up
including spouses.
• Furthermore, our results also show that the spouses of stroke survivors reported lower
general health than the spouses of controls, which might be due to perceived stress or strain
for a long period, or due to shared lifestyle factors. Hence, it is important for the society to
provide support to prevent or reduce the burden on spouses and health promotion to
prevent unhealthy lifestyle. A societal effort could be aimed to reduce the burden on spouses
and to provide health promotion against an unhealthy lifestyle.
Read the rest of the interviews on MedicalResearch.com
Content NOT an endorsement of efficacy and NOT intended as specific medical advice.
Mental and Physical Health Issues Common In Caregivers of Stroke Survivors
MedicalResearch.com Interview with: Josefine Persson Doctoral student
Institute of Neuroscience and Physiology Sahlgrenska Academy
University of Gothenburg
• Medical Research: What recommendations do you have for future research as a result of
this study?
• Response: The spouses who are at risk are those who experience a supporting role towards
the stroke survivors, especially if the spouses have experienced lack of social support. Further
research is needed to explore the demanding situation for the supporting spouses and to
investigate the quantity of time spent on providing support as well as which components it
contains. A key question is to understand how the caregiver role impacts the spouses own
health and how information and support can positively affect their own health and
cohabitation.
• Citation:
• Spouses of Stroke Survivors Report Reduced Health-Related Quality of Life Even in Long-Term
Follow-Up: Results From Sahlgrenska Academy Study on Ischemic Stroke
• Josefine Persson, Lukas Holmegaard, Ingvar Karlberg, Petra Redfors,Katarina Jood, Christina
Jern, Christian Blomstrand, and Gunilla Forsberg-Wärleby
• Stroke. 2015;STROKEAHA.115.009791published online before print August 20
2015, doi:10.1161/STROKEAHA.115.009791
Read the rest of the interviews on MedicalResearch.com
Content NOT an endorsement of efficacy and NOT intended as specific medical advice.
Fortunately, Good Moods Are Contagious But Depression Is Not
MedicalResearch.com Interview with:
Edward Hill PhD student Centre for Complexity Science
Member of the Warwick Infectious Disease Epidemiology Research Centre (WIDER)
at the University of Warwick
• Medical Research: What is the background for this study?
• Response: Depression is a major public health concern worldwide. We know social factors,
such as living alone, can influence whether someone becomes depressed. We also know that
social support (having people to talk to) is important for recovery from depression.
• Our study is slightly different as we looked at the effect of being friends with people on
whether you are likely to develop depression or recover from being depressed. To do this, we
looked at over 2,000 adolescents in a network of US high school students to see how their
mood influenced each other.
Read the rest of the interviews on MedicalResearch.com
Content NOT an endorsement of efficacy and NOT intended as specific medical advice.
Fortunately, Good Moods Are Contagious But Depression Is Not
MedicalResearch.com Interview with:
Edward Hill PhD student Centre for Complexity Science
Member of the Warwick Infectious Disease Epidemiology Research Centre (WIDER)
at the University of Warwick
• Medical Research: What are the main findings?
• Response: Our study revealed that having mentally healthy friends can help someone recover
from depression or even remain mentally healthy in the first place. But crucially, having
depressed friends does not make you more likely to become depressed yourself. In other
words, it is healthy mood that spreads through social networks but depression does not.
• In particular, an individual with healthy mood having five or more friends with a healthy
mood can halve their probability of developing depression over a six-to-12 month period,
compared to those with no friends with healthy mood. Over a similar time period, a
depressed individual having ten healthy-mood friends doubles their probability of recovering
from depression compared to those who have just three such friends.
Read the rest of the interviews on MedicalResearch.com
Content NOT an endorsement of efficacy and NOT intended as specific medical advice.
Fortunately, Good Moods Are Contagious But Depression Is Not
MedicalResearch.com Interview with:
Edward Hill PhD student Centre for Complexity Science
Member of the Warwick Infectious Disease Epidemiology Research Centre (WIDER)
at the University of Warwick
• Medical Research: What should clinicians and patients take away from your report?
• Response: A potential way to tackle depression among young people would be to simply
encourage friendships.Having depressed friends does not put an individual at risk of getting
depression themselves, but having healthy friends is both protective and curative. If every
teenager had more friends it could go some way to providing them with a protective effect
against mental health problems. This also supports the idea that depression should be a
public health issue rather than an individual one and should be used to help counter the
stigma attached to the condition.
Read the rest of the interviews on MedicalResearch.com
Content NOT an endorsement of efficacy and NOT intended as specific medical advice.
Fortunately, Good Moods Are Contagious But Depression Is Not
MedicalResearch.com Interview with:
Edward Hill PhD student Centre for Complexity Science
Member of the Warwick Infectious Disease Epidemiology Research Centre (WIDER)
at the University of Warwick
• Medical Research: What recommendations do you have for future research as a result of
this study?
• Response: In the context of mood, it is important to stress that we do not believe that the
analogy with viral spreading is literally true, but rather that it emerges from the behaviour of
a complex social system. Future work could therefore try to look in more detail at how
different aspects of mood are influenced by friends to understand this phenomenon better.
• Furthermore, our methods could be used in other areas of public-health importance,
investigating if smoking or obesity spreads for example.
• Citation:
• Spreading of healthy mood in adolescent social networks
• E. M. Hill, F. E. Griffiths, T. House
• Published 19 August 2015. DOI: 10.1098/rspb.2015.1180
Read the rest of the interviews on MedicalResearch.com
Content NOT an endorsement of efficacy and NOT intended as specific medical advice.
Chemotherapy and Radiation For Brain Cancer Lead To Brain Shrinkage
MedicalResearch.com Interview with: Jorg Dietrich, MBA MMSc MD PhD
Director, Cancer & Neurotoxicity Clinic and Brain Repair Research Program
Massachusetts General Hospital Cancer Center Assistant Professor of Neurology
Harvard Medical School
• Medical Research: What is the background for this study? What are the main findings?
• Dr. Dietrich: Understanding the adverse effects associated with cancer therapy is an
important issue in oncology. Specifically, management of acute and delayed neurotoxicity of
chemotherapy and radiation in brain cancer patients has been challenging. There is an unmet
clinical need to better characterize the effects of standard cancer therapy on the normal
brain and to identify patients at risk of developing neurotoxicity. In this regard, identifying
novel biomarkers of neurotoxicity is essential to develop strategies to protect the brain and
promote repair of treatment-induced damage.
• In this study, we demonstrate that standard chemotherapy and radiation in patients treated
for glioblastoma is associated with progressive brain volume loss and damage to gray matter
– the area of the brain that contains most neurons.
• A cohort of 14 patients underwent sequential magnetic resonance imaging studies prior to,
during and following standard chemoradiation to characterize the pattern of structural
changes that occur as a consequence of treatment.
Read the rest of the interviews on MedicalResearch.com
Content NOT an endorsement of efficacy and NOT intended as specific medical advice.
Chemotherapy and Radiation For Brain Cancer Lead To Brain Shrinkage
MedicalResearch.com Interview with: Jorg Dietrich, MBA MMSc MD PhD
Director, Cancer & Neurotoxicity Clinic and Brain Repair Research Program
Massachusetts General Hospital Cancer Center Assistant Professor of Neurology
Harvard Medical School
• Medical Research: What should clinicians and patients take away from your report?
• Dr. Dietrich: The effects of chemotherapy and radiation on the brain remain poorly
understood. With an increasing number of long-term survivors, it is increasingly important to
better understand how cancer therapy affects the nervous system and to identify patients at
highest risk to develop neurotoxic symptoms.
• The findings from this study should not discourage the use of chemotherapy and radiation, as
these remain the standard of care to fight brain cancer. In contrast, the results from this
study should promote further investigations in this field to minimize the harmful effects of
existing treatments on the brain.
• Importantly, understanding the mechanisms of neurotoxicity will be the basis for
development of more selective therapies designed to avoid harmful effects on normal
tissues, and to improve quality of life of patients.
Read the rest of the interviews on MedicalResearch.com
Content NOT an endorsement of efficacy and NOT intended as specific medical advice.
Chemotherapy and Radiation For Brain Cancer Lead To Brain Shrinkage
MedicalResearch.com Interview with: Jorg Dietrich, MBA MMSc MD PhD
Director, Cancer & Neurotoxicity Clinic and Brain Repair Research Program
Massachusetts General Hospital Cancer Center Assistant Professor of Neurology
Harvard Medical School
• Medical Research: What recommendations do you have for future research as a result of
this study?
• Dr. Dietrich: It remains unclear whether the structural changes that we have observed in our
study are associated with neurocognitive symptoms, such as memory impairment. This is the
subject of ongoing investigations. Future studies will have to demonstrate how structural
changes in the brain are linked to functional outcomes, such as memory problems, cognitive
impairment and mood changes.
• Citation:
• Standard chemoradiation for glioblastoma results in progressive brain volume loss
• Morgan J. Prust, MD, Kourosh Jafari-Khouzani, PhD, Jayashree Kalpathy-Cramer, PhD, Pavlina
Polaskova, MD,Tracy T. Batchelor, MD, Elizabeth R. Gerstner, MD and Jorg Dietrich, MD, PhD
• Published online before print July 24, 2015,
• doi: http:/ / dx. doi. org/ 10. 1212/ WNL. 0000000000001861
• Neurology 10.1212/WNL.0000000000001861
Read the rest of the interviews on MedicalResearch.com
Content NOT an endorsement of efficacy and NOT intended as specific medical advice.
Up to 70% of Marijuana Users Also Use Tobacco With Complex Results
MedicalResearch.com Interview with:
Francesca M. Filbey PhD School of Behavioral and Brain Sciences
Center for Brain Health University of Texas at Dallas
Dallas, TX
• Medical Research: What is the background for this study? What are the main findings?
• Dr. Filbey: Most studies exclude tobacco users from participating, but 70% of marijuana users
also use tobacco. We were interested in investigating the combined effects of marijuana and
tobacco. Our research targeted the hippocampus because smaller hippocampal size is
associated with marijuana use. We chose to study short term memory because the
hippocampus is an area of the brain associated with memory and learning. The main finding
was surprising. The smaller the hippocampus in the marijuana plus nicotine user, the greater
the memory performance. We expected the opposite, which was true of the non-using
control group.
Read the rest of the interviews on MedicalResearch.com
Content NOT an endorsement of efficacy and NOT intended as specific medical advice.
Up to 70% of Marijuana Users Also Use Tobacco With Complex Results
MedicalResearch.com Interview with:
Francesca M. Filbey PhD School of Behavioral and Brain Sciences
Center for Brain Health University of Texas at Dallas
Dallas, TX
• Medical Research: What should clinicians and patients take away from your report?
• Dr. Filbey: It is important to note that substances interact with each other, and, therefore,
combined use could lead to complex effects. More research needs to be done to fully
understand the effects of marijuana on the brain. Very rarely are individuals using one type of
substance and it is very difficult to generalize findings from these studies across the board. It
is hard to know what the effects are of marijuana, for instance, if we don’t take into account
other substance use such as tobacco.
Read the rest of the interviews on MedicalResearch.com
Content NOT an endorsement of efficacy and NOT intended as specific medical advice.
Up to 70% of Marijuana Users Also Use Tobacco With Complex Results
MedicalResearch.com Interview with:
Francesca M. Filbey PhD School of Behavioral and Brain Sciences
Center for Brain Health University of Texas at Dallas
Dallas, TX
• Medical Research: What recommendations do you have for future research as a result of
this study?
• Dr. Filbey: Future studies should consider directly examining the combined effects of these
highly prevalently co-used substances. More research is also needed to further characterize
the relationship between the hippocampus and memory in combined users – for instance
brain connectivity, and the role of other brain regions such as the prefrontal cortex.
• Citation:
• Francesca M. Filbey, Tim McQueeny, Shrinath Kadamangudi, Collette Bice, Ariel
Ketcherside. Combined effects of marijuana and nicotine on memory performance and
hippocampal volume. Behavioural Brain Research, 2015; 293: 46
DOI: 10.1016/j.bbr.2015.07.029
Read the rest of the interviews on MedicalResearch.com
Content NOT an endorsement of efficacy and NOT intended as specific medical advice.
Commercial Brain Training Device Reduces Working Memory
MedicalResearch.com Interview with: Laura Steenbergen, MSc., PhD Candidate Cognitive Psychology at Institute of Psychology
Leiden University
• Medical Research: What is the background for this study? What are the main findings?
• Response: A recent initiative supported by several eminent research institutes and scientists
calls for a more critical and active role of the scientific community in evaluating the
sometimes far-reaching, sweeping claims from the brain training industry with regard to the
impact of their products on cognitive performance. tDCS is a noninvasive brain stimulation
technique that has developed into a promising tool to boost human cognition. Previous
studies using medical tDCS devices have shown that tDCS promotes working memory (WM)
updating in healthy individuals and patients. The aim of the current study was to investigate
whether the commercial tDCS headset foc.us (v.1), which is easily and freely available to
anyone in the world, does in fact improve cognitive performance, as advertised in the media.
Results showed that active stimulation with the commercial device, compared to sham
stimulation, significantly decreased working memory performance. The device we tested is
just one example of a commercial device that can easily be purchased and, without any
control or expert knowledge, used by anyone. The results of our study are straightforward in
showing that the claims made by companies manufacturing such devices need to be
validated. Even if the consequences of long-term or frequent use of the device are yet to be
demonstrated, our findings provide strong support the important role of the scientific
community in validating and testing far reaching claims made by the brain training industry.
Read the rest of the interviews on MedicalResearch.com
Content NOT an endorsement of efficacy and NOT intended as specific medical advice.
MedicalResearch.com Leading Medical Research Interviews August 24 2015
MedicalResearch.com Leading Medical Research Interviews August 24 2015
MedicalResearch.com Leading Medical Research Interviews August 24 2015
MedicalResearch.com Leading Medical Research Interviews August 24 2015
MedicalResearch.com Leading Medical Research Interviews August 24 2015
MedicalResearch.com Leading Medical Research Interviews August 24 2015
MedicalResearch.com Leading Medical Research Interviews August 24 2015
MedicalResearch.com Leading Medical Research Interviews August 24 2015
MedicalResearch.com Leading Medical Research Interviews August 24 2015
MedicalResearch.com Leading Medical Research Interviews August 24 2015
MedicalResearch.com Leading Medical Research Interviews August 24 2015
MedicalResearch.com Leading Medical Research Interviews August 24 2015
MedicalResearch.com Leading Medical Research Interviews August 24 2015
MedicalResearch.com Leading Medical Research Interviews August 24 2015
MedicalResearch.com Leading Medical Research Interviews August 24 2015
MedicalResearch.com Leading Medical Research Interviews August 24 2015
MedicalResearch.com Leading Medical Research Interviews August 24 2015
MedicalResearch.com Leading Medical Research Interviews August 24 2015
MedicalResearch.com Leading Medical Research Interviews August 24 2015
MedicalResearch.com Leading Medical Research Interviews August 24 2015
MedicalResearch.com Leading Medical Research Interviews August 24 2015
MedicalResearch.com Leading Medical Research Interviews August 24 2015
MedicalResearch.com Leading Medical Research Interviews August 24 2015
MedicalResearch.com Leading Medical Research Interviews August 24 2015
MedicalResearch.com Leading Medical Research Interviews August 24 2015
MedicalResearch.com Leading Medical Research Interviews August 24 2015
MedicalResearch.com Leading Medical Research Interviews August 24 2015
MedicalResearch.com Leading Medical Research Interviews August 24 2015
MedicalResearch.com Leading Medical Research Interviews August 24 2015
MedicalResearch.com Leading Medical Research Interviews August 24 2015
MedicalResearch.com Leading Medical Research Interviews August 24 2015
MedicalResearch.com Leading Medical Research Interviews August 24 2015
MedicalResearch.com Leading Medical Research Interviews August 24 2015
MedicalResearch.com Leading Medical Research Interviews August 24 2015
MedicalResearch.com Leading Medical Research Interviews August 24 2015
MedicalResearch.com Leading Medical Research Interviews August 24 2015
MedicalResearch.com Leading Medical Research Interviews August 24 2015
MedicalResearch.com Leading Medical Research Interviews August 24 2015
MedicalResearch.com Leading Medical Research Interviews August 24 2015
MedicalResearch.com Leading Medical Research Interviews August 24 2015
MedicalResearch.com Leading Medical Research Interviews August 24 2015
MedicalResearch.com Leading Medical Research Interviews August 24 2015
MedicalResearch.com Leading Medical Research Interviews August 24 2015
MedicalResearch.com Leading Medical Research Interviews August 24 2015
MedicalResearch.com Leading Medical Research Interviews August 24 2015
MedicalResearch.com Leading Medical Research Interviews August 24 2015
MedicalResearch.com Leading Medical Research Interviews August 24 2015
MedicalResearch.com Leading Medical Research Interviews August 24 2015
MedicalResearch.com Leading Medical Research Interviews August 24 2015
MedicalResearch.com Leading Medical Research Interviews August 24 2015
MedicalResearch.com Leading Medical Research Interviews August 24 2015
MedicalResearch.com Leading Medical Research Interviews August 24 2015
MedicalResearch.com Leading Medical Research Interviews August 24 2015
MedicalResearch.com Leading Medical Research Interviews August 24 2015
MedicalResearch.com Leading Medical Research Interviews August 24 2015
MedicalResearch.com Leading Medical Research Interviews August 24 2015
MedicalResearch.com Leading Medical Research Interviews August 24 2015
MedicalResearch.com Leading Medical Research Interviews August 24 2015
MedicalResearch.com Leading Medical Research Interviews August 24 2015
MedicalResearch.com Leading Medical Research Interviews August 24 2015
MedicalResearch.com Leading Medical Research Interviews August 24 2015
MedicalResearch.com Leading Medical Research Interviews August 24 2015
MedicalResearch.com Leading Medical Research Interviews August 24 2015
MedicalResearch.com Leading Medical Research Interviews August 24 2015
MedicalResearch.com Leading Medical Research Interviews August 24 2015
MedicalResearch.com Leading Medical Research Interviews August 24 2015
MedicalResearch.com Leading Medical Research Interviews August 24 2015
MedicalResearch.com Leading Medical Research Interviews August 24 2015
MedicalResearch.com Leading Medical Research Interviews August 24 2015
MedicalResearch.com Leading Medical Research Interviews August 24 2015
MedicalResearch.com Leading Medical Research Interviews August 24 2015
MedicalResearch.com Leading Medical Research Interviews August 24 2015
MedicalResearch.com Leading Medical Research Interviews August 24 2015
MedicalResearch.com Leading Medical Research Interviews August 24 2015
MedicalResearch.com Leading Medical Research Interviews August 24 2015

More Related Content

What's hot

Web only rx16 pharma wed_200_1_hagemeier_2fleming_3vernachio
Web only rx16 pharma wed_200_1_hagemeier_2fleming_3vernachioWeb only rx16 pharma wed_200_1_hagemeier_2fleming_3vernachio
Web only rx16 pharma wed_200_1_hagemeier_2fleming_3vernachioOPUNITE
 
Online patients: characteristics and behaviour on health social networks - fe...
Online patients: characteristics and behaviour on health social networks - fe...Online patients: characteristics and behaviour on health social networks - fe...
Online patients: characteristics and behaviour on health social networks - fe...Ricardo Sousa
 
Linkage to Care Strategies
Linkage to Care StrategiesLinkage to Care Strategies
Linkage to Care StrategiesRandi Sylve
 
Patient engagement in medication safety at the point of care – roles, respons...
Patient engagement in medication safety at the point of care – roles, respons...Patient engagement in medication safety at the point of care – roles, respons...
Patient engagement in medication safety at the point of care – roles, respons...Canadian Patient Safety Institute
 
The context of dropping out of ART programs in
The context of dropping out of ART programs in The context of dropping out of ART programs in
The context of dropping out of ART programs in MEASURE Evaluation
 
Pittsburgh Nonprofit Summit - Health Care & Health Care Reform - Implications...
Pittsburgh Nonprofit Summit - Health Care & Health Care Reform - Implications...Pittsburgh Nonprofit Summit - Health Care & Health Care Reform - Implications...
Pittsburgh Nonprofit Summit - Health Care & Health Care Reform - Implications...GPNP
 
Integrating Behavioral Health into Primary Care – Thought Leaders in Populati...
Integrating Behavioral Health into Primary Care – Thought Leaders in Populati...Integrating Behavioral Health into Primary Care – Thought Leaders in Populati...
Integrating Behavioral Health into Primary Care – Thought Leaders in Populati...Epstein Becker Green
 
What kind of ancestor will you be - ILN Insights Volume 8
What kind of ancestor will you be  - ILN Insights Volume 8What kind of ancestor will you be  - ILN Insights Volume 8
What kind of ancestor will you be - ILN Insights Volume 8Ted Eytan, MD, MS, MPH
 
NMA Presentation Ddey 04Jun08
NMA Presentation Ddey 04Jun08NMA Presentation Ddey 04Jun08
NMA Presentation Ddey 04Jun08aplace2relax
 
Vilma-'s resume-1(4) copy
Vilma-'s resume-1(4) copyVilma-'s resume-1(4) copy
Vilma-'s resume-1(4) copyVilma Mejia
 
HSOM Final Project (Actual)
HSOM Final Project (Actual)HSOM Final Project (Actual)
HSOM Final Project (Actual)Ian Brewer
 
Claire Foley & Tracy Torchetti - Editing Health Information for a Limited Eng...
Claire Foley & Tracy Torchetti - Editing Health Information for a Limited Eng...Claire Foley & Tracy Torchetti - Editing Health Information for a Limited Eng...
Claire Foley & Tracy Torchetti - Editing Health Information for a Limited Eng...Plain Talk 2015
 
What Social Media Can Tell Us About The Patient Journey
What Social Media Can Tell Us About The Patient JourneyWhat Social Media Can Tell Us About The Patient Journey
What Social Media Can Tell Us About The Patient Journeye-Patient Connections
 
Health Insurance Information Needs: How Librarians Can Help
Health Insurance Information Needs: How Librarians Can HelpHealth Insurance Information Needs: How Librarians Can Help
Health Insurance Information Needs: How Librarians Can Helpevardell
 
Steve Sparks - "Let's Talk about the Flu": Communicating Health Prevention to...
Steve Sparks - "Let's Talk about the Flu": Communicating Health Prevention to...Steve Sparks - "Let's Talk about the Flu": Communicating Health Prevention to...
Steve Sparks - "Let's Talk about the Flu": Communicating Health Prevention to...Plain Talk 2015
 
Va Health Literacy Research Presentation
Va Health Literacy Research PresentationVa Health Literacy Research Presentation
Va Health Literacy Research Presentationguest169e62f
 
Americans Difficulty Attaining Wellness | Providers Difficulty Delivering Value
Americans Difficulty Attaining Wellness | Providers Difficulty Delivering ValueAmericans Difficulty Attaining Wellness | Providers Difficulty Delivering Value
Americans Difficulty Attaining Wellness | Providers Difficulty Delivering ValueNick Gaudiosi
 
Key Principles and Approaches to Populaiton Health mManagement - HAS Session 21
Key Principles and Approaches to Populaiton Health mManagement - HAS Session 21Key Principles and Approaches to Populaiton Health mManagement - HAS Session 21
Key Principles and Approaches to Populaiton Health mManagement - HAS Session 21Health Catalyst
 
China health management survey report, 2011
China health management survey report, 2011China health management survey report, 2011
China health management survey report, 2011ResearchInChina
 

What's hot (20)

Web only rx16 pharma wed_200_1_hagemeier_2fleming_3vernachio
Web only rx16 pharma wed_200_1_hagemeier_2fleming_3vernachioWeb only rx16 pharma wed_200_1_hagemeier_2fleming_3vernachio
Web only rx16 pharma wed_200_1_hagemeier_2fleming_3vernachio
 
Online patients: characteristics and behaviour on health social networks - fe...
Online patients: characteristics and behaviour on health social networks - fe...Online patients: characteristics and behaviour on health social networks - fe...
Online patients: characteristics and behaviour on health social networks - fe...
 
Linkage to Care Strategies
Linkage to Care StrategiesLinkage to Care Strategies
Linkage to Care Strategies
 
Patient engagement in medication safety at the point of care – roles, respons...
Patient engagement in medication safety at the point of care – roles, respons...Patient engagement in medication safety at the point of care – roles, respons...
Patient engagement in medication safety at the point of care – roles, respons...
 
The context of dropping out of ART programs in
The context of dropping out of ART programs in The context of dropping out of ART programs in
The context of dropping out of ART programs in
 
Pittsburgh Nonprofit Summit - Health Care & Health Care Reform - Implications...
Pittsburgh Nonprofit Summit - Health Care & Health Care Reform - Implications...Pittsburgh Nonprofit Summit - Health Care & Health Care Reform - Implications...
Pittsburgh Nonprofit Summit - Health Care & Health Care Reform - Implications...
 
Integrating Behavioral Health into Primary Care – Thought Leaders in Populati...
Integrating Behavioral Health into Primary Care – Thought Leaders in Populati...Integrating Behavioral Health into Primary Care – Thought Leaders in Populati...
Integrating Behavioral Health into Primary Care – Thought Leaders in Populati...
 
What kind of ancestor will you be - ILN Insights Volume 8
What kind of ancestor will you be  - ILN Insights Volume 8What kind of ancestor will you be  - ILN Insights Volume 8
What kind of ancestor will you be - ILN Insights Volume 8
 
NMA Presentation Ddey 04Jun08
NMA Presentation Ddey 04Jun08NMA Presentation Ddey 04Jun08
NMA Presentation Ddey 04Jun08
 
Vilma-'s resume-1(4) copy
Vilma-'s resume-1(4) copyVilma-'s resume-1(4) copy
Vilma-'s resume-1(4) copy
 
HSOM Final Project (Actual)
HSOM Final Project (Actual)HSOM Final Project (Actual)
HSOM Final Project (Actual)
 
Claire Foley & Tracy Torchetti - Editing Health Information for a Limited Eng...
Claire Foley & Tracy Torchetti - Editing Health Information for a Limited Eng...Claire Foley & Tracy Torchetti - Editing Health Information for a Limited Eng...
Claire Foley & Tracy Torchetti - Editing Health Information for a Limited Eng...
 
What Social Media Can Tell Us About The Patient Journey
What Social Media Can Tell Us About The Patient JourneyWhat Social Media Can Tell Us About The Patient Journey
What Social Media Can Tell Us About The Patient Journey
 
Health Insurance Information Needs: How Librarians Can Help
Health Insurance Information Needs: How Librarians Can HelpHealth Insurance Information Needs: How Librarians Can Help
Health Insurance Information Needs: How Librarians Can Help
 
Steve Sparks - "Let's Talk about the Flu": Communicating Health Prevention to...
Steve Sparks - "Let's Talk about the Flu": Communicating Health Prevention to...Steve Sparks - "Let's Talk about the Flu": Communicating Health Prevention to...
Steve Sparks - "Let's Talk about the Flu": Communicating Health Prevention to...
 
The Health Literacy Questionnaire (HLQ): A new approach to measuring health l...
The Health Literacy Questionnaire (HLQ): A new approach to measuring health l...The Health Literacy Questionnaire (HLQ): A new approach to measuring health l...
The Health Literacy Questionnaire (HLQ): A new approach to measuring health l...
 
Va Health Literacy Research Presentation
Va Health Literacy Research PresentationVa Health Literacy Research Presentation
Va Health Literacy Research Presentation
 
Americans Difficulty Attaining Wellness | Providers Difficulty Delivering Value
Americans Difficulty Attaining Wellness | Providers Difficulty Delivering ValueAmericans Difficulty Attaining Wellness | Providers Difficulty Delivering Value
Americans Difficulty Attaining Wellness | Providers Difficulty Delivering Value
 
Key Principles and Approaches to Populaiton Health mManagement - HAS Session 21
Key Principles and Approaches to Populaiton Health mManagement - HAS Session 21Key Principles and Approaches to Populaiton Health mManagement - HAS Session 21
Key Principles and Approaches to Populaiton Health mManagement - HAS Session 21
 
China health management survey report, 2011
China health management survey report, 2011China health management survey report, 2011
China health management survey report, 2011
 

Viewers also liked

MedicalResearch.com: Medical Research Interviews March 2014
MedicalResearch.com:  Medical Research Interviews March 2014MedicalResearch.com:  Medical Research Interviews March 2014
MedicalResearch.com: Medical Research Interviews March 2014Marie Benz MD FAAD
 
MedicalResearch.com: Medical Research Exclusive Interviews October 27 2014
MedicalResearch.com:  Medical Research Exclusive Interviews October 27  2014MedicalResearch.com:  Medical Research Exclusive Interviews October 27  2014
MedicalResearch.com: Medical Research Exclusive Interviews October 27 2014Marie Benz MD FAAD
 
MedicalResearch.com: Medical Research Updates and Interviews March 27 2014
MedicalResearch.com:  Medical Research Updates and Interviews March 27 2014MedicalResearch.com:  Medical Research Updates and Interviews March 27 2014
MedicalResearch.com: Medical Research Updates and Interviews March 27 2014Marie Benz MD FAAD
 
MedicalResearch.com: Medical Research Exclusive Interviews November 26 2014
MedicalResearch.com:  Medical Research Exclusive Interviews November 26 2014MedicalResearch.com:  Medical Research Exclusive Interviews November 26 2014
MedicalResearch.com: Medical Research Exclusive Interviews November 26 2014Marie Benz MD FAAD
 
MedicalResearch.com: Medical Research Exclusive Interviews July 20 2015
MedicalResearch.com:  Medical Research Exclusive Interviews July 20  2015MedicalResearch.com:  Medical Research Exclusive Interviews July 20  2015
MedicalResearch.com: Medical Research Exclusive Interviews July 20 2015Marie Benz MD FAAD
 
MedicalResearch.com: Medical Research Exclusive Interviews October 18 2014
MedicalResearch.com:  Medical Research Exclusive Interviews October 18  2014MedicalResearch.com:  Medical Research Exclusive Interviews October 18  2014
MedicalResearch.com: Medical Research Exclusive Interviews October 18 2014Marie Benz MD FAAD
 
MedicalResearch.com: Medical Research Exclusive Interviews August 8 2015
MedicalResearch.com:  Medical Research Exclusive Interviews August 8 2015MedicalResearch.com:  Medical Research Exclusive Interviews August 8 2015
MedicalResearch.com: Medical Research Exclusive Interviews August 8 2015Marie Benz MD FAAD
 
MedicalResearch.com: Medical Research Exclusive Interviews April 1 2015
MedicalResearch.com:  Medical Research Exclusive Interviews April 1  2015MedicalResearch.com:  Medical Research Exclusive Interviews April 1  2015
MedicalResearch.com: Medical Research Exclusive Interviews April 1 2015Marie Benz MD FAAD
 
MedicalResearch.com: Medical Research Interviews September 22 2014
MedicalResearch.com:  Medical Research Interviews September 22 2014MedicalResearch.com:  Medical Research Interviews September 22 2014
MedicalResearch.com: Medical Research Interviews September 22 2014Marie Benz MD FAAD
 
MedicalResearch.com: Medical Research Interviews September12 2014
MedicalResearch.com:  Medical Research Interviews September12 2014MedicalResearch.com:  Medical Research Interviews September12 2014
MedicalResearch.com: Medical Research Interviews September12 2014Marie Benz MD FAAD
 

Viewers also liked (10)

MedicalResearch.com: Medical Research Interviews March 2014
MedicalResearch.com:  Medical Research Interviews March 2014MedicalResearch.com:  Medical Research Interviews March 2014
MedicalResearch.com: Medical Research Interviews March 2014
 
MedicalResearch.com: Medical Research Exclusive Interviews October 27 2014
MedicalResearch.com:  Medical Research Exclusive Interviews October 27  2014MedicalResearch.com:  Medical Research Exclusive Interviews October 27  2014
MedicalResearch.com: Medical Research Exclusive Interviews October 27 2014
 
MedicalResearch.com: Medical Research Updates and Interviews March 27 2014
MedicalResearch.com:  Medical Research Updates and Interviews March 27 2014MedicalResearch.com:  Medical Research Updates and Interviews March 27 2014
MedicalResearch.com: Medical Research Updates and Interviews March 27 2014
 
MedicalResearch.com: Medical Research Exclusive Interviews November 26 2014
MedicalResearch.com:  Medical Research Exclusive Interviews November 26 2014MedicalResearch.com:  Medical Research Exclusive Interviews November 26 2014
MedicalResearch.com: Medical Research Exclusive Interviews November 26 2014
 
MedicalResearch.com: Medical Research Exclusive Interviews July 20 2015
MedicalResearch.com:  Medical Research Exclusive Interviews July 20  2015MedicalResearch.com:  Medical Research Exclusive Interviews July 20  2015
MedicalResearch.com: Medical Research Exclusive Interviews July 20 2015
 
MedicalResearch.com: Medical Research Exclusive Interviews October 18 2014
MedicalResearch.com:  Medical Research Exclusive Interviews October 18  2014MedicalResearch.com:  Medical Research Exclusive Interviews October 18  2014
MedicalResearch.com: Medical Research Exclusive Interviews October 18 2014
 
MedicalResearch.com: Medical Research Exclusive Interviews August 8 2015
MedicalResearch.com:  Medical Research Exclusive Interviews August 8 2015MedicalResearch.com:  Medical Research Exclusive Interviews August 8 2015
MedicalResearch.com: Medical Research Exclusive Interviews August 8 2015
 
MedicalResearch.com: Medical Research Exclusive Interviews April 1 2015
MedicalResearch.com:  Medical Research Exclusive Interviews April 1  2015MedicalResearch.com:  Medical Research Exclusive Interviews April 1  2015
MedicalResearch.com: Medical Research Exclusive Interviews April 1 2015
 
MedicalResearch.com: Medical Research Interviews September 22 2014
MedicalResearch.com:  Medical Research Interviews September 22 2014MedicalResearch.com:  Medical Research Interviews September 22 2014
MedicalResearch.com: Medical Research Interviews September 22 2014
 
MedicalResearch.com: Medical Research Interviews September12 2014
MedicalResearch.com:  Medical Research Interviews September12 2014MedicalResearch.com:  Medical Research Interviews September12 2014
MedicalResearch.com: Medical Research Interviews September12 2014
 

Similar to MedicalResearch.com Leading Medical Research Interviews August 24 2015

MedicalResearch.com: Medical Research Exclusive Interviews May 18 2015
MedicalResearch.com:  Medical Research Exclusive Interviews May 18 2015MedicalResearch.com:  Medical Research Exclusive Interviews May 18 2015
MedicalResearch.com: Medical Research Exclusive Interviews May 18 2015Marie Benz MD FAAD
 
MedicalResearch.com: Medical Research Exclusive Interviews March 24 2015
MedicalResearch.com:  Medical Research Exclusive Interviews March 24 2015MedicalResearch.com:  Medical Research Exclusive Interviews March 24 2015
MedicalResearch.com: Medical Research Exclusive Interviews March 24 2015Marie Benz MD FAAD
 
Medical research slideshare_june_24_2015
Medical research slideshare_june_24_2015 Medical research slideshare_june_24_2015
Medical research slideshare_june_24_2015 Marie Benz MD FAAD
 
MedicalResearch.com: Medical Research Exclusive Interviews November 21 2014
MedicalResearch.com:  Medical Research Exclusive Interviews November 21 2014MedicalResearch.com:  Medical Research Exclusive Interviews November 21 2014
MedicalResearch.com: Medical Research Exclusive Interviews November 21 2014Marie Benz MD FAAD
 
MedicalResearch.com: Medical Research Exclusive Interviews February 4 2015
MedicalResearch.com:  Medical Research Exclusive Interviews February 4 2015MedicalResearch.com:  Medical Research Exclusive Interviews February 4 2015
MedicalResearch.com: Medical Research Exclusive Interviews February 4 2015Marie Benz MD FAAD
 
MedicalResearch.com: Medical Research Exclusive Interviews July 24 2015
MedicalResearch.com:  Medical Research Exclusive Interviews July 24 2015MedicalResearch.com:  Medical Research Exclusive Interviews July 24 2015
MedicalResearch.com: Medical Research Exclusive Interviews July 24 2015Marie Benz MD FAAD
 
Medical research slideshare_june_18_2015
Medical research slideshare_june_18_2015 Medical research slideshare_june_18_2015
Medical research slideshare_june_18_2015 Marie Benz MD FAAD
 
Medical research slideshare_june_18_2015
Medical research slideshare_june_18_2015 Medical research slideshare_june_18_2015
Medical research slideshare_june_18_2015 Marie Benz MD FAAD
 
MedicalResearch.com: Medical Research Exclusive Interviews March 5 2015
MedicalResearch.com:  Medical Research Exclusive Interviews March 5  2015MedicalResearch.com:  Medical Research Exclusive Interviews March 5  2015
MedicalResearch.com: Medical Research Exclusive Interviews March 5 2015Marie Benz MD FAAD
 
MedicalResearch.com: Medical Research Exclusive Interviews February 10 2015
MedicalResearch.com:  Medical Research Exclusive Interviews February 10  2015MedicalResearch.com:  Medical Research Exclusive Interviews February 10  2015
MedicalResearch.com: Medical Research Exclusive Interviews February 10 2015Marie Benz MD FAAD
 
Medical research slideshare_may_6_2015
Medical research slideshare_may_6_2015 Medical research slideshare_may_6_2015
Medical research slideshare_may_6_2015 Marie Benz MD FAAD
 
MedicalResearch.com: Medical Research Exclusive Interviews December 31 2014
MedicalResearch.com:  Medical Research Exclusive Interviews December 31 2014MedicalResearch.com:  Medical Research Exclusive Interviews December 31 2014
MedicalResearch.com: Medical Research Exclusive Interviews December 31 2014Marie Benz MD FAAD
 
MedicalResearch.com Breaking Medical Research News August 30 2015
MedicalResearch.com Breaking Medical Research News August 30 2015MedicalResearch.com Breaking Medical Research News August 30 2015
MedicalResearch.com Breaking Medical Research News August 30 2015Marie Benz
 
MedicalResearch.com: Medical Research Exclusive Interviews July 16 2015
MedicalResearch.com:  Medical Research Exclusive Interviews July 16 2015MedicalResearch.com:  Medical Research Exclusive Interviews July 16 2015
MedicalResearch.com: Medical Research Exclusive Interviews July 16 2015Marie Benz MD FAAD
 
MedicalResearch.com: Medical Research Exclusive Interviews July 9 2015
MedicalResearch.com:  Medical Research Exclusive Interviews July 9 2015MedicalResearch.com:  Medical Research Exclusive Interviews July 9 2015
MedicalResearch.com: Medical Research Exclusive Interviews July 9 2015Marie Benz MD FAAD
 
MedicalResearch.com Top Medical Research Interviews September 25 2015
MedicalResearch.com Top Medical Research Interviews September 25 2015MedicalResearch.com Top Medical Research Interviews September 25 2015
MedicalResearch.com Top Medical Research Interviews September 25 2015Marie Benz
 
MedicalResearch.com: Medical Research Interviews September 3 2014
MedicalResearch.com:  Medical Research Interviews September 3 2014MedicalResearch.com:  Medical Research Interviews September 3 2014
MedicalResearch.com: Medical Research Interviews September 3 2014Marie Benz MD FAAD
 
MedicalResearch.com: Medical Research Interviews Month in Review
MedicalResearch.com:  Medical Research Interviews Month in ReviewMedicalResearch.com:  Medical Research Interviews Month in Review
MedicalResearch.com: Medical Research Interviews Month in ReviewMarie Benz MD FAAD
 

Similar to MedicalResearch.com Leading Medical Research Interviews August 24 2015 (20)

MedicalResearch.com: Medical Research Exclusive Interviews May 18 2015
MedicalResearch.com:  Medical Research Exclusive Interviews May 18 2015MedicalResearch.com:  Medical Research Exclusive Interviews May 18 2015
MedicalResearch.com: Medical Research Exclusive Interviews May 18 2015
 
MedicalResearch.com: Medical Research Exclusive Interviews March 24 2015
MedicalResearch.com:  Medical Research Exclusive Interviews March 24 2015MedicalResearch.com:  Medical Research Exclusive Interviews March 24 2015
MedicalResearch.com: Medical Research Exclusive Interviews March 24 2015
 
Medical research slideshare_june_24_2015
Medical research slideshare_june_24_2015 Medical research slideshare_june_24_2015
Medical research slideshare_june_24_2015
 
MedicalResearch.com: Medical Research Exclusive Interviews November 21 2014
MedicalResearch.com:  Medical Research Exclusive Interviews November 21 2014MedicalResearch.com:  Medical Research Exclusive Interviews November 21 2014
MedicalResearch.com: Medical Research Exclusive Interviews November 21 2014
 
MedicalResearch.com: Medical Research Exclusive Interviews February 4 2015
MedicalResearch.com:  Medical Research Exclusive Interviews February 4 2015MedicalResearch.com:  Medical Research Exclusive Interviews February 4 2015
MedicalResearch.com: Medical Research Exclusive Interviews February 4 2015
 
MedicalResearch.com: Medical Research Exclusive Interviews July 24 2015
MedicalResearch.com:  Medical Research Exclusive Interviews July 24 2015MedicalResearch.com:  Medical Research Exclusive Interviews July 24 2015
MedicalResearch.com: Medical Research Exclusive Interviews July 24 2015
 
Medical research slideshare_june_18_2015
Medical research slideshare_june_18_2015 Medical research slideshare_june_18_2015
Medical research slideshare_june_18_2015
 
Medical research slideshare_june_18_2015
Medical research slideshare_june_18_2015 Medical research slideshare_june_18_2015
Medical research slideshare_june_18_2015
 
MedicalResearch.com: Medical Research Exclusive Interviews March 5 2015
MedicalResearch.com:  Medical Research Exclusive Interviews March 5  2015MedicalResearch.com:  Medical Research Exclusive Interviews March 5  2015
MedicalResearch.com: Medical Research Exclusive Interviews March 5 2015
 
MedicalResearch.com: Medical Research Exclusive Interviews February 10 2015
MedicalResearch.com:  Medical Research Exclusive Interviews February 10  2015MedicalResearch.com:  Medical Research Exclusive Interviews February 10  2015
MedicalResearch.com: Medical Research Exclusive Interviews February 10 2015
 
Medical research slideshare_may_6_2015
Medical research slideshare_may_6_2015 Medical research slideshare_may_6_2015
Medical research slideshare_may_6_2015
 
MedicalResearch.com: Medical Research Exclusive Interviews December 31 2014
MedicalResearch.com:  Medical Research Exclusive Interviews December 31 2014MedicalResearch.com:  Medical Research Exclusive Interviews December 31 2014
MedicalResearch.com: Medical Research Exclusive Interviews December 31 2014
 
MedicalResearch.com Breaking Medical Research News August 30 2015
MedicalResearch.com Breaking Medical Research News August 30 2015MedicalResearch.com Breaking Medical Research News August 30 2015
MedicalResearch.com Breaking Medical Research News August 30 2015
 
MedicalResearch.com: Medical Research Exclusive Interviews July 16 2015
MedicalResearch.com:  Medical Research Exclusive Interviews July 16 2015MedicalResearch.com:  Medical Research Exclusive Interviews July 16 2015
MedicalResearch.com: Medical Research Exclusive Interviews July 16 2015
 
MedicalResearch.com: Medical Research Exclusive Interviews July 9 2015
MedicalResearch.com:  Medical Research Exclusive Interviews July 9 2015MedicalResearch.com:  Medical Research Exclusive Interviews July 9 2015
MedicalResearch.com: Medical Research Exclusive Interviews July 9 2015
 
E4P0815_PatientInterest_V6
E4P0815_PatientInterest_V6E4P0815_PatientInterest_V6
E4P0815_PatientInterest_V6
 
MedicalResearch.com Top Medical Research Interviews September 25 2015
MedicalResearch.com Top Medical Research Interviews September 25 2015MedicalResearch.com Top Medical Research Interviews September 25 2015
MedicalResearch.com Top Medical Research Interviews September 25 2015
 
MedicalResearch.com: Medical Research Interviews September 3 2014
MedicalResearch.com:  Medical Research Interviews September 3 2014MedicalResearch.com:  Medical Research Interviews September 3 2014
MedicalResearch.com: Medical Research Interviews September 3 2014
 
hospitalGuide2007
hospitalGuide2007hospitalGuide2007
hospitalGuide2007
 
MedicalResearch.com: Medical Research Interviews Month in Review
MedicalResearch.com:  Medical Research Interviews Month in ReviewMedicalResearch.com:  Medical Research Interviews Month in Review
MedicalResearch.com: Medical Research Interviews Month in Review
 

Recently uploaded

Artifacts in Nuclear Medicine with Identifying and resolving artifacts.
Artifacts in Nuclear Medicine with Identifying and resolving artifacts.Artifacts in Nuclear Medicine with Identifying and resolving artifacts.
Artifacts in Nuclear Medicine with Identifying and resolving artifacts.MiadAlsulami
 
Bangalore Call Girls Majestic 📞 9907093804 High Profile Service 100% Safe
Bangalore Call Girls Majestic 📞 9907093804 High Profile Service 100% SafeBangalore Call Girls Majestic 📞 9907093804 High Profile Service 100% Safe
Bangalore Call Girls Majestic 📞 9907093804 High Profile Service 100% Safenarwatsonia7
 
Call Girls Service Chennai Jiya 7001305949 Independent Escort Service Chennai
Call Girls Service Chennai Jiya 7001305949 Independent Escort Service ChennaiCall Girls Service Chennai Jiya 7001305949 Independent Escort Service Chennai
Call Girls Service Chennai Jiya 7001305949 Independent Escort Service ChennaiNehru place Escorts
 
Call Girls Cuttack Just Call 9907093804 Top Class Call Girl Service Available
Call Girls Cuttack Just Call 9907093804 Top Class Call Girl Service AvailableCall Girls Cuttack Just Call 9907093804 Top Class Call Girl Service Available
Call Girls Cuttack Just Call 9907093804 Top Class Call Girl Service AvailableDipal Arora
 
VIP Mumbai Call Girls Hiranandani Gardens Just Call 9920874524 with A/C Room ...
VIP Mumbai Call Girls Hiranandani Gardens Just Call 9920874524 with A/C Room ...VIP Mumbai Call Girls Hiranandani Gardens Just Call 9920874524 with A/C Room ...
VIP Mumbai Call Girls Hiranandani Gardens Just Call 9920874524 with A/C Room ...Garima Khatri
 
Premium Call Girls Cottonpet Whatsapp 7001035870 Independent Escort Service
Premium Call Girls Cottonpet Whatsapp 7001035870 Independent Escort ServicePremium Call Girls Cottonpet Whatsapp 7001035870 Independent Escort Service
Premium Call Girls Cottonpet Whatsapp 7001035870 Independent Escort Servicevidya singh
 
💎VVIP Kolkata Call Girls Parganas🩱7001035870🩱Independent Girl ( Ac Rooms Avai...
💎VVIP Kolkata Call Girls Parganas🩱7001035870🩱Independent Girl ( Ac Rooms Avai...💎VVIP Kolkata Call Girls Parganas🩱7001035870🩱Independent Girl ( Ac Rooms Avai...
💎VVIP Kolkata Call Girls Parganas🩱7001035870🩱Independent Girl ( Ac Rooms Avai...Taniya Sharma
 
Best Rate (Hyderabad) Call Girls Jahanuma ⟟ 8250192130 ⟟ High Class Call Girl...
Best Rate (Hyderabad) Call Girls Jahanuma ⟟ 8250192130 ⟟ High Class Call Girl...Best Rate (Hyderabad) Call Girls Jahanuma ⟟ 8250192130 ⟟ High Class Call Girl...
Best Rate (Hyderabad) Call Girls Jahanuma ⟟ 8250192130 ⟟ High Class Call Girl...astropune
 
Russian Call Girls in Pune Riya 9907093804 Short 1500 Night 6000 Best call gi...
Russian Call Girls in Pune Riya 9907093804 Short 1500 Night 6000 Best call gi...Russian Call Girls in Pune Riya 9907093804 Short 1500 Night 6000 Best call gi...
Russian Call Girls in Pune Riya 9907093804 Short 1500 Night 6000 Best call gi...Miss joya
 
Call Girls Yelahanka Bangalore 📲 9907093804 💞 Full Night Enjoy
Call Girls Yelahanka Bangalore 📲 9907093804 💞 Full Night EnjoyCall Girls Yelahanka Bangalore 📲 9907093804 💞 Full Night Enjoy
Call Girls Yelahanka Bangalore 📲 9907093804 💞 Full Night Enjoynarwatsonia7
 
Call Girl Coimbatore Prisha☎️ 8250192130 Independent Escort Service Coimbatore
Call Girl Coimbatore Prisha☎️  8250192130 Independent Escort Service CoimbatoreCall Girl Coimbatore Prisha☎️  8250192130 Independent Escort Service Coimbatore
Call Girl Coimbatore Prisha☎️ 8250192130 Independent Escort Service Coimbatorenarwatsonia7
 
CALL ON ➥9907093804 🔝 Call Girls Hadapsar ( Pune) Girls Service
CALL ON ➥9907093804 🔝 Call Girls Hadapsar ( Pune)  Girls ServiceCALL ON ➥9907093804 🔝 Call Girls Hadapsar ( Pune)  Girls Service
CALL ON ➥9907093804 🔝 Call Girls Hadapsar ( Pune) Girls ServiceMiss joya
 
Call Girls Service Navi Mumbai Samaira 8617697112 Independent Escort Service ...
Call Girls Service Navi Mumbai Samaira 8617697112 Independent Escort Service ...Call Girls Service Navi Mumbai Samaira 8617697112 Independent Escort Service ...
Call Girls Service Navi Mumbai Samaira 8617697112 Independent Escort Service ...Call girls in Ahmedabad High profile
 
Call Girl Number in Panvel Mumbai📲 9833363713 💞 Full Night Enjoy
Call Girl Number in Panvel Mumbai📲 9833363713 💞 Full Night EnjoyCall Girl Number in Panvel Mumbai📲 9833363713 💞 Full Night Enjoy
Call Girl Number in Panvel Mumbai📲 9833363713 💞 Full Night Enjoybabeytanya
 
Vip Call Girls Anna Salai Chennai 👉 8250192130 ❣️💯 Top Class Girls Available
Vip Call Girls Anna Salai Chennai 👉 8250192130 ❣️💯 Top Class Girls AvailableVip Call Girls Anna Salai Chennai 👉 8250192130 ❣️💯 Top Class Girls Available
Vip Call Girls Anna Salai Chennai 👉 8250192130 ❣️💯 Top Class Girls AvailableNehru place Escorts
 
Aspirin presentation slides by Dr. Rewas Ali
Aspirin presentation slides by Dr. Rewas AliAspirin presentation slides by Dr. Rewas Ali
Aspirin presentation slides by Dr. Rewas AliRewAs ALI
 
Kesar Bagh Call Girl Price 9548273370 , Lucknow Call Girls Service
Kesar Bagh Call Girl Price 9548273370 , Lucknow Call Girls ServiceKesar Bagh Call Girl Price 9548273370 , Lucknow Call Girls Service
Kesar Bagh Call Girl Price 9548273370 , Lucknow Call Girls Servicemakika9823
 
Call Girls In Andheri East Call 9920874524 Book Hot And Sexy Girls
Call Girls In Andheri East Call 9920874524 Book Hot And Sexy GirlsCall Girls In Andheri East Call 9920874524 Book Hot And Sexy Girls
Call Girls In Andheri East Call 9920874524 Book Hot And Sexy Girlsnehamumbai
 

Recently uploaded (20)

Artifacts in Nuclear Medicine with Identifying and resolving artifacts.
Artifacts in Nuclear Medicine with Identifying and resolving artifacts.Artifacts in Nuclear Medicine with Identifying and resolving artifacts.
Artifacts in Nuclear Medicine with Identifying and resolving artifacts.
 
Bangalore Call Girls Majestic 📞 9907093804 High Profile Service 100% Safe
Bangalore Call Girls Majestic 📞 9907093804 High Profile Service 100% SafeBangalore Call Girls Majestic 📞 9907093804 High Profile Service 100% Safe
Bangalore Call Girls Majestic 📞 9907093804 High Profile Service 100% Safe
 
Call Girls Service Chennai Jiya 7001305949 Independent Escort Service Chennai
Call Girls Service Chennai Jiya 7001305949 Independent Escort Service ChennaiCall Girls Service Chennai Jiya 7001305949 Independent Escort Service Chennai
Call Girls Service Chennai Jiya 7001305949 Independent Escort Service Chennai
 
Call Girls Cuttack Just Call 9907093804 Top Class Call Girl Service Available
Call Girls Cuttack Just Call 9907093804 Top Class Call Girl Service AvailableCall Girls Cuttack Just Call 9907093804 Top Class Call Girl Service Available
Call Girls Cuttack Just Call 9907093804 Top Class Call Girl Service Available
 
VIP Mumbai Call Girls Hiranandani Gardens Just Call 9920874524 with A/C Room ...
VIP Mumbai Call Girls Hiranandani Gardens Just Call 9920874524 with A/C Room ...VIP Mumbai Call Girls Hiranandani Gardens Just Call 9920874524 with A/C Room ...
VIP Mumbai Call Girls Hiranandani Gardens Just Call 9920874524 with A/C Room ...
 
Russian Call Girls in Delhi Tanvi ➡️ 9711199012 💋📞 Independent Escort Service...
Russian Call Girls in Delhi Tanvi ➡️ 9711199012 💋📞 Independent Escort Service...Russian Call Girls in Delhi Tanvi ➡️ 9711199012 💋📞 Independent Escort Service...
Russian Call Girls in Delhi Tanvi ➡️ 9711199012 💋📞 Independent Escort Service...
 
Premium Call Girls Cottonpet Whatsapp 7001035870 Independent Escort Service
Premium Call Girls Cottonpet Whatsapp 7001035870 Independent Escort ServicePremium Call Girls Cottonpet Whatsapp 7001035870 Independent Escort Service
Premium Call Girls Cottonpet Whatsapp 7001035870 Independent Escort Service
 
Escort Service Call Girls In Sarita Vihar,, 99530°56974 Delhi NCR
Escort Service Call Girls In Sarita Vihar,, 99530°56974 Delhi NCREscort Service Call Girls In Sarita Vihar,, 99530°56974 Delhi NCR
Escort Service Call Girls In Sarita Vihar,, 99530°56974 Delhi NCR
 
💎VVIP Kolkata Call Girls Parganas🩱7001035870🩱Independent Girl ( Ac Rooms Avai...
💎VVIP Kolkata Call Girls Parganas🩱7001035870🩱Independent Girl ( Ac Rooms Avai...💎VVIP Kolkata Call Girls Parganas🩱7001035870🩱Independent Girl ( Ac Rooms Avai...
💎VVIP Kolkata Call Girls Parganas🩱7001035870🩱Independent Girl ( Ac Rooms Avai...
 
Best Rate (Hyderabad) Call Girls Jahanuma ⟟ 8250192130 ⟟ High Class Call Girl...
Best Rate (Hyderabad) Call Girls Jahanuma ⟟ 8250192130 ⟟ High Class Call Girl...Best Rate (Hyderabad) Call Girls Jahanuma ⟟ 8250192130 ⟟ High Class Call Girl...
Best Rate (Hyderabad) Call Girls Jahanuma ⟟ 8250192130 ⟟ High Class Call Girl...
 
Russian Call Girls in Pune Riya 9907093804 Short 1500 Night 6000 Best call gi...
Russian Call Girls in Pune Riya 9907093804 Short 1500 Night 6000 Best call gi...Russian Call Girls in Pune Riya 9907093804 Short 1500 Night 6000 Best call gi...
Russian Call Girls in Pune Riya 9907093804 Short 1500 Night 6000 Best call gi...
 
Call Girls Yelahanka Bangalore 📲 9907093804 💞 Full Night Enjoy
Call Girls Yelahanka Bangalore 📲 9907093804 💞 Full Night EnjoyCall Girls Yelahanka Bangalore 📲 9907093804 💞 Full Night Enjoy
Call Girls Yelahanka Bangalore 📲 9907093804 💞 Full Night Enjoy
 
Call Girl Coimbatore Prisha☎️ 8250192130 Independent Escort Service Coimbatore
Call Girl Coimbatore Prisha☎️  8250192130 Independent Escort Service CoimbatoreCall Girl Coimbatore Prisha☎️  8250192130 Independent Escort Service Coimbatore
Call Girl Coimbatore Prisha☎️ 8250192130 Independent Escort Service Coimbatore
 
CALL ON ➥9907093804 🔝 Call Girls Hadapsar ( Pune) Girls Service
CALL ON ➥9907093804 🔝 Call Girls Hadapsar ( Pune)  Girls ServiceCALL ON ➥9907093804 🔝 Call Girls Hadapsar ( Pune)  Girls Service
CALL ON ➥9907093804 🔝 Call Girls Hadapsar ( Pune) Girls Service
 
Call Girls Service Navi Mumbai Samaira 8617697112 Independent Escort Service ...
Call Girls Service Navi Mumbai Samaira 8617697112 Independent Escort Service ...Call Girls Service Navi Mumbai Samaira 8617697112 Independent Escort Service ...
Call Girls Service Navi Mumbai Samaira 8617697112 Independent Escort Service ...
 
Call Girl Number in Panvel Mumbai📲 9833363713 💞 Full Night Enjoy
Call Girl Number in Panvel Mumbai📲 9833363713 💞 Full Night EnjoyCall Girl Number in Panvel Mumbai📲 9833363713 💞 Full Night Enjoy
Call Girl Number in Panvel Mumbai📲 9833363713 💞 Full Night Enjoy
 
Vip Call Girls Anna Salai Chennai 👉 8250192130 ❣️💯 Top Class Girls Available
Vip Call Girls Anna Salai Chennai 👉 8250192130 ❣️💯 Top Class Girls AvailableVip Call Girls Anna Salai Chennai 👉 8250192130 ❣️💯 Top Class Girls Available
Vip Call Girls Anna Salai Chennai 👉 8250192130 ❣️💯 Top Class Girls Available
 
Aspirin presentation slides by Dr. Rewas Ali
Aspirin presentation slides by Dr. Rewas AliAspirin presentation slides by Dr. Rewas Ali
Aspirin presentation slides by Dr. Rewas Ali
 
Kesar Bagh Call Girl Price 9548273370 , Lucknow Call Girls Service
Kesar Bagh Call Girl Price 9548273370 , Lucknow Call Girls ServiceKesar Bagh Call Girl Price 9548273370 , Lucknow Call Girls Service
Kesar Bagh Call Girl Price 9548273370 , Lucknow Call Girls Service
 
Call Girls In Andheri East Call 9920874524 Book Hot And Sexy Girls
Call Girls In Andheri East Call 9920874524 Book Hot And Sexy GirlsCall Girls In Andheri East Call 9920874524 Book Hot And Sexy Girls
Call Girls In Andheri East Call 9920874524 Book Hot And Sexy Girls
 

MedicalResearch.com Leading Medical Research Interviews August 24 2015

  • 1. MedicalResearch.com Exclusive Interviews with Medical Research and Health Care Researchers from Major and Specialty Medical Research Journals and Meetings Editor: Marie Benz, MD info@medicalresearch.com August 24 2015 For Informational Purposes Only: Not for Specific Medical Advice.
  • 2. Medical Disclaimer | Terms and Conditions • The contents of the MedicalResearch.com Site, such as text, graphics, images, and other material contained on the MedicalResearch.com Site ("Content") are for informational purposes only. The Content is not intended to be a substitute for professional medical advice, diagnosis, or treatment. Always seek the advice of your physician or other qualified health provider with any questions you may have regarding a medical condition. Never disregard professional medical advice or delay in seeking it because of something you have read on the Hemodialysis.com Site! • If you think you may have a medical emergency, call your doctor or 911 immediately. MedicalResearch.com does not recommend or endorse any specific tests, physicians, products, procedures, opinions, or other information that may be mentioned on the Site. Reliance on any information provided by MedicalResearch.com or other Eminent Domains Inc (EDI) websites, EDI employees, others appearing on the Site at the invitation of MedicalResearch.com or EDI, or other visitors to the Site is solely at your own risk. • The Site may contain health- or medical-related materials that are sexually explicit. If you find these materials offensive, you may not want to use our Site. The Site and the Content are provided on an "as is" basis. Read more interviews on MedicalResearch.com
  • 3. Hospital Improvements Have Led To Marked Decreased In Inpatient Mortality Over Ten Years Posted on August 16, 2015 MedicalResearch.com Interview with: Kevin Heslin, Ph.D., Staff Service Fellow, Center for Delivery, Organization and Markets, Agency for Healthcare Research and Quality Medical Research: What is the background for this study? Dr. Heslin: Previous trends in inpatient mortality suggest that rates have been decreasing for acute myocardial infarction (AMI), congestive heart failure (CHF), stroke, and pneumonia (Hines et al., 2010; Stepanova et al., 2013). Continued study of these trends can help researchers and policymakers assess the impact of health care quality efforts. Further, examining trends across patient and hospital subgroups may inform strategies for addressing disparities in health care quality by identifying groups that are leading and lagging in improvement. Read the rest of the interviews on MedicalResearch.com Content NOT an endorsement of efficacy and NOT intended as specific medical advice.
  • 4. Hospital Improvements Have Led To Marked Decreased In Inpatient Mortality Over Ten Years Posted on August 16, 2015 MedicalResearch.com Interview with: Kevin Heslin, Ph.D., Staff Service Fellow, Center for Delivery, Organization and Markets, Agency for Healthcare Research and Quality • Medical Research: What is the background for this study • Dr. Heslin: From 2002 to 2012, inpatient mortality decreased among patients admitted to U.S. hospitals for pneumonia (45 percent decrease, from 65.0 to 35.8 deaths per 1,000 admissions), AMI (41 percent decrease, from 94.0 to 55.9 deaths per 1,000 admissions), CHF (29 percent decrease, from 44.4 to 31.4 deaths per 1,000 admissions), and stroke (27 percent decrease, from 112.6 to 82.6 deaths per 1,000 admissions). The inpatient mortality rate for all four conditions decreased among both younger and older patients, and among men and women. Read the rest of the interviews on MedicalResearch.com Content NOT an endorsement of efficacy and NOT intended as specific medical advice.
  • 5. Hospital Improvements Have Led To Marked Decreased In Inpatient Mortality Over Ten Years Posted on August 16, 2015 MedicalResearch.com Interview with: Kevin Heslin, Ph.D., Staff Service Fellow, Center for Delivery, Organization and Markets, Agency for Healthcare Research and Quality • Medical Research: What should clinicians and patients take away from your report? • Dr. Heslin: The reductions in inpatient mortality rates reported by this brief suggest that hospitals are improving care for these conditions. This explanation is supported by other work on the quality of inpatient care for AMI, CHF, and pneumonia. Specifically, Nuti and colleagues (2015) analyzed data on more than 4,000 hospitals over the 2006-2011 period and ranked hospitals by their performance on evidence-based process of care measures, with a quality score of 100 being equal to perfect performance. Over this period, hospital performance on all measures improved. There was a decreasing difference between the quality scores of hospitals ranked at the 90th percentile and those of hospitals ranked 10th percentile. The variation between hospitals at these percentiles decreased from a 9% to 2% absolute difference for AMI, a 25% to 8% difference for congestive heart failure, and a 20% to 7% difference for pneumonia in 2006 and 2011, respectively. Read the rest of the interviews on MedicalResearch.com Content NOT an endorsement of efficacy and NOT intended as specific medical advice.
  • 6. Hospital Improvements Have Led To Marked Decreased In Inpatient Mortality Over Ten Years Posted on August 16, 2015 MedicalResearch.com Interview with: Kevin Heslin, Ph.D., Staff Service Fellow, Center for Delivery, Organization and Markets, Agency for Healthcare Research and Quality • Medical Research: What recommendations do you have for future research as a result of this study? • Dr. Heslin: Future research to better ascertain the reasons for these reductions in inpatient mortality could inform not only improvements in the clinical management of these conditions, but also the development and modification of relevant health policies. Given the dramatic decline in the number of Americans without health insurance since 2013 (Cohen and Martinez, 2015), one of the most important areas for future research is to assess the impact of the insurance coverage expansions made possible by the Affordable Care Act on hospitalization rates and outcomes for these four conditions. • References • Cohen RA, Martinez ME. Health Insurance Coverage: Early Release of Estimates from the National Health Interview Survey, January–March 2015. Centers for Disease Control and Prevention, National Center for Health Statistics, National Health Interview Survey, 2010– 2015. Read the rest of the interviews on MedicalResearch.com Content NOT an endorsement of efficacy and NOT intended as specific medical advice.
  • 7. MMR Vaccination May Not Be Reliable in HIV+ Children MedicalResearch.com Interview with: George K Siberry, MD, MPH, Medical Officer Maternal and Pediatric Infectious Disease (MPID) Branch Eunice Kennedy Shriver National Institute of Child Health and Human Development National Institutes of Health Bethesda, MD • Medical Research: What is the background for this study? • Dr. Siberry: Vaccines may not work as reliably in children with HIV infection, especially when their HIV is not under effective treatment. Today, most children in the United States who were born with HIV infection are receiving effective HIV treatment and have reached school age or even young adulthood. However, many received their childhood vaccines before they got started on their HIV treatment (because modern HIV treatments weren’t available when they were very young or their HIV infection was diagnosed late). So we wanted to see if these older children still had immunity from the vaccines they received when they were much younger. Read the rest of the interviews on MedicalResearch.com Content NOT an endorsement of efficacy and NOT intended as specific medical advice.
  • 8. MMR Vaccination May Not Be Reliable in HIV+ Children MedicalResearch.com Interview with: George K Siberry, MD, MPH, Medical Officer Maternal and Pediatric Infectious Disease (MPID) Branch Eunice Kennedy Shriver National Institute of Child Health and Human Development National Institutes of Health Bethesda, MD • Medical Research: What are the main findings? • Dr. Siberry: We looked specifically at whether older children with HIV since birth were protected against measles, mumps, and rubella, the three viral infections covered by the measles-mumps-rubella (or MMR) vaccine. We found that 1/3 up to almost 1/2 of these children were not protected against these viruses, even though nearly all of the children had received at least 2 MMR doses, as recommended. And even if their HIV was currently under excellent control. When we analyzed factors that were linked to being protected, we found that one of the most important factors was whether you got your MMR vaccine doses after you got on good treatment for your HIV infection. For instance, over 85% of children who had gotten at least 2 MMR vaccine doses after being on effective HIV treatment were protected against measles compared to less than half of those who didn’t get both of their MMR vaccine doses while on effective HIV treatment. Read the rest of the interviews on MedicalResearch.com Content NOT an endorsement of efficacy and NOT intended as specific medical advice.
  • 9. MMR Vaccination May Not Be Reliable in HIV+ Children MedicalResearch.com Interview with: George K Siberry, MD, MPH, Medical Officer Maternal and Pediatric Infectious Disease (MPID) Branch Eunice Kennedy Shriver National Institute of Child Health and Human Development National Institutes of Health Bethesda, MD • Medical Research: What should clinicians and patients take away from your report? • Dr. Siberry: For clinicians caring for people (children and young adults) who were born with HIV infection, it is important to review each patient’s medical record to determine the dates the MMR vaccines were given and the date when effective HIV treatment was started. If 2 MMR vaccines were given at least 3 months after effective HIV treatment was started, then that patient should be protected against measles, mumps and rubella. But MMR vaccine doses given prior to at least 3 months of effective HIV treatment should be repeated now (while the patient is on effective HIV treatment) to make sure the patient is protected against these viral infections. This practice is consistent with the current recommendations from the CDC for measles immunization in people born with HIV infection. Read the rest of the interviews on MedicalResearch.com Content NOT an endorsement of efficacy and NOT intended as specific medical advice.
  • 10. MMR Vaccination May Not Be Reliable in HIV+ Children MedicalResearch.com Interview with: George K Siberry, MD, MPH, Medical Officer Maternal and Pediatric Infectious Disease (MPID) Branch Eunice Kennedy Shriver National Institute of Child Health and Human Development National Institutes of Health Bethesda, MD • Medical Research: What recommendations do you have for future research as a result of this study? • Dr. Siberry: We noted that the levels of protection were much higher (e.g., 85% for measles) in HIV-infected children who got their MMR doses while on effective HIV treatment but still not as high as the >95% of healthy children protected after 2 doses of MMR. We would like to find strategies that further improve MMR vaccine response in HIV-infected children. • We are also interested in determining if this approach for determining which HIV-infected children need additional MMR vaccine doses may also apply to other childhood vaccines. • Citation: • George K. Siberry, Kunjal Patel, William Bellini, Brad Karalius, Murli Purswani, Sandra K. Burchett, William A. Meyer III, Sun Bae Sowers, Angela Ellis,and Russell B. Van Dyke For the Pediatric HIV AIDS Cohort Study (PHACS) • Immunity to Measles, Mumps and Rubella in US Children with Perinatal HIV Infection or Perinatal HIV Exposure without Infection • Clin Infect Dis. 2015 Jun 9. pii: civ440. [Epub ahead of print] Read the rest of the interviews on MedicalResearch.com Content NOT an endorsement of efficacy and NOT intended as specific medical advice.
  • 11. Biopolymers from Algae May Provide Broad Spectrum Photoprotection Susana C. M. Fernandes, PhD Researcher (Individual Marie Curie Fellowship – IEF) and Professor Vincent Bulone • MedicalResearch: What is the background for this study? What are the main findings? • Response: We have exploited unique properties of natural compounds to develop novel materials that are capable of absorbing both UV-A and UV-B radiations. The active UV- absorbing molecules are known as mycosporines and mycosporines like-amino acids and they occur in different organisms such as algae, photosynthetic bacteria (cyanobacteria) and some fish species that thrive in, e.g., the tropical waters of the Great Barrier Reef. These compounds were combined with a carbohydrate polymer found in the shells of crustaceans, the exoskeleton of insects and the cell walls of fungi. Chitosan provided a matrix on which mycosporines were attached using a simple chemical method already used for other purposes in, e.g., the pharmaceutical industry. Chitosan can typically be extracted from food waste such as the shells of shrimps. The immobilization of mycosporines on chitosan allowed the development of unique materials that have many potential applications relevant to a wide range of sectors, including cosmetics, sunscreen creams, wound dressings, plasticizers in paints and varnishes, coatings of outdoor furniture and other materials such as fabrics for shades, textiles, car dashboards, etc. In addition to being highly efficient for protection against UV-A and UV-B, the materials were shown to be photostable, thermoresistant and biocompatible. Compared to existing sunblock formulations, the materials have no detrimental effects on health and the environment. They are also fully recyclable. Read the rest of the interviews on MedicalResearch.com Content NOT an endorsement of efficacy and NOT intended as specific medical advice.
  • 12. Biopolymers from Algae May Provide Broad Spectrum Photoprotection Susana C. M. Fernandes, PhD Researcher (Individual Marie Curie Fellowship – IEF) and Professor Vincent Bulone • MedicalResearch: What should clinicians and patients take away from your report? • Response: The materials we have generated can be applicable to different biomedical areas. They are compatible with cell proliferation and adhesion with no cytotoxic or allergenic effect. They are therefore ideal for human use and can be exploited for tissue regeneration in humans. Biomedical applications comprise the engineering of artificial skin, contact lenses, artificial cornea, and so on. They can be further modified by other bioactive compounds to generate multifunctional materials in different forms, that is, films, sprays, solutions, hydrogels and 3D scaffolds. The materials we have generated are highly relevant to patients with photopathologies (e.g., albinos) in which the lack of skin pigmentation makes the organism more susceptible to sunburn and skin cancers. Read the rest of the interviews on MedicalResearch.com Content NOT an endorsement of efficacy and NOT intended as specific medical advice.
  • 13. Biopolymers from Algae May Provide Broad Spectrum Photoprotection Susana C. M. Fernandes, PhD Researcher (Individual Marie Curie Fellowship – IEF) and Professor Vincent Bulone • MedicalResearch: What recommendations do you have for future research as a result of this study? • Response: Development toward commercialization requires the actual assessment of more developed materials for their specific targeted applications. For instance, in the next phase, we will prepare physical creams, hydrogels and other forms of the materials, and evaluate the potential for commercialization by further assessing their advantages over existing materials through experiments specifically relevant to the desired applications. • The concept will be extended to the modification of other biopolymers and to the generation of multifunctional materials, thereby increasing the range of products and potential areas of application. • Citation: • Exploiting Mycosporines as Natural Molecular Sunscreens for the Fabrication of UV- Absorbing Green Materials ACS Appl. Mater. Interfaces, 2015, 7 (30), pp 16558–16564 • DOI: 10.1021/acsami.5b04064 • Susana C. M. Fernandes, Ana Alonso-Varona , Teodoro Palomares , Verónica Zubillaga , Jalel Labidi ‡, and Vincent Bulone Read the rest of the interviews on MedicalResearch.com Content NOT an endorsement of efficacy and NOT intended as specific medical advice.
  • 14. Hospital Mortality Outcomes Differ After Lung Cancer Surgery Complications MedicalResearch.com Interview with: Tyler Grenda, MD House Officer VI Section of General Surgery Department of Surgery University of Michigan • Medical Research: What is the background for this study? What are the main findings? • Dr. Grenda: The main purpose for this study was to better understand the factors underlying differences in mortality rates for hospitals performing lung cancer resection. The methodology we used included only the highest and lowest mortality hospitals (Commission on Cancer accredited cancer programs) so the sampling frame was specific. There are wide variations in mortality rates across hospitals performing lung cancer resection (overall unadjusted mortality rates were 10.8% vs. 1.6%, respectively. Read the rest of the interviews on MedicalResearch.com Content NOT an endorsement of efficacy and NOT intended as specific medical advice.
  • 15. Hospital Mortality Outcomes Differ After Lung Cancer Surgery Complications MedicalResearch.com Interview with: Tyler Grenda, MD House Officer VI Section of General Surgery Department of Surgery University of Michigan • Medical Research: What should clinicians and patients take away from your report? • Dr. Grenda: To better understand the factors that may underlie differences in mortality rates, we examined adherence to perioperative processes of care (e.g. adherence to venous thromboembolism prophylaxis) and outcomes following surgery between high mortality hospitals (HMHs) and low mortality hospitals (LMHs). While there were variations in adherence to some evidence-based perioperative processes of care, those variations did not translate to differences in directly related complications. In fact, there was no significant difference in complication rates between HMH and LMHs. However, we found that failure to rescue rates (or case fatality rates—deaths that occur after complications) explain much of the differences in mortality rates between HMHs and LMHs. Read the rest of the interviews on MedicalResearch.com Content NOT an endorsement of efficacy and NOT intended as specific medical advice.
  • 16. Hospital Mortality Outcomes Differ After Lung Cancer Surgery Complications MedicalResearch.com Interview with: Tyler Grenda, MD House Officer VI Section of General Surgery Department of Surgery University of Michigan • Medical Research: What recommendations do you have for future research as a result of this study? • Dr. Grenda: Further investigation is needed to better understand the main factors related to the development of complications with a focus on their subsequent management. We need to improve our understanding of why there are differences in rates of failure to rescue. • Citation: • Grenda TR, Revels SL, Yin H, Birkmeyer JD, Wong SL. Lung Cancer Resection at Hospitals With High vs Low Mortality Rates JAMA Surg. Published online August 12, 2015. doi:10.1001/jamasurg.2015.2199. Read the rest of the interviews on MedicalResearch.com Content NOT an endorsement of efficacy and NOT intended as specific medical advice.
  • 17. Increased Indoor Tanning Among Gay/Bisexual Men Raises Skin Cancer Risk MedicalResearch.com Interview with: Howa Yeung, MD PGY3, Emory Dermatology Emory University • Medical Research: What is the background for this study? What are the main findings? • Dr. Yeung: Indoor tanning is a well-established and preventable cause for melanomas and non-melanoma skin cancers. Public health efforts in curbing indoor tanning have focused on known high-risk populations, such as young, college-aged, White women. However, other demographic risk factors for indoor tanning remain unknown. • As our nation increasingly focuses on addressing and improving the health of lesbian, gay, bisexual, and transgender (LGBT) individuals, more and more evidence demonstrates that various LGBT subpopulations face higher rates of cancer-related behavior risk factors, such as smoking, alcohol use, obesity, etc. We wanted to find out whether risk factors for skin cancer, such as indoor tanning, disproportionately affected LGBT populations. • Our study showed higher rates of indoor tanning among gay and bisexual men, with 1.8-fold and 3.6-fold higher odds of tanning bed use within the past year, compared to straight men, after adjusting for sociodemographic factors. Disparities in frequent tanning, defined as using tanning bed 10 or more times within the past year, are even more prominent among gay and bisexual men. In contrast, no significant sexual orientation disparities were noted among women after adjusting for sociodemographic factors. Read the rest of the interviews on MedicalResearch.com Content NOT an endorsement of efficacy and NOT intended as specific medical advice.
  • 18. Increased Indoor Tanning Among Gay/Bisexual Men Raises Skin Cancer Risk MedicalResearch.com Interview with: Howa Yeung, MD PGY3, Emory Dermatology Emory University • Medical Research: What should clinicians and patients take away from your report? • Dr. Yeung: Indoor tanning among gay and bisexual men is significant and underappreciated problem, with rates approaching those among other known high-risk demographic groups. These results call for critically needed clinical a Read the rest of the interviews on MedicalResearch.com Content NOT an endorsement of efficacy and NOT intended as specific medical advice.
  • 19. Increased Indoor Tanning Among Gay/Bisexual Men Raises Skin Cancer Risk MedicalResearch.com Interview with: Howa Yeung, MD PGY3, Emory Dermatology Emory University • Medical Research: What recommendations do you have for future research as a result of this study? • Dr. Yeung: We hope our study will encourage further research on skin health among LGBT individuals. Future studies should delineate the burden of skin cancer and its modifiable risk factors among LGBT individuals. In addition, specific mediators for indoor tanning among gay and bisexual men should be examined, in order to develop targeted interventions that address the root causes of disparities. • Citation: • Yeung H, Chen SC. Sexual Orientation and Indoor Tanning Device Use: A Population-Based Study. JAMA Dermatol. Published online August 05, 2015. doi:10.1001/jamadermatol.2015.2038. Read the rest of the interviews on MedicalResearch.com Content NOT an endorsement of efficacy and NOT intended as specific medical advice.
  • 20. Several Modifiable Risk Factors for Alzheimer’s Disease Identified MedicalResearch.com Interview with: Jin-Tai Yu MD, PhD Memory and Aging Center, Department of Neurology University of California San Francisco San Francisco, CA 94158 • Medical Research: What is the background for this study? What are the main findings? • Response: The number of dementia cases in the whole world was estimated to be 35.6 million in 2010 and this number was expected to almost double every 20 years, to 65.7 million in 2030 and 115.4 million in 2050. The global prevalence of dementia was 5-7% and Alzheimer’s disease accounts for roughly 60%. This data means that we are facing an increasing number of global populations of this specific neurodegenerative disease and also the heavy burden brought by it. • Data from the website of global clinical trials (http://clinicalTrials.gov) showed that a total of 1,732 clinical trials for Alzheimer’s disease were under way. However, the previous results are not so optimistic, possibly due to the complex etiological mechanisms. In one word, we had currently no effective drugs for this disease. Figuring out how to effectively prevent its occurrence is increasingly attracting people’s attentions.Therefore, we have done the most extensive and comprehensive systematic review and meta-analysis to date, which employs a full-scale search of observational studies to calculate effect sizes and grade the evidence strength of various modifiable risk factors for this disease. We hope these results will be informative and instructive. Read the rest of the interviews on MedicalResearch.com Content NOT an endorsement of efficacy and NOT intended as specific medical advice.
  • 21. Several Modifiable Risk Factors for Alzheimer’s Disease Identified MedicalResearch.com Interview with: Jin-Tai Yu MD, PhD Memory and Aging Center, Department of Neurology University of California San Francisco San Francisco, CA 94158 We systematically searched PubMed and the Cochrane Database of Systematic Reviews from inception to July 2014, and the references of retrieved relevant articles. We included prospective cohort studies and retrospective case–control studies. 16,.906 articles were identified of which 323 with 93 factors met the inclusion criteria for meta-analysis. In addition, we assigned three grades of evidence in support of the conclusion according to two elements including the pooled sample size and heterogeneity: ‘grade I evidence’ was defined as both pooled population >5000 and lower heterogeneity (I2<50%); ‘grade II-A evidence’ was defined as pooled population >5000 but with higher heterogeneity (I2≥50%); ‘grade II-B evidence’ was defined as lower heterogeneity (I2≥50%) but with pooled population <5000; ‘grade III evidence’ was defined as both pooled population <5000 and higher heterogeneity. Read the rest of the interviews on MedicalResearch.com Content NOT an endorsement of efficacy and NOT intended as specific medical advice.
  • 22. Several Modifiable Risk Factors for Alzheimer’s Disease Identified MedicalResearch.com Interview with: Jin-Tai Yu MD, PhD Memory and Aging Center, Department of Neurology University of California San Francisco San Francisco, CA 94158 Eleven risk factors with grade I evidence included heavy smoking, low DBP, high BMI in midlife, carotid atherosclerosis, DM-2 in Asian population, low BMI, low educational attainment, high tHcy level, depression, SBP > 160mmHg and frailty; and 2 risk factors with grade II-A evidence included current smoking in Asian population and neuroticism. Our study also found that 23 protective factors for Alzheimer’s Disease (Grade I evidence: coffee/caffeine drinking, high folate intake, cognitive activity, high vitamin E intake, high vitamin C intake, current statin use, arthritis, light-to-moderate drinking, ever use of estrogens, anti-hypertensive medications, NASIDs use, high BMI in late-life, et al.; Grad II-A evidence: healthy dietary pattern, fish consumption, high education and physical activity). Given that most of the studies included in the meta-analysis are case-control studies, what is seen here is an association rather than a direct cause and effect relationship between any one factor and Alzheimer’s risk. Read the rest of the interviews on MedicalResearch.com Content NOT an endorsement of efficacy and NOT intended as specific medical advice.
  • 23. Several Modifiable Risk Factors for Alzheimer’s Disease Identified MedicalResearch.com Interview with: Jin-Tai Yu MD, PhD Memory and Aging Center, Department of Neurology University of California San Francisco San Francisco, CA 94158 • Medical Research: What should clinicians and patients take away from your report? • Response: Our study found that individuals would benefit from addressing the related potentially modifiable factors. Effective interventions in diet, medications, biochemical exposures, psychological condition, preexisting disease and lifestyle might decrease new incidence of Alzheimer’s Disease. Read the rest of the interviews on MedicalResearch.com Content NOT an endorsement of efficacy and NOT intended as specific medical advice.
  • 24. Several Modifiable Risk Factors for Alzheimer’s Disease Identified MedicalResearch.com Interview with: Jin-Tai Yu MD, PhD Memory and Aging Center, Department of Neurology University of California San Francisco San Francisco, CA 94158 • Medical Research: What recommendations do you have for future research as a result of this study? • Response: Given that, as yet, there is no cure forAlzheimer’s Disease. Our study indicates that Alzheimer’s Disease might be preventable by effectively addressing the identified modifiable factors. More prospective cohort studies are necessary for clearing the exact direct relationship between the related modifiable risk factors and Alzheimer’s risk and exploring the possibility of preventing Alzheimer’s Disease by effectively addressing the identified modifiable factors. • Citation: • J Neurol Neurosurg Psychiatry. 2015 Aug 20. pii: jnnp-2015-310548. doi: 10.1136/jnnp-2015- 310548. [Epub ahead of print] • Meta-analysis of modifiable risk factors for Alzheimer’s disease. • Xu W1, Tan L2, Wang HF3, Jiang T3, Tan MS1, Tan L4, Zhao QF1, Li JQ1, Wang J1, Yu JT5. Read the rest of the interviews on MedicalResearch.com Content NOT an endorsement of efficacy and NOT intended as specific medical advice.
  • 25. Research Into Many Skin Diseases Receives Too Little NIH Funding MedicalResearch.com Interview with: Erika Hagstrom, M.D., M.A. Preliminary Internal Medicine PGY-1 Loyola University Medical Center • Medical Research: What is the background for this study? What are the main findings? • Dr. Hagstrom: Allocation of funding dollars to research is a critical and daunting task. While many factors may impact research-funding decisions, establishing a transparent priority- setting exercise is paramount. This is particularly important for the National Institutes of Health, which invests over $30 billion for medical research each year. Diseases that have the greatest impact on our population warrant increased research dollars to reduce disease burden. The Global Burden of Disease Study (GBD) is an epidemiological effort to quantify the global burden of disease in a universal metric called disability-adjusted life years (DALYs). Focusing on our particular interest of dermatology, we investigated the 2012-2013 NIH funding for 15 skin diseases and matched this to the corresponding DALY metrics. Read the rest of the interviews on MedicalResearch.com Content NOT an endorsement of efficacy and NOT intended as specific medical advice.
  • 26. Research Into Many Skin Diseases Receives Too Little NIH Funding MedicalResearch.com Interview with: Erika Hagstrom, M.D., M.A. Preliminary Internal Medicine PGY-1 Loyola University Medical Center • Medical Research: What is the background for this study? What are the main findings? • Dr. Hagstrom: Allocation of funding dollars to research is a critical and daunting task. While many factors may impact research-funding decisions, establishing a transparent priority- setting exercise is paramount. This is particularly important for the National Institutes of Health, which invests over $30 billion for medical research each year. Diseases that have the greatest impact on our population warrant increased research dollars to reduce disease burden. The Global Burden of Disease Study (GBD) is an epidemiological effort to quantify the global burden of disease in a universal metric called disability-adjusted life years (DALYs). Focusing on our particular interest of dermatology, we investigated the 2012-2013 NIH funding for 15 skin diseases and matched this to the corresponding DALY metrics. Read the rest of the interviews on MedicalResearch.com Content NOT an endorsement of efficacy and NOT intended as specific medical advice.
  • 27. Research Into Many Skin Diseases Receives Too Little NIH Funding MedicalResearch.com Interview with: Erika Hagstrom, M.D., M.A. Preliminary Internal Medicine PGY-1 Loyola University Medical Center • Medical Research: What recommendations do you have for future research as a result of this study? • Dr. Hagstrom: We hope that our investigation demonstrates the usefulness of high-quality disease burden metrics to inform priority-setting decisions for any funding center. • Citation: • J Am Acad Dermatol. 2015 Sep;73(3):383-391.e1. doi: 10.1016/j.jaad.2015.04.039. Epub 2015 Jun 4. • Comparing cutaneous research funded by the US National Institutes of Health (NIH) with the US skin disease burden. • Hagstrom EL1, Patel S2, Karimkhani C3, Boyers LN4, Williams HC5, Hay RJ6, Weinstock MA7, Armstrong AW8, Dunnick CA9, Margolis DJ10, Dellavalle RP11. Read the rest of the interviews on MedicalResearch.com Content NOT an endorsement of efficacy and NOT intended as specific medical advice.
  • 28. Spinach Bleach Contours May Prevent Adequate Decontamination MedicalResearch.com Interview with: Nichola Kinsinger, Ph.D Postdoctoral Researcher, Chemical & Environmental Engineering University of California, Riverside USDA National Institute for Food and Agricultural Postdoctoral Fellow DoD Office of Naval • Medical Research: What is the background for this study? What are the main findings? • Dr. Kinsinger: Outbreaks observed in produce are becoming increasing common possibly due to contaminated irrigation waters or contaminated waters used during processing. In 2006 California had spinach-borne E. coli outbreak that impacted 26 states and 200 confirmed sickened. Leafy greens account for 20% of the outbreaks alone and are of increased concern since they are frequently consumed raw. These outbreaks drew our attention over the past few years and we started applying methods originally developed for studying bacterial adhesion on engineered surfaces to the issue of food safety. Although food safety is a new area of study for our lab, the project is based upon the concept of pathogen adhesion transport which has been the focus of my advisor’s lab for many years previously. Rather than the previous scenario looking at pathogen interaction with engineered or mineral surfaces, we are looking at a spinach leaf instead. Read the rest of the interviews on MedicalResearch.com Content NOT an endorsement of efficacy and NOT intended as specific medical advice.
  • 29. Spinach Bleach Contours May Prevent Adequate Decontamination MedicalResearch.com Interview with: Nichola Kinsinger, Ph.D Postdoctoral Researcher, Chemical & Environmental Engineering University of California, Riverside USDA National Institute for Food and Agricultural Postdoctoral Fellow DoD Office of Naval We are using a parallel plate flow chamber system developed by Professor Sharon Walker to evaluate the real time attachment and detachment of the pathogens to the spinach epicuticle layer (surface layer of the leaf) in realistic water chemistries and flow conditions. Subsequently we can evaluate the efficacy of the rinsing process to kill the bacteria that may remain on the leaf. Initially we were finding that at low concentrations of bleach, bacteria will detach from the leaf surface allowing for potential cross-contamination later in the process. However above 500ppb we observed 100% of the attached bacterial cells are killed. Read the rest of the interviews on MedicalResearch.com Content NOT an endorsement of efficacy and NOT intended as specific medical advice.
  • 30. Spinach Bleach Contours May Prevent Adequate Decontamination MedicalResearch.com Interview with: Nichola Kinsinger, Ph.D Postdoctoral Researcher, Chemical & Environmental Engineering University of California, Riverside USDA National Institute for Food and Agricultural Postdoctoral Fellow DoD Office of Naval So how in commercial rinsing operations that use bleach concentration ranging from 50-200ppm result in outbreaks? Through this study we analyzed the topography of the leaf and modeled the concentration gradient across the surface of a leaf based on commercial rinsing practices. We found that even at high bleach concentrations within rinse water result in low bleach concentrations at the leaf surface on the order of 5-1000 ppb bleach, which in our study has shown to that the bacteria can survive when attached to the leaf and/or detach causing concern for cross-contamination. In this case the very disinfection processes intended to clean, remove, and prevent contamination was found to be a potential pathway to amplifying foodborne outbreaks. Read the rest of the interviews on MedicalResearch.com Content NOT an endorsement of efficacy and NOT intended as specific medical advice.
  • 31. Spinach Bleach Contours May Prevent Adequate Decontamination MedicalResearch.com Interview with: Nichola Kinsinger, Ph.D Postdoctoral Researcher, Chemical & Environmental Engineering University of California, Riverside USDA National Institute for Food and Agricultural Postdoctoral Fellow DoD Office of Naval • Medical Research: What recommendations do you have for future research as a result of this study? • Dr. Kinsinger: The work presented at the conference is part of an ongoing effort in my advisor’s lab. The work will continue to look at a broader range of foods, engineered surfaces, and pathogen types. • Citation: • 250th American Chemical Society National Meeting & Exposition discussing: • “Is our salad safe? Efficacy of disinfection techniques to decontaminate spinach leaves and reduce cross-contamination” Read the rest of the interviews on MedicalResearch.com Content NOT an endorsement of efficacy and NOT intended as specific medical advice.
  • 32. Early Life Intelligence Linked To Better Physical Fitness in Middle Age MedicalResearch.com Interview with: Rikke Hodal Meincke PhD student Center for Healthy Aging and the Department of Public Health University of Copenhagen • Medical Research: What is the background for this study? • Response: A sufficient level of physical capability is a precondition for maintaining independence and quality of life. Physical capability can be assessed objectively by tests of physical performance, for instance handgrip strength, chair-rising and postural balance. Physical performance is associated with mortality and disability in late life, so gaining insights into the variance in physical performance is important to promote sustained physical capability and prevent disability. Research has previously found physical activity, health status and socioeconomic position to be associated with physical performance. In addition, early life factors, such as childhood SEP, have been found to be associated with measures of physical performance later in life. The objective of our study was to examine the association between intelligence in early adulthood and midlife physical performance in Danish men. Read the rest of the interviews on MedicalResearch.com Content NOT an endorsement of efficacy and NOT intended as specific medical advice.
  • 33. Early Life Intelligence Linked To Better Physical Fitness in Middle Age MedicalResearch.com Interview with: Rikke Hodal Meincke PhD student Center for Healthy Aging and the Department of Public Health University of Copenhagen If an association between intelligence in early life and midlife physical performance exists it may indicate that cognitive abilities and physical performance share some of the same neurodevelopmental processes, but may also indicate that intelligence has an independent effect on later physical performance through various pathways. Read the rest of the interviews on MedicalResearch.com Content NOT an endorsement of efficacy and NOT intended as specific medical advice.
  • 34. Early Life Intelligence Linked To Better Physical Fitness in Middle Age MedicalResearch.com Interview with: Rikke Hodal Meincke PhD student Center for Healthy Aging and the Department of Public Health University of Copenhagen • Medical Research: What are the main findings? • Response: In our study of more than 2800 Danish men, we found positive associations between intelligence in early adulthood and five objective measures of physical performance in midlife independent of other early life factors. A one standard deviation increase in intelligence score resulted in 1.1 more chair-rises in 30 seconds, a 1 cm higher jump, a 3.7% smaller balance area, a 0.7 kg increase in handgrip-strength, and a 0.5 kg increase in lower back force. Read the rest of the interviews on MedicalResearch.com Content NOT an endorsement of efficacy and NOT intended as specific medical advice.
  • 35. Early Life Intelligence Linked To Better Physical Fitness in Middle Age MedicalResearch.com Interview with: Rikke Hodal Meincke PhD student Center for Healthy Aging and the Department of Public Health University of Copenhagen • Medical Research: What should clinicians and patients take away from your report? • Response: This knowledge can be used in the development of interventions which should be designed to address people with different cognitive abilities. Furthermore, policy-makers and scientists should bear in mind that the prevention of mobility limitations in midlife might need to start early in life. Read the rest of the interviews on MedicalResearch.com Content NOT an endorsement of efficacy and NOT intended as specific medical advice.
  • 36. Early Life Intelligence Linked To Better Physical Fitness in Middle Age MedicalResearch.com Interview with: Rikke Hodal Meincke PhD student Center for Healthy Aging and the Department of Public Health University of Copenhagen • Medical Research: What recommendations do you have for future research as a result of this study? • Response: Future research should look into the mechanisms behind the association between intelligence and physical performance, for instance if physical activity or other lifestyle factors mediate the relationship. When the mechanisms are better understood, interventions aiming to prevent mobility limitations should incorporate this knowledge. • Citation: • H. Meincke, M. Osler, E. L. Mortensen, A. M. Hansen. Is Intelligence in Early Adulthood Associated With Midlife Physical Performance Among Danish Males?Journal of Aging and Health, 2015; DOI:10.1177/0898264315594139 Read the rest of the interviews on MedicalResearch.com Content NOT an endorsement of efficacy and NOT intended as specific medical advice.
  • 37. Extent of Liver Damage May Be Underestimated In Hepatitis C Patients MedicalResearch.com Interview with: Stuart Gordon, M.D. Director of Hepatology at Henry Ford Hospital Detroit, Michigan • Medical Research: What is the background for this study? What are the main findings? • Dr. Gordon: The U.S. Centers for Disease Control and Prevention’s Division of Viral Hepatitis estimates 2.7 to 3.9 million people in the United States currently suffer from chronic hepatitis C. But, unfortunately, many of these patients may be unaware of the severity of their liver damage. We looked at evidence of cirrhosis among hepatitis C patients by examining four different parameters: ICD9 codes; liver biopsy reports; evidence of liver failure; and the FIB-4 test, an easily calculated biomarker. By using all four indicators of cirrhosis, we found a far higher prevalence of cirrhosis than would be indicated by any one method. Read the rest of the interviews on MedicalResearch.com Content NOT an endorsement of efficacy and NOT intended as specific medical advice.
  • 38. Extent of Liver Damage May Be Underestimated In Hepatitis C Patients MedicalResearch.com Interview with: Stuart Gordon, M.D. Director of Hepatology at Henry Ford Hospital Detroit, Michigan • Medical Research: What should clinicians and patients take away from your report? • Dr. Gordon: Presence of cirrhosis in hepatitis C may be hidden or not easily recognizable; is often asymptomatic in its early form; and may require a high degree of suspicion in order to diagnose. Read the rest of the interviews on MedicalResearch.com Content NOT an endorsement of efficacy and NOT intended as specific medical advice.
  • 39. Extent of Liver Damage May Be Underestimated In Hepatitis C Patients MedicalResearch.com Interview with: Stuart Gordon, M.D. Director of Hepatology at Henry Ford Hospital Detroit, Michigan • Medical Research: What recommendations do you have for future research as a result of this study? • Dr. Gordon: Studies that rely on diagnostic codes or biopsies performed in routine care for evidence of cirrhosis may be underestimating the extent of cirrhosis, which is a leading cause of death in the United States. Future research should investigate the value of indirect markers of cirrhosis. This could result in earlier diagnosis and prevention of liver disease complications. • Citation: • Stuart C Gordon, Lois E Lamerato, Loralee B Rupp, Scott D Holmberg, Anne C Moorman, Philip R Spradling, Eyasu Teshale, Fujie Xu, Joseph A Boscarino, Vinutha Vijayadeva, Mark A Schmidt, Nancy Oja-Tebbe, Mei Lu. Prevalence of Cirrhosis in Hepatitis C Patients in the Chronic Hepatitis Cohort Study (CHeCS): A Retrospective and Prospective Observational Study. The American Journal of Gastroenterology, 2015; 110 (8): 1169 DOI: 10.1038/ajg.2015.203 Read the rest of the interviews on MedicalResearch.com Content NOT an endorsement of efficacy and NOT intended as specific medical advice.
  • 40. Protein Irisin Linked to Muscle Wasting and Atherosclerosis in Dialysis Patients MedicalResearch.com Interview with: Tae-Hyun Yoo MD PhD Department of Internal Medicine, College of Medicine Severance Biomedical Science Institute, Brain Korea 21 PLUS Yonsei University, Seoul, Korea • Medical Research: What is the background for this study? What are the main findings? • Dr. Tae-Hyun Yoo: Sarcopenia, reduction in muscle mass, is frequently observed in PEW and is prevalent in chronic kidney disease (CKD) patients. In ESRD patients, sarcopenia is significantly associated with greater mortality. Skeletal muscles produce and release myokines, which suggested to mediate their protective effects. Irisin, a novel myokine, has been introduced to drive brown-fat-like conversion of white adipose tissue and has beneficial effects of skeletal muscle on energy homeostasis and glucose metabolisms. Therefore, we hypothesized that irisin had significant association with sarcopenia and cardiovascular disease in dialysis patients. In peritoneal dialysis patients, serum irisin was positively correlated with mid-arm muscle circumference and thigh circumference. In addition, serum irisin was a significant independent predictor for carotid atherosclerosis even after adjustment for high-sensitivity C-reactive protein in these patients. This study demonstrated that serum irisin was significantly associated with sarcopenia and carotid atherosclerosis in peritoneal dialysis patients. Read the rest of the interviews on MedicalResearch.com Content NOT an endorsement of efficacy and NOT intended as specific medical advice.
  • 41. Protein Irisin Linked to Muscle Wasting and Atherosclerosis in Dialysis Patients MedicalResearch.com Interview with: Tae-Hyun Yoo MD PhD Department of Internal Medicine, College of Medicine Severance Biomedical Science Institute, Brain Korea 21 PLUS Yonsei University, Seoul, Korea • Medical Research: What should clinicians and patients take away from your report? • Dr. Tae-Hyun Yoo: Malnutrition and chronic inflammation have been proposed to explain the significant association between sarcopenia and cardiovascular disease in (end stage renal disease) ESRD population. Lower muscle mass might reflect poor nutritional status and higher degrees of unopposed inflammation. In addition to these factors, a myokine could be a contributor. In this study, irisin as a myokine could be helpful for stratifying risk of sarcopenia and carotid atherosclerosis in peritoneal dialysis patients. In addition, present study suggested muscle is an another important organ for protection and modulating cardiovascular diseases in dialysis population. Read the rest of the interviews on MedicalResearch.com Content NOT an endorsement of efficacy and NOT intended as specific medical advice.
  • 42. Protein Irisin Linked to Muscle Wasting and Atherosclerosis in Dialysis Patients MedicalResearch.com Interview with: Tae-Hyun Yoo MD PhD Department of Internal Medicine, College of Medicine Severance Biomedical Science Institute, Brain Korea 21 PLUS Yonsei University, Seoul, Korea • Medical Research: What recommendations do you have for future research as a result of this study? • Dr. Tae-Hyun Yoo: A study with follow-up carotid ultrasound and measurement of anthropometric indices and serum irisin is necessary to clarify the association of irisin, sarcopenia, and cardiovascular disease. Therefore, the impact of irisin on future cardiovascular outcomes should be clarified in the future study. The effect of the intervention using exercise training on irisin, sarcopenia, and cardiovascular disease might be also worth investigating in dialysis patients. • Citation: • Irisin, a novel myokine is an independent predictor for sarcopenia and carotid atherosclerosis in dialysis patients • Lee, Mi Jung et al. Atherosclerosis • Received: February 20, 2015; Received in revised form: July 7, 2015; Accepted: August 6, 2015; Published Online: August 13, 2015 • Publication stage: In Press Accepted Manuscript • DOI: http://dx.doi.org/10.1016/j.atherosclerosis.2015.08.002 Read the rest of the interviews on MedicalResearch.com Content NOT an endorsement of efficacy and NOT intended as specific medical advice.
  • 43. Factors Identified For Lasting Total Hip Replacement MedicalResearch.com Interview with: Sandrine Colas, MSc, MPH Department of Epidemiology of Health Products French National Agency for Medicines and Health Products Safety (ANSM) Saint-Denis, France • Medical Research: What is the background for this study? What are the main findings? • Response: Total hip replacement (THR) is to replace a damaged coxofemoral joint with a prosthetic implant. Primary or secondary degenerative osteoarthritis of the hip joint is the main indication for THR (other indications are essentially trauma, which is more common in the elderly and mostly affects women over 80 years of age). The number of THR has increased in all industrialized countries, particularly on account of the ageng population.Total hip replacement is one of the most common and successful surgical procedures in modern practice. Although results are generally good, revision (consisting in changing one or all components of the implant) is sometimes necessary (about 1% per year). Prosthetic revision is a longer and more complex operation than primary implantation and it has a higher incidence of post-surgical complications. Read the rest of the interviews on MedicalResearch.com Content NOT an endorsement of efficacy and NOT intended as specific medical advice.
  • 44. Factors Identified For Lasting Total Hip Replacement MedicalResearch.com Interview with: Sandrine Colas, MSc, MPH Department of Epidemiology of Health Products French National Agency for Medicines and Health Products Safety (ANSM) Saint-Denis, France • Several prosthetic revision risk factors have been highlighted recently in published studies, but results relating to prosthetic and/or patient characteristics and total hip replacement survivorship tend to vary. The existence of an association between the fixation technique and/or bearing surface and prosthetic survivorship has yet to be established. • The main aim of our work was therefore to compare total hip replacement short-term survivorship according to cement type and bearing surface, in a large population of subjects who have undergone total hip replacement for reasons other than trauma (25%) and bone tumor (<0.1%), taking prosthetic revision risk factors (age, gender, comorbidities, concomitant medication, implanting center, etc.) into account. • Total hip replacement characteristics are related to early implant survivorship. After 33 months of follow-up, antibiotic-impregnated cemented THRs have a better prognosis. MoM total hip replacemenst have a slightly worse prognosis. Read the rest of the interviews on MedicalResearch.com Content NOT an endorsement of efficacy and NOT intended as specific medical advice.
  • 45. Factors Identified For Lasting Total Hip Replacement MedicalResearch.com Interview with: Sandrine Colas, MSc, MPH Department of Epidemiology of Health Products French National Agency for Medicines and Health Products Safety (ANSM) Saint-Denis, France • Medical Research: What should clinicians and patients take away from your report? • Response: These findings are useful in helping surgeons select a total hip replacement fixation technique and bearing surface, and helpful for both patient and surgeon in the decision-making process. • Citation: • Colas S, Collin C, Piriou P, Zureik M. Association Between Total Hip Replacement Characteristics and 3-Year Prosthetic Survivorship : A Population-Based Study.JAMA Surg. Published online August 19, 2015. doi:10.1001/jamasurg.2015.1325. Read the rest of the interviews on MedicalResearch.com Content NOT an endorsement of efficacy and NOT intended as specific medical advice.
  • 46. Noninvasive FFR Study Superior to CT Angiography In Patients With Highly Calcified Coronary Disease MedicalResearch.com Interview with: Bjarne Linde Norgaard, MD PhD Department of Cardiology, Aarhus University Hospital Skejby Aarhus, Denmark • Medical Research: What is the background for this study? What are the main findings? • Dr. Norgaard: Noninvasive fractional flow reserve derived from standard acquired coronary CT angiography (CTA) (FFRct) in patients with suspected coronary artery disease exhibits high and superior diagnostic performance when compared to coronary CTA alone in identifying lesion-specific ischemia (which is the established metric for decision-making on coronary revascularization). As the presence of coronary calcification may compromise the diagnostic accuracy and specificity of coronary CTA, this study was performed in order to investigate the influence of calcification on the diagnostic performance of FFRct in patients (214) and vessels (333) with suspected coronary artery disease. The main finding in this study was that FFRct provides high and superior diagnostic performance and discrimination of ischemia compared with coronary CTA interpretation alone in patients and vessels with high levels of calcification. Read the rest of the interviews on MedicalResearch.com Content NOT an endorsement of efficacy and NOT intended as specific medical advice.
  • 47. Noninvasive FFR Study Superior to CT Angiography In Patients With Highly Calcified Coronary Disease MedicalResearch.com Interview with: Bjarne Linde Norgaard, MD PhD Department of Cardiology, Aarhus University Hospital Skejby Aarhus, Denmark • Medical Research: What should clinicians and patients take away from your report? • Dr. Norgaard: In the event of good CT image quality, which can be established in most patients by careful attention to pre-acquisition heart-rate control and use of nitroglycerin, noninvasive FFRct provides high diagnostic performance in identifying lesion-specific ischemia in patients and vessels over a wide range of coronary calcification. Read the rest of the interviews on MedicalResearch.com Content NOT an endorsement of efficacy and NOT intended as specific medical advice.
  • 48. Novel DNA Vaccine Provides Protection Against MERS Virus MedicalResearch.com Interview with: David B. Weiner, Ph.D. Professor, Department of Pathology and Laboratory Medicine Chair, Gene Therapy and Vaccine Program, CAMB Co-Leader Tumor Virology Program, Abramson Cancer Program University of Pennsylvania, Perelman School of Medicine • Medical Research: What is the background for this study? What are the main findings? • Dr. Weiner: MERS, like the Severe Acute Respiratory Syndrome (SARS), is characterized by high fever and severe cough from pneumonia. MERS is caused by an emerging human coronavirus, which is distinct from the SARS coronavirus. Since its identification in 2012, MERS has been linked to over 1,300 infections and close to 400 deaths. It has occurred in the Arabian Peninsula, Europe, and in the US and in Asia. It can be spread in a hospital setting. • Scientists now report that a novel synthetic DNA vaccine can, for the first time, induce protective immunity against the Middle EastRespiratory Syndrome (MERS) coronavirus in animal species. Researchers from the Perelman School of Medicine at the University of Pennsylvania. The NIH, the Public Health agency of Canada, and from a leading company in the development of synthetic DNA vaccine technology, Inovio described the results in a paper published their work in Science Translational Medicine (STM) this week. The experimental, preventive vaccine, given six weeks before exposure to the MERS virus, fully protects rhesus macaques from disease. The vaccine also generated potentially protective antibodies in blood drawn from camels, the purported source of MERS transmission in the Middle East. Read the rest of the interviews on MedicalResearch.com Content NOT an endorsement of efficacy and NOT intended as specific medical advice.
  • 49. Novel DNA Vaccine Provides Protection Against MERS Virus MedicalResearch.com Interview with: David B. Weiner, Ph.D. Professor, Department of Pathology and Laboratory Medicine Chair, Gene Therapy and Vaccine Program, CAMB Co-Leader Tumor Virology Program, Abramson Cancer Program University of Pennsylvania, Perelman School of Medicine • Medical Research: What should clinicians and patients take away from your report? • Response: Dr David Weiner, of the Perelman School of Medicine, senior author of the study described “The significant recent increase in MERS cases, coupled with the lack of effective antiviral therapies or vaccines to treat or prevent this infection, have raised significant concern,” Weiner said. “Accordingly the development of a vaccine for MERS remains a high priority. and the results reported here in this regard are encouraging.” • The STM study describes a vaccine to prevent disease and control virus replication in an important primate model of MERS pathogenesis, which was developed by the National Institutes of Health (NIH). The vaccine was able to prevent MERS disease and offered benefit to 100 percent of the animals in this study. In addition, the vaccine induced antibodies that are linked with protection in camels, a species that is thought to be a major source of transmission to humans from the Middle East, showing that this vaccine could be deployed to break this this link in the MERS transmission cycle. In the field, say the researchers, this vaccine could decrease person-to-person spread of infection in the event of an outbreak and help to protect health care workers or exposed individuals. • Dr. Muthumani first author of the study described “This simple synthetic vaccine has the potential to overcome important production and deployment limitations, and what’s more, the vaccine is non-live, so does not pose a risk of spreading to unintended individuals.” Read the rest of the interviews on MedicalResearch.com Content NOT an endorsement of efficacy and NOT intended as specific medical advice.
  • 50. Novel DNA Vaccine Provides Protection Against MERS Virus MedicalResearch.com Interview with: David B. Weiner, Ph.D. Professor, Department of Pathology and Laboratory Medicine Chair, Gene Therapy and Vaccine Program, CAMB Co-Leader Tumor Virology Program, Abramson Cancer Program University of Pennsylvania, Perelman School of Medicine • Medical Research: What recommendations do you have for future research as a result of this study? • Dr. Weiner: Most importantly the corportate collaborator for this study, Inovio, will move this program into human testing. Under a previously announced collaboration agreement with GeneOne Life Science Inc. (KSE: 011000), Inovio’s MERS vaccine product will advance into a clinical trial in healthy volunteers before the year end. In addition studies of the use of such a vaccine in a formulation specific for Camels, the animal species associated with transmission to humans, is continuing. • Citation: • Muthumani, D. Falzarano, E. L. Reuschel, C. Tingey, S. Flingai, D. O. Villarreal, M. Wise, A. Patel, A. Izmirly, A. Aljuaid, A. M. Seliga, G. Soule, M. Morrow, K. A. Kraynyak, A. S. Khan, D. P. Scott, F. Feldmann, R. LaCasse, K. Meade-White, A. Okumura, K. E. Ugen, N. Y. Sardesai, J. J. Kim, G. Kobinger, H. Feldmann, D. B. Weiner. A synthetic consensus anti-spike protein DNA vaccine induces protective immunity against Middle East respiratory syndrome coronavirus in nonhuman primates. Science Translational Medicine, 2015; 7 (301): 301ra132 DOI:10.1126/scitranslmed.aac7462 Read the rest of the interviews on MedicalResearch.com Content NOT an endorsement of efficacy and NOT intended as specific medical advice.
  • 51. Single Vaccine Can Protect Against Six Common Childhood Diseases MedicalResearch.com Interview with: Gary S. Marshall, M.D. Professor of Pediatrics Chief, Division of Pediatric Infectious Diseases Director, Pediatric Clinical Trials Unit University of Louisville School of Medicine • Medical Research: What is the background for this study? What are the main findings? • Dr. Marshall: The infant immunization schedule has become crowded. That’s great news, in a sense, because it means that our children have become better protected against more diseases. At the same time, this has led to well child visits during which many shots are recommended, and some parents want to limit the number of injections their children receive at one time. This leads to deferrals, poor timeliness and decreased coverage rates, all of which could impair protection. This study shows that a hexavalent vaccine—one that combines diphtheria, tetanus, pertussis, polio, Haemophilus influenzae type b, and hepatitis B vaccines in one syringe—is safe and just as immunogenic as the currently used component vaccines. Read the rest of the interviews on MedicalResearch.com Content NOT an endorsement of efficacy and NOT intended as specific medical advice.
  • 52. Single Vaccine Can Protect Against Six Common Childhood Diseases MedicalResearch.com Interview with: Gary S. Marshall, M.D. Professor of Pediatrics Chief, Division of Pediatric Infectious Diseases Director, Pediatric Clinical Trials Unit University of Louisville School of Medicine • Medical Research: What should clinicians and patients take away from your report? • Dr. Marshall: Pending FDA approval, pediatric practices should have a new option for infant immunization, one that will allow for fewer shots without compromising protection. • Medical Research: What recommendations do you have for future research as a result of this study? • Dr. Marshall: The more vaccine antigens that can be combined into single injections the better, as long as safety and protection are preserved. The performance of higher-valency combination vaccines cannot be predicted a priori, so the formulations have to be prepared and the clinical studies have to be done to assess reactogenicity, safety and immunogenicity. • Citation: • Immunogenicity, Safety, and Tolerability of a Hexavalent Vaccine in Infants • Gary S. Marshall, Gregory L. Adams, Michael L. Leonardi, Maria Petrecz,Sheryl A. Flores, Angela L. Ngai, Jin Xu, Guanghan Liu, Jon E. Stek, Ginamarie Foglia, and Andrew W. Lee • Pediatrics peds.2014-4102; published ahead of print July 27, 2015,doi:10.1542/peds.2014- 4102 Read the rest of the interviews on MedicalResearch.com Content NOT an endorsement of efficacy and NOT intended as specific medical advice.
  • 53. How Do Female Cells Inactivate One X Chromosome? MedicalResearch.com Interview with: Hendrik Marks Ph.D Group leader Epigenetics of Stem Cells Radboud University, Department of Molecular Biology, RIMLS Nijmegen, The Netherlands • Medical Research: What is the background for this study? What are the main findings? • Dr. Marks: In mammals, sex is determined by two so-called “sex” chromosome: males have a single X chromosome as well as a Y chromosome, whereas females have two copies of the X chromosome. However, if both X chromosomes were to be active in female cells, these cells would have a double dosis of X-chromosomal gene products as compared to male cells. As this is lethal for almost all cells, female cells shut off one X chromosome in every cell in a process called X inactivation. This process occurs during early embryonic development. Read the rest of the interviews on MedicalResearch.com Content NOT an endorsement of efficacy and NOT intended as specific medical advice.
  • 54. How Do Female Cells Inactivate One X Chromosome? MedicalResearch.com Interview with: Hendrik Marks Ph.D Group leader Epigenetics of Stem Cells Radboud University, Department of Molecular Biology, RIMLS Nijmegen, The Netherlands A lot is known about how this process is turned on, but is was unclear how such a silencing process spreads along a full chromosome. In order to further study this, we used female mouse embryonic stem cells (mESCs) as a model system and initiated X inactivation by means of differentiation. With the latest technologies, we were able to keep the two X chromosomes apart and measure one of them – with its 166 million base pairs (Mbs) – in detail. Every day we checked which parts of the chromosome had been switched off. The whole process took about eight days, and the inactivation spreads out from the centre of the X chromosome towards the ends. That doesn’t happen gradually but moves jumpwise from domain to domain. Domains are long pieces of DNA (of around 1Mb) that cluster together in knots. As it seems that X inactivation jumps from domain to domain, we now know that these domains are co-regulated. Also, we collected strong evidence that the same process is occurring in human. Read the rest of the interviews on MedicalResearch.com Content NOT an endorsement of efficacy and NOT intended as specific medical advice.
  • 55. How Do Female Cells Inactivate One X Chromosome? MedicalResearch.com Interview with: Hendrik Marks Ph.D Group leader Epigenetics of Stem Cells Radboud University, Department of Molecular Biology, RIMLS Nijmegen, The Netherlands • Medical Research: What should clinicians and patients take away from your report? • Dr. Marks: Our study is very fundamental in nature. It shows how a full chromosome gets silenced, and it also points towards a very important role of the recently-discovered domains it this process. However, It is not unlikely that some diseases that are linked to the X chromosome in female, the so-called X linked diseases, are due to improper spreading across domains. Thus, in some of these diseases it might turn out very helpful investigating the domain structure of the X chromosomes, and/or SNPs that present at the borders of the domains. Read the rest of the interviews on MedicalResearch.com Content NOT an endorsement of efficacy and NOT intended as specific medical advice.
  • 56. How Do Female Cells Inactivate One X Chromosome? MedicalResearch.com Interview with: Hendrik Marks Ph.D Group leader Epigenetics of Stem Cells Radboud University, Department of Molecular Biology, RIMLS Nijmegen, The Netherlands • Medical Research: What recommendations do you have for future research as a result of this study? • Dr. Marks: After one of the X chromosomes has been inactivated, it will stay inactive forever. It would be very interesting to discover why sometimes the one while in other cases the other X chromosome is inactivated during development. That could help in treating X-linked diseases – like Rett syndrome and fragile X syndrome in females. Reactivating (part of) the ‘right’ X chromosome could be a potential treatment for these diseases. So one of the next steps that the field is aiming at is to figure out how to do that, and our current studies provide new clues for potential strategies. • Citation: • ‘Dynamics of gene silencing during X inactivation using allele-specific RNA-seq’ • Hendrik Marks, Hindrik H. D. Kerstens, Tahsin Stefan Barakat, Erik Splinter, René A. M. Dirks, Guido van Mierlo, Onkar Joshi, Shuang-Yin Wang, Tomas Babak, Cornelis A. Albers, Tüzer Kalkan, Austin Smith, Alice Jouneau, Wouter de Laat, Joost Gribnau and Hendrik G. Stunnenberg • Genome Biology; doi:10.1186/s13059-015-0698-x Read the rest of the interviews on MedicalResearch.com Content NOT an endorsement of efficacy and NOT intended as specific medical advice.
  • 57. Florida’s Drug and Pill Mills Law Reduced Opioids Prescriptions MedicalResearch.com Interview with: Lainie Rutkow, JD, PhD, MPH Associate Professor Department of Health Policy and Management Johns Hopkins Bloomberg School of Public Health • Medical Research: What is the background for this study? What are the main findings? • Dr. Rutkow: Rates of prescription drug diversion and misuse, as well as overdose deaths, have increased throughout the United States. CDC estimates that each day, 44 people die from a prescription drug overdose. In the mid-2000s, Florida was viewed as the epicenter of this epidemic, with prescription drug overdose deaths increasing more than 80% from 2003 to 2009. In response, Florida enacted several laws to mitigate prescription drug abuse and diversion. Its pill mill law required pain management clinics to register with the state and prohibited physician dispensing of certain drugs. Florida’s Prescription Drug Monitoring Program (PDMP) collects data about dispensing of prescription drugs and can be accessed by physicians and pharmacists. Little is known about how these laws have affected prescribing of opioids. Read the rest of the interviews on MedicalResearch.com Content NOT an endorsement of efficacy and NOT intended as specific medical advice.
  • 58. Florida’s Drug and Pill Mills Law Reduced Opioids Prescriptions MedicalResearch.com Interview with: Lainie Rutkow, JD, PhD, MPH Associate Professor Department of Health Policy and Management Johns Hopkins Bloomberg School of Public Health We applied comparative interrupted time series analyses to pharmacy claims data to examine four outcomes related to opioid prescribing in Florida, with Georgia as a comparison state. We found that in the first year of implementation, Florida’s Prescription Drug Monitoring Program and pill mill law were associated with modest reductions in prescription opioid volume, prescriptions written, and the dose per prescription. These declines were statistically significant among the highest volume prescribers and patients at baseline. Read the rest of the interviews on MedicalResearch.com Content NOT an endorsement of efficacy and NOT intended as specific medical advice.
  • 59. Florida’s Drug and Pill Mills Law Reduced Opioids Prescriptions MedicalResearch.com Interview with: Lainie Rutkow, JD, PhD, MPH Associate Professor Department of Health Policy and Management Johns Hopkins Bloomberg School of Public Health • Medical Research: What should clinicians and patients take away from your report? • Dr. Rutkow: Our findings suggest that policy responses to the prescription drug epidemic – such as Prescription Drug Monitoring Program and pill mill laws – are viable options. Most prescribers are supportive of policies that limit access to opioids. Given this support, along with our findings, other states may want to consider implementing both a PDMP and a pill mill law. Read the rest of the interviews on MedicalResearch.com Content NOT an endorsement of efficacy and NOT intended as specific medical advice.
  • 60. Florida’s Drug and Pill Mills Law Reduced Opioids Prescriptions MedicalResearch.com Interview with: Lainie Rutkow, JD, PhD, MPH Associate Professor Department of Health Policy and Management Johns Hopkins Bloomberg School of Public Health • Medical Research: What recommendations do you have for future research as a result of this study? • Dr. Rutkow: While our study looked at the first year of implementation for these laws, future research should consider their longer-term effects. • Citation: • Rutkow L, Chang H, Daubresse M, Webster DW, Stuart EA, Alexander G. Effect of Florida’s Prescription Drug Monitoring Program and Pill Mill Laws on Opioid Prescribing and Use. JAMA Intern Med.Published online August 17, 2015. doi:10.1001/jamainternmed.2015.3931. Read the rest of the interviews on MedicalResearch.com Content NOT an endorsement of efficacy and NOT intended as specific medical advice.
  • 61. Polygenetic Score Predicts Educational Attainment MedicalResearch.com Interview with: Ben Domingue Assistant Professor (starting 9/2015) Stanford Graduate School of Education • Medical Research: What is the background for this study? What are the main findings? • Response: Earlier research has started to illuminate which genetic variants are associated with educational attainment. Subsequent work has taken these variants, combined them into a “polygenic score”, and studied how that polygenic score predicts educational attainment. Our research continues this line of inquiry by examining the predictive performance of that polygenic score in a representative sample of US adults who are now in their 30s. A few notable findings include that: • (A) the polygenic score predicts educational attainment in the African Americans in our sample and • (B) that the polygenic score is associated with neighborhood characteristics. As with earlier research, we are able to show that the higher score sibling from within a family will complete more years of schooling (on average) than their lower score co-sib. Read the rest of the interviews on MedicalResearch.com Content NOT an endorsement of efficacy and NOT intended as specific medical advice.
  • 62. Polygenetic Score Predicts Educational Attainment MedicalResearch.com Interview with: Ben Domingue Assistant Professor (starting 9/2015) Stanford Graduate School of Education • Medical Research: What should clinicians and patients take away from your report? • Response: Our study adds to the growing body of knowledge about how genetics predict outcomes that we care about. Although it is unlikely that this score would be used as a basis for intervention, our results may serve as a useful basis for comparison for those polygenic scores (e.g., for cardiovascular disease) which may be of clinical utility. Read the rest of the interviews on MedicalResearch.com Content NOT an endorsement of efficacy and NOT intended as specific medical advice.
  • 63. Polygenetic Score Predicts Educational Attainment MedicalResearch.com Interview with: Ben Domingue Assistant Professor (starting 9/2015) Stanford Graduate School of Education • Medical Research: What recommendations do you have for future research as a result of this study? • Response: As a social scientist, my attention will now be shifting towards research focusing on how the predictive performance of this score may function as a product of the environment in which a focal individual resides. • Citation: • W. Domingue, D. W. Belsky, D. Conley, K. M. Harris, J. D. Boardman.Polygenic Influence on Educational Attainment: New Evidence From the National Longitudinal Study of Adolescent to Adult Health. AERA Open, 2015; 1 (3) DOI: 10.1177/2332858415599972 Read the rest of the interviews on MedicalResearch.com Content NOT an endorsement of efficacy and NOT intended as specific medical advice.
  • 64. Extreme Obesity Can Cause Kidney Disease Especially In Elderly MedicalResearch.com Interview with: Dr. Csaba P. Kovesdy, MD Professor of Medicine University of Tennessee Health Science Center Chief of Nephrology Memphis Veterans Affairs Medical Center • Medical Research: What is the background for this study? What are the main findings? • Dr. Kovesdy: Obesity has reached epidemic proportions in modern societies, and has been linked to adverse outcomes such as diabetes mellitus, hypertension, cardiovascular disease and mortality. In addition, obesity is also associated with chronic kidney disease through a variety of mechanisms. Our population is ageing, and previous studies have suggested that the effect of obesity on certain outcomes like mortality may be different in older vs. younger individuals, but this has not been previously examined for chronic kidney disease. We have this examined the association of granular BMI categories with progressive loss of kidney function in a very large cohort of patients with normal estimated GFR in patients of different ages. We found that the association of a BMI of >30 kg/m2 with progressive loss of kidney function was not present in younger individuals (< 40 years of age), and increased as people aged, with >80 years old displaying the strongest associations between obesity and loss of kidney function. In addition to this we also examined the association of BMI with mortality in different age groups, and found uniform U-shaped associations that did not vary by age. Read the rest of the interviews on MedicalResearch.com Content NOT an endorsement of efficacy and NOT intended as specific medical advice.
  • 65. Extreme Obesity Can Cause Kidney Disease Especially In Elderly MedicalResearch.com Interview with: Dr. Csaba P. Kovesdy, MD Professor of Medicine University of Tennessee Health Science Center Chief of Nephrology Memphis Veterans Affairs Medical Center • Medical Research: What should clinicians and patients take away from your report? • Dr. Kovesdy: These results reinforce the widely held notion that extreme obesity can cause kidney damage. The novelty of these data is that it seems that oldest patients are most prone to such damage. This may be because ageing kidneys are more vulnerable to any kind of insult, or because older individuals have been exposed to the ill-effects of obesity for a much longer time, experiencing cumulative damage. Read the rest of the interviews on MedicalResearch.com Content NOT an endorsement of efficacy and NOT intended as specific medical advice.
  • 66. Extreme Obesity Can Cause Kidney Disease Especially In Elderly MedicalResearch.com Interview with: Dr. Csaba P. Kovesdy, MD Professor of Medicine University of Tennessee Health Science Center Chief of Nephrology Memphis Veterans Affairs Medical Center • Medical Research: What recommendations do you have for future research as a result of this study? • Dr. Kovesdy: The observational nature of our data does not allow us to make concrete practice recommendations; that would require clinical trials of weight loss. Our data raises interesting questions about when weight loss interventions should commence: if the kidney damage in older individuals is due to the combined effects of age-related changes and obesity-related pathophysiology, then interventions could be effective at any age, and especially when started in the oldest individuals. If, on the other hand, kidney damage is more of a function of cumulative damage experienced over many decades, then interventions started at earlier ages, and maintained for a long time would be most effective. This dilemma will have to be answered by future interventional studies. • Another important practical point is the ideal BMI which is associated with the best clinical outcomes. In our population this was a BMI of 25-30 kg/m2. This range has been supported by several other studies in the recent past, and suggests that current guidelines about what an ideal BMI should be may need to be re-evaluated in certain groups of patients. • Citation: • Association of age and BMI with kidney function and mortality: a cohort study • Jun Ling Lu, MD, Miklos Z Molnar, MD, Adnan Naseer, MD,Margit K Mikkelsen, Prof Kamyar Kalantar-Zadeh, MD,ProfCsaba P Kovesdy, MD • The Lancet Diabetes & Endocrinology Published Online: 30 July 2015 • DOI: http://dx.doi.org/10.1016/S2213-8587(15)00128-X Read the rest of the interviews on MedicalResearch.com Content NOT an endorsement of efficacy and NOT intended as specific medical advice.
  • 67. Mental and Physical Health Issues Common In Caregivers of Stroke Survivors MedicalResearch.com Interview with: Josefine Persson Doctoral student Institute of Neuroscience and Physiology Sahlgrenska Academy University of Gothenburg • Medical Research: What is the background for this study? • Response: Stroke is a major global disease that requires extensive care and support from the society and the family. We know from previous research that a stroke often has a wide- spread impact on the daily life of the family. To provide support to a partner is often perceived as natural and important, but can be demanding and have an impact on the spouses own health. The situation for spouses as caregivers is well studied during the first two years after the stroke, while the long-term effects are less well known. By this, we studied the physical and mental health of 248 spouses of stroke survivors, below age 70 at stroke onset, seven years after the stroke event and compared our result with 245 spouses of non-stroke, age- and sex-matched controls. Read the rest of the interviews on MedicalResearch.com Content NOT an endorsement of efficacy and NOT intended as specific medical advice.
  • 68. Mental and Physical Health Issues Common In Caregivers of Stroke Survivors MedicalResearch.com Interview with: Josefine Persson Doctoral student Institute of Neuroscience and Physiology Sahlgrenska Academy University of Gothenburg • Medical Research: What are the main findings? • Response: The main finding of our study is that caregiver spouses of stroke survivors are at an increased risk of mental and physical health issues even seven years after stroke onset. This is the first study with this long period of follow up and the results show that the restriction on the spouses own activity and social relationships studied in shorter follow up is also obvious for a large proportion of the spouses in a very long perspective. Spouses’ quality of life was most adversely affected by their partners’ level of disability, cognitive difficulties and depressive symptoms. Read the rest of the interviews on MedicalResearch.com Content NOT an endorsement of efficacy and NOT intended as specific medical advice.
  • 69. Mental and Physical Health Issues Common In Caregivers of Stroke Survivors MedicalResearch.com Interview with: Josefine Persson Doctoral student Institute of Neuroscience and Physiology Sahlgrenska Academy University of Gothenburg • Medical Research: What should clinicians and patients take away from your report? • Response: The findings of this study suggest the importance for long-lasting, target societal support to the spouses to prevent or reduce the burden, perceived stress and strain which might contribute to health benefits, strength to fulfil goals in work and education reduced risk of disease. In addition, our study indicates that the level of depression and level of disability of the stroke survivor is associated with the spouses’ physical and mental health, which strengthens the need for adequate workarounds for rehabilitation and follow-up including spouses. • Furthermore, our results also show that the spouses of stroke survivors reported lower general health than the spouses of controls, which might be due to perceived stress or strain for a long period, or due to shared lifestyle factors. Hence, it is important for the society to provide support to prevent or reduce the burden on spouses and health promotion to prevent unhealthy lifestyle. A societal effort could be aimed to reduce the burden on spouses and to provide health promotion against an unhealthy lifestyle. Read the rest of the interviews on MedicalResearch.com Content NOT an endorsement of efficacy and NOT intended as specific medical advice.
  • 70. Mental and Physical Health Issues Common In Caregivers of Stroke Survivors MedicalResearch.com Interview with: Josefine Persson Doctoral student Institute of Neuroscience and Physiology Sahlgrenska Academy University of Gothenburg • Medical Research: What recommendations do you have for future research as a result of this study? • Response: The spouses who are at risk are those who experience a supporting role towards the stroke survivors, especially if the spouses have experienced lack of social support. Further research is needed to explore the demanding situation for the supporting spouses and to investigate the quantity of time spent on providing support as well as which components it contains. A key question is to understand how the caregiver role impacts the spouses own health and how information and support can positively affect their own health and cohabitation. • Citation: • Spouses of Stroke Survivors Report Reduced Health-Related Quality of Life Even in Long-Term Follow-Up: Results From Sahlgrenska Academy Study on Ischemic Stroke • Josefine Persson, Lukas Holmegaard, Ingvar Karlberg, Petra Redfors,Katarina Jood, Christina Jern, Christian Blomstrand, and Gunilla Forsberg-Wärleby • Stroke. 2015;STROKEAHA.115.009791published online before print August 20 2015, doi:10.1161/STROKEAHA.115.009791 Read the rest of the interviews on MedicalResearch.com Content NOT an endorsement of efficacy and NOT intended as specific medical advice.
  • 71. Fortunately, Good Moods Are Contagious But Depression Is Not MedicalResearch.com Interview with: Edward Hill PhD student Centre for Complexity Science Member of the Warwick Infectious Disease Epidemiology Research Centre (WIDER) at the University of Warwick • Medical Research: What is the background for this study? • Response: Depression is a major public health concern worldwide. We know social factors, such as living alone, can influence whether someone becomes depressed. We also know that social support (having people to talk to) is important for recovery from depression. • Our study is slightly different as we looked at the effect of being friends with people on whether you are likely to develop depression or recover from being depressed. To do this, we looked at over 2,000 adolescents in a network of US high school students to see how their mood influenced each other. Read the rest of the interviews on MedicalResearch.com Content NOT an endorsement of efficacy and NOT intended as specific medical advice.
  • 72. Fortunately, Good Moods Are Contagious But Depression Is Not MedicalResearch.com Interview with: Edward Hill PhD student Centre for Complexity Science Member of the Warwick Infectious Disease Epidemiology Research Centre (WIDER) at the University of Warwick • Medical Research: What are the main findings? • Response: Our study revealed that having mentally healthy friends can help someone recover from depression or even remain mentally healthy in the first place. But crucially, having depressed friends does not make you more likely to become depressed yourself. In other words, it is healthy mood that spreads through social networks but depression does not. • In particular, an individual with healthy mood having five or more friends with a healthy mood can halve their probability of developing depression over a six-to-12 month period, compared to those with no friends with healthy mood. Over a similar time period, a depressed individual having ten healthy-mood friends doubles their probability of recovering from depression compared to those who have just three such friends. Read the rest of the interviews on MedicalResearch.com Content NOT an endorsement of efficacy and NOT intended as specific medical advice.
  • 73. Fortunately, Good Moods Are Contagious But Depression Is Not MedicalResearch.com Interview with: Edward Hill PhD student Centre for Complexity Science Member of the Warwick Infectious Disease Epidemiology Research Centre (WIDER) at the University of Warwick • Medical Research: What should clinicians and patients take away from your report? • Response: A potential way to tackle depression among young people would be to simply encourage friendships.Having depressed friends does not put an individual at risk of getting depression themselves, but having healthy friends is both protective and curative. If every teenager had more friends it could go some way to providing them with a protective effect against mental health problems. This also supports the idea that depression should be a public health issue rather than an individual one and should be used to help counter the stigma attached to the condition. Read the rest of the interviews on MedicalResearch.com Content NOT an endorsement of efficacy and NOT intended as specific medical advice.
  • 74. Fortunately, Good Moods Are Contagious But Depression Is Not MedicalResearch.com Interview with: Edward Hill PhD student Centre for Complexity Science Member of the Warwick Infectious Disease Epidemiology Research Centre (WIDER) at the University of Warwick • Medical Research: What recommendations do you have for future research as a result of this study? • Response: In the context of mood, it is important to stress that we do not believe that the analogy with viral spreading is literally true, but rather that it emerges from the behaviour of a complex social system. Future work could therefore try to look in more detail at how different aspects of mood are influenced by friends to understand this phenomenon better. • Furthermore, our methods could be used in other areas of public-health importance, investigating if smoking or obesity spreads for example. • Citation: • Spreading of healthy mood in adolescent social networks • E. M. Hill, F. E. Griffiths, T. House • Published 19 August 2015. DOI: 10.1098/rspb.2015.1180 Read the rest of the interviews on MedicalResearch.com Content NOT an endorsement of efficacy and NOT intended as specific medical advice.
  • 75. Chemotherapy and Radiation For Brain Cancer Lead To Brain Shrinkage MedicalResearch.com Interview with: Jorg Dietrich, MBA MMSc MD PhD Director, Cancer & Neurotoxicity Clinic and Brain Repair Research Program Massachusetts General Hospital Cancer Center Assistant Professor of Neurology Harvard Medical School • Medical Research: What is the background for this study? What are the main findings? • Dr. Dietrich: Understanding the adverse effects associated with cancer therapy is an important issue in oncology. Specifically, management of acute and delayed neurotoxicity of chemotherapy and radiation in brain cancer patients has been challenging. There is an unmet clinical need to better characterize the effects of standard cancer therapy on the normal brain and to identify patients at risk of developing neurotoxicity. In this regard, identifying novel biomarkers of neurotoxicity is essential to develop strategies to protect the brain and promote repair of treatment-induced damage. • In this study, we demonstrate that standard chemotherapy and radiation in patients treated for glioblastoma is associated with progressive brain volume loss and damage to gray matter – the area of the brain that contains most neurons. • A cohort of 14 patients underwent sequential magnetic resonance imaging studies prior to, during and following standard chemoradiation to characterize the pattern of structural changes that occur as a consequence of treatment. Read the rest of the interviews on MedicalResearch.com Content NOT an endorsement of efficacy and NOT intended as specific medical advice.
  • 76. Chemotherapy and Radiation For Brain Cancer Lead To Brain Shrinkage MedicalResearch.com Interview with: Jorg Dietrich, MBA MMSc MD PhD Director, Cancer & Neurotoxicity Clinic and Brain Repair Research Program Massachusetts General Hospital Cancer Center Assistant Professor of Neurology Harvard Medical School • Medical Research: What should clinicians and patients take away from your report? • Dr. Dietrich: The effects of chemotherapy and radiation on the brain remain poorly understood. With an increasing number of long-term survivors, it is increasingly important to better understand how cancer therapy affects the nervous system and to identify patients at highest risk to develop neurotoxic symptoms. • The findings from this study should not discourage the use of chemotherapy and radiation, as these remain the standard of care to fight brain cancer. In contrast, the results from this study should promote further investigations in this field to minimize the harmful effects of existing treatments on the brain. • Importantly, understanding the mechanisms of neurotoxicity will be the basis for development of more selective therapies designed to avoid harmful effects on normal tissues, and to improve quality of life of patients. Read the rest of the interviews on MedicalResearch.com Content NOT an endorsement of efficacy and NOT intended as specific medical advice.
  • 77. Chemotherapy and Radiation For Brain Cancer Lead To Brain Shrinkage MedicalResearch.com Interview with: Jorg Dietrich, MBA MMSc MD PhD Director, Cancer & Neurotoxicity Clinic and Brain Repair Research Program Massachusetts General Hospital Cancer Center Assistant Professor of Neurology Harvard Medical School • Medical Research: What recommendations do you have for future research as a result of this study? • Dr. Dietrich: It remains unclear whether the structural changes that we have observed in our study are associated with neurocognitive symptoms, such as memory impairment. This is the subject of ongoing investigations. Future studies will have to demonstrate how structural changes in the brain are linked to functional outcomes, such as memory problems, cognitive impairment and mood changes. • Citation: • Standard chemoradiation for glioblastoma results in progressive brain volume loss • Morgan J. Prust, MD, Kourosh Jafari-Khouzani, PhD, Jayashree Kalpathy-Cramer, PhD, Pavlina Polaskova, MD,Tracy T. Batchelor, MD, Elizabeth R. Gerstner, MD and Jorg Dietrich, MD, PhD • Published online before print July 24, 2015, • doi: http:/ / dx. doi. org/ 10. 1212/ WNL. 0000000000001861 • Neurology 10.1212/WNL.0000000000001861 Read the rest of the interviews on MedicalResearch.com Content NOT an endorsement of efficacy and NOT intended as specific medical advice.
  • 78. Up to 70% of Marijuana Users Also Use Tobacco With Complex Results MedicalResearch.com Interview with: Francesca M. Filbey PhD School of Behavioral and Brain Sciences Center for Brain Health University of Texas at Dallas Dallas, TX • Medical Research: What is the background for this study? What are the main findings? • Dr. Filbey: Most studies exclude tobacco users from participating, but 70% of marijuana users also use tobacco. We were interested in investigating the combined effects of marijuana and tobacco. Our research targeted the hippocampus because smaller hippocampal size is associated with marijuana use. We chose to study short term memory because the hippocampus is an area of the brain associated with memory and learning. The main finding was surprising. The smaller the hippocampus in the marijuana plus nicotine user, the greater the memory performance. We expected the opposite, which was true of the non-using control group. Read the rest of the interviews on MedicalResearch.com Content NOT an endorsement of efficacy and NOT intended as specific medical advice.
  • 79. Up to 70% of Marijuana Users Also Use Tobacco With Complex Results MedicalResearch.com Interview with: Francesca M. Filbey PhD School of Behavioral and Brain Sciences Center for Brain Health University of Texas at Dallas Dallas, TX • Medical Research: What should clinicians and patients take away from your report? • Dr. Filbey: It is important to note that substances interact with each other, and, therefore, combined use could lead to complex effects. More research needs to be done to fully understand the effects of marijuana on the brain. Very rarely are individuals using one type of substance and it is very difficult to generalize findings from these studies across the board. It is hard to know what the effects are of marijuana, for instance, if we don’t take into account other substance use such as tobacco. Read the rest of the interviews on MedicalResearch.com Content NOT an endorsement of efficacy and NOT intended as specific medical advice.
  • 80. Up to 70% of Marijuana Users Also Use Tobacco With Complex Results MedicalResearch.com Interview with: Francesca M. Filbey PhD School of Behavioral and Brain Sciences Center for Brain Health University of Texas at Dallas Dallas, TX • Medical Research: What recommendations do you have for future research as a result of this study? • Dr. Filbey: Future studies should consider directly examining the combined effects of these highly prevalently co-used substances. More research is also needed to further characterize the relationship between the hippocampus and memory in combined users – for instance brain connectivity, and the role of other brain regions such as the prefrontal cortex. • Citation: • Francesca M. Filbey, Tim McQueeny, Shrinath Kadamangudi, Collette Bice, Ariel Ketcherside. Combined effects of marijuana and nicotine on memory performance and hippocampal volume. Behavioural Brain Research, 2015; 293: 46 DOI: 10.1016/j.bbr.2015.07.029 Read the rest of the interviews on MedicalResearch.com Content NOT an endorsement of efficacy and NOT intended as specific medical advice.
  • 81. Commercial Brain Training Device Reduces Working Memory MedicalResearch.com Interview with: Laura Steenbergen, MSc., PhD Candidate Cognitive Psychology at Institute of Psychology Leiden University • Medical Research: What is the background for this study? What are the main findings? • Response: A recent initiative supported by several eminent research institutes and scientists calls for a more critical and active role of the scientific community in evaluating the sometimes far-reaching, sweeping claims from the brain training industry with regard to the impact of their products on cognitive performance. tDCS is a noninvasive brain stimulation technique that has developed into a promising tool to boost human cognition. Previous studies using medical tDCS devices have shown that tDCS promotes working memory (WM) updating in healthy individuals and patients. The aim of the current study was to investigate whether the commercial tDCS headset foc.us (v.1), which is easily and freely available to anyone in the world, does in fact improve cognitive performance, as advertised in the media. Results showed that active stimulation with the commercial device, compared to sham stimulation, significantly decreased working memory performance. The device we tested is just one example of a commercial device that can easily be purchased and, without any control or expert knowledge, used by anyone. The results of our study are straightforward in showing that the claims made by companies manufacturing such devices need to be validated. Even if the consequences of long-term or frequent use of the device are yet to be demonstrated, our findings provide strong support the important role of the scientific community in validating and testing far reaching claims made by the brain training industry. Read the rest of the interviews on MedicalResearch.com Content NOT an endorsement of efficacy and NOT intended as specific medical advice.